Quantcast
Channel: Learn CBSE
Viewing all 7904 articles
Browse latest View live

CBSE Class 9 Hindi A पत्र लेखन

$
0
0

CBSE Class 9 Hindi A लेखन कौशल पत्र लेखन

औपचारिक पत्र
इस श्रेणी के पत्रों को कई उपवर्गों में बाँटा जा सकता है; जैसे –

  • प्रधानाचार्य को लिखा जाने वाला प्रार्थना पत्र
  • कार्यालयों को लिखा जाने वाला प्रार्थना पत्र
  • शिकायत सुझाव संबंधी पत्र
  • संपादकीय पत्र
  • आवेदन पत्र
  • अन्य पत्र

इन पत्रों को लिखने का ढंग तथा इनकी भाषा-शैली में अंतर होता है। इन पत्रों का प्रारूप समझ लेने से पत्र लेखन में सुविधा होती है तथा परीक्षा में पूरे अंक लाए जा सकते हैं। आइए इनके प्रारूप और उदाहरण देखते हैं।

1. प्रधानाचार्य को लिखे जाने वाले पत्र का प्रारूप
CBSE Class 9 Hindi A पत्र लेखन - 1

प्रधानाचार्य के नाम पत्रों के कुछ उदाहरण

1. आप ज्वरग्रस्न हैं। डॉक्टर ने आपको तीन दिन आराम करने की सलाह दी है। इसका उल्लेख करते हुए अपने विद्यालय
के प्रधानाचार्य को अवकाश हेतु प्रार्थना पत्र लिखिए।
उत्तरः

सेवा में प्रधानाचार्य जी
रा.व.मा. बाल विद्यालय नं. 1
श्रीनिवासपुरी, दिल्ली।
विषय-ज्वरग्रस्त होने पर अवकाश के संबंध में।

महोदय,

सविनय निवेदन यह है कि मैं आपके विद्यालय की IX कक्षा का छात्र हूँ। परसों विद्यालय से घर जाते समय मैं भीग गया था। इससे मुझे कल शाम से अचानक तेज़ बुखार आ रहा है। डॉक्टर ने मुझे दवाओं के साथ तीन दिन का आराम करने की सलाह दी है ताकि मैं पूरी तरह ठीक हो सकूँ।

आपसे प्रार्थना है कि मेरी अस्वस्थता को ध्यान में रखते हुए मुझे तीन दिन का अवकाश देने की कृपा करें।

सधन्यवाद
आपका आज्ञाकारी शिष्य
विनय वर्मा
IX A अनु० 25
22 अगस्त, 20XX

2. आपके विद्यालय में पीने के पानी की व्यवस्था अच्छी नहीं है। छात्रों की धक्का-मुक्की में एक छात्र गिरकर चोटिल हो गया। इसका उल्लेख करते हुए अपने विद्यालय के प्रधानाचार्य को पीने के पानी की व्यवस्था ठीक करवाने हेतु प्रार्थना पत्र लिखिए।
उत्तरः

सेवा में
प्रधानाचार्य जी
राजकीय सर्वोदय विद्यालय
सेक्टर 15, रोहिणी
दिल्ली।
विषय-पीने के पानी की अव्यवस्था के संबंध में।

महोदय

सविनय निवेदन यह है कि इस विद्यालय की नौवीं कक्षा का छात्र हूँ। इस विद्यालय में लगभग ढाई हज़ार छात्र-छात्राएँ अध्ययनरत हैं। इतने छात्रों के लिए पीने के पानी की व्यवस्था के रूप में तीन टोंटियाँ लगी हैं जिससे हर समय यहाँ भीड़ लगी रहती है। यहाँ अक्सर छात्रों को धक्का-मुक्की करते देखा जा सकता है। प्रायः बड़ी कक्षा के छात्र छोटे बच्चों को किनारे करके खुद पानी पीने की जल्दी में रहते हैं। परसों ही किसी बड़े छात्र के धक्के से छठी कक्षा का छात्र गिर गया। इससे उसका हाथ टूट गया और उसे अस्पताल ले जाना पड़ा।

आपसे प्रार्थना है कि हम छात्रों की समस्या को ध्यान में रखते हुए नई टंकी रखवाने एवं टोटियों की संख्या बढ़ाने का कष्ट करें।

सधन्यवाद
आपका आज्ञाकारी शिष्य
क्षितिज शर्मा
IX B अनु. 15
10 जुलाई, 20xx

3. आपके विद्यालय में खेल सुविधाएँ बहुत कम हैं। अपने विद्यालय के प्रधानाचार्य को पत्र लिखिए, जिसमें खेल संबंधी सुविधाएँ उपलब्ध कराने की प्रार्थना की गई हो।
उत्तरः

सेवा में
प्रधानाचार्य महोदय
रा.व.मा. बाल विदयालय
मंगोलपुरी दिल्ली।
विषय-खेल संबंधी असुविधाओं के संबंध में।

महोदय

मैं आपके विद्यालय की नौवीं कक्षा का छात्र हूँ। मैं आपका ध्यान इस विद्यालय में खेल संबंधी कमियों की ओर आकर्षित कराना चाहता हूँ।

श्रीमान, बरसात के बाद हमारे विद्यालय के खेल का मैदान जगह-जगह के कारण मच्छर एवं अन्य कीट-पतंगों की भरमार हो गई है जिससे वहाँ खेला नहीं जा सकता है। इसके अलावा यहाँ खेल के सामानों की घोर कमी है। इससे खेल-पीरियड में हमें माँगने पर सामान नहीं मिल पाता है। जो सामान मिलते हैं वे दयनीय स्थिति में होते हैं। इससे हम छात्र खेलने से वंचित रह जाते हैं और हम चाहकर भी खेल प्रतियोगिताओं में कोई पदक नहीं ला पाते हैं।

अतः आपसे प्रार्थना है कि खेलों का नया सामान मँगवाने के अलावा खेल के मैदान की दशा सुधारने के लिए आवश्यक कदम उठाने की कृपा करें।

धन्यवाद
भवदीय
तुषार कुमार
IX ‘अ’ अनु-10
13 अगस्त, 20XX

4. पंद्रह दिनों तक लगातार अनुपस्थित रहने के कारण आपका नाम काट दिया गया है। अपनी अनुपस्थिति का उचित कारण बताते हुए पुनः प्रवेश के लिए अपने विद्यालय के प्रधानाचार्य को प्रार्थना पत्र लिखिए।
उत्तरः

सेवा में
प्रधानाचार्य महोदय
नंद नगरी, दिल्ली।
विषय-पुनः प्रवेश के संबंध में

श्रीमान जी

विनम्र निवेदन यह है कि मैं इस विद्यालय की नौंवी कक्षा का छात्र हूँ। लगभग बीस दिन पहले मेरे दादा जी की तबीयत अचानक खराब हो गई। उन्हें अस्पताल में भरती करवाना पड़ा। दुर्भाग्य से उस समय पिता जी अपनी कंपनी के काम से लखनऊ गए थे और कई दिन बाद लौट सके। ऐसे में दादा जी की देखभाल के लिए मुझे ही अस्पताल में रुकना पड़ा पर मैं इसकी सूचना कक्षाध्यापक को न दे सका। कल ही दादा जी को अस्पताल से लेकर मैं घर आया। आज विद्यालय आने पर ज्ञात हुआ कि लगातार अनुपस्थित रहने के कारण मेरा नाम काटा जा चुका है।

श्रीमान जी, मैं अपनी पढ़ाई जारी रखना चाहता हूँ। आपसे प्रार्थना है कि परिस्थितियों पर सहानुभूतिपूर्वक विचार करते हुए मुझे पुनः प्रवेश लेने की अपुमति देकर कृतार्थ करें।

सधन्यवाद
आपका आज्ञाकारी शिष्य
अनुराग शर्मा
IX-सी, अनु. 23
07 अगस्त, 20XX

5. आपके विद्यालय के पुस्तकालय में हिंदी की पत्र-पत्रिकाओं की घोर कमी है, जिससे हिंदी माध्यम के छात्र पुस्तकालय जाने में अरुचि दिखाने लगे हैं। इस ओर प्रधानाचार्य का ध्यान आकृष्ट करवाते हुए प्रार्थना पत्र लिखिए।
उत्तरः

सेवा में
प्रधानाचार्य जी
नवदीय सीनियर सैकेंड्री स्कूल
नांगलोई, दिल्ली।
विषय-विद्यालय के पुस्तकालय में हिंदी पत्र-पत्रिकाओं की संख्या बढ़ाने के संबंध में।

महोदय

विनम्र निवेदन यह है कि मैं इस विद्यालय की नौवीं कक्षा का छात्र हूँ। हमारे विद्यालय का पुस्तकालय अत्यंत समृद्ध है। यहाँ तरह-तरह के विषयों की हज़ारों पुस्तकें हैं। यहाँ नियमित रूप से अनेक समाचार-पत्र और पत्रिकाएँ मँगवाई जाती हैं पर इनमें हिंदी माध्यम के समाचार पत्र और पत्रिकाओं की संख्या नगण्य है। कभी-कभी एक-दो पत्रिकाएँ मँगवाकर खानापूर्ति कर दी जाती है। यहाँ की पुरानी पत्र-पत्रिकाएँ पढ़कर हमारा जी भर गया है। इससे अब हिंदी माध्यम के छात्र पुस्तकालय आने में रुचि नहीं लेते हैं। हम छात्र चाहते हैं कि यहाँ भी चंदा माना, चंपक, लोट-पोट, बाल हंस, पराग, नंदन, सुमन सौरभ आदि पत्रिकाएँ मँगवाई जाएँ।

आपसे प्रार्थना है कि हम छात्रों की रुचि देखते हुए हिंदी की उक्त पत्रिकाएँ मँगवाने का कष्ट करें।

सधन्यवाद
आपका आज्ञाकारी शिष्य
पुष्कर शर्मा
IX ‘स’, अनु. 27
10 नवंबर 20XX

6. आपने आठवीं तक की पढ़ाई हिंदी माध्यम से की है। आपको नौवीं के जिस वर्ग में प्रवेश मिला है, उसमें अंग्रेजी माध्यम से पढ़ाई की जाती है। इससे आपकी समझ में नहीं आ रहा है। इसका उल्लेख करते हुए विद्यालय की प्रधानाचार्या को प्रार्थना पत्र लिखिए।
उत्तरः

सेवा में
प्रधानाचार्या जी
सर्वोदय कन्या विद्यालय
पुलिस लाइंस, दिल्ली
विषय-सेक्शन बदलवाने के संबंध में।

महोदया

सविनय निवेदन यह है कि मैं इस विद्यालय में नौवीं ‘अ’ कक्षा की छात्रा हूँ। मैंने आठवीं तक की पढ़ाई गाँव में रहकर हिंदी माध्यम से की है। मुझे नौवीं के जिस सेक्शन में प्रवेश दिया गया है उसमें अंग्रेजी माध्यम से पढ़ाई कराई जाती है। इस कारण पाठ्यक्रम मेरी समझ में नहीं आ रहा है और मैं पढ़ाई में लगातार पिछड़ती जा रही हूँ। विज्ञान और गणित जैसे विषयों में मुझे विशेष परेशानी का सामना करना पड़ रहा है।

अतः आपसे प्रार्थना है कि मेरी परेशानी को ध्यान में रखते हुए मेरा सेक्शन IX ‘अ’ से हिंदी माध्यम वाले सेक्शन IX ‘द’ में करने की कृपा करें ताकि पढ़ाई में अपना मन लगा सकूँ।

सधन्यवाद
आपकी आज्ञाकारिणी छात्रा
अनुपमा मौर्या
IX ‘अ’ अनु. 34
15 अप्रैल 20XX

7. आपके विद्यालय की कैंटीन में खाद्य सामग्री की घटती गुणवत्ता की ओर ध्यान आकर्षित कराते हुए अपने विद्यालय के प्रधानाचार्य को पत्र लिखिए।
उत्तरः

सेवा में
प्रधानाचार्य जी
जयहिंदी पब्लिक स्कूल
लाजपत नगर IV, दिल्ली
विषय-कैंटीन की खाद्य सामग्री की घटती गुणवत्ता के संबंध में।

महोदय

निवेदन यह है कि मैं इस विद्यालय की नौवीं कक्षा का छात्र हूँ। छात्रों को ताजा एवं पौष्टिक भोजन मिल सके, इसके लिए कैंटीन की व्यवस्था की गई थी। शुरू-शुरू में कैंटीन में मिलने वाला भोजन घर के भोजन के समान ही स्वादिष्ट एवं पौष्टिक होता था परंतु आजकल इस कैंटीन के भोजन की गुणवत्ता का स्तर गिर गया है। अब तो बस यहाँ जंक फूड की अधिकता में बाकी सब दबकर रह गया है। कभी-कभी तो बासी समोसे और बासी ब्रेड-पकौड़े ताज़ा के नाम पर बेच दिए जाते हैं जिसका प्रतिकूल असर छात्रों के स्वास्थ्य पर पड़ रहा है। ये वस्तुएँ खाकर कई छात्र बीमार भी पड़ चुके हैं।

आपसे प्रार्थना है कि आप स्वयं औचक निरीक्षण कर वास्तविकता को जानें और खाद्य सामग्री की गुणवत्ता में सुधार लाने का कष्ट करें।

धन्यवाद
आपका आज्ञाकारी शिष्य
मयंक वर्मा
IX-सी अनु-44
20 अप्रैल, 20XX

8. आपके विद्यालय में कई छात्र ऐसे हैं जो शारीरिक चुनौतियों का सामना कर रहे हैं। अपने विद्यालय के प्रधानाचार्य को प्रार्थना पत्र लिखकर ऐसे छात्रों को विशेष सुविधाएँ दिलवाने का प्रयास कीजिए।
उत्तरः

सेवा में
सरस्वती पब्लिक स्कूल
मान सरोवर गार्डन, दिल्ली
विषय-शारीरिक चुनौतियों का सामना कर रहे छात्रों को सुविधाएँ प्रदान करवाने के संबंध में।

महोदय

मैं इस विद्यालय की नौवीं कक्षा का छात्र हूँ। हमारे विद्यालय में कई छात्र ऐसे हैं, जिन्हें शारीरिक चुनौतियों का सामना करना पड़ रहा है। ये छात्र विद्यालय की ऊपरी मंजिल में लगने वाली कक्षाओं तक पहुँचने में कठिनाई महसूस करते हैं। इन छात्रों को अन्य छात्रों की मदद लेने पर विवश होना पड़ता है। इससे इनकी पढ़ाई में बाधा आती है और उनका स्वाभिमान भी आहत होता है। ऐसे छात्रों की कक्षाएँ भूतल पर लगने से इनकी परेशानियाँ कम की जा सकती हैं।

आपसे प्रार्थना है कि उक्त छात्रों की समस्याओं को ध्यान रखते इनकी कक्षाएँ भूतल पर आयोजित करवाने की कृपा करें।

सधन्यवाद
आपका आज्ञाकारी शिष्य
मनीश मौर्य
IX बी, अनु. 25
20 अप्रैल, 20XX

9. विज्ञान एवं गणित विषयों की अतिरिक्त कक्षाएँ आयोजित करवाने के लिए उचित कारण बताते हुए अपने विद्यालय के प्रधानाचार्य को प्रार्थना पत्र लिखिए।
उत्तरः

सेवा में
प्रधानाचार्य जी
रा.व.मा. बाल विद्यालय
ज्योतिनगर, दिल्ली।
विषय-विज्ञान एवं गणित विषयों की अतिरिक्त कक्षाएँ आयोजित करवाने के संबंध में।

महोदय

विनम्र निवेदन यह है कि मैं इस विद्यालय की नौवीं कक्षा का छात्र हैं। हमारी कक्षा में विज्ञान एवं गणित विषयों का पाठ्यक्रम पूरा नहीं करवाया गया है। हमारे विज्ञान शिक्षक का स्थानांतरण हुए एक महीना बीत गया है परंतु कोई अध्यापक हमें पढ़ाने नहीं आता है। इसी तरह गणित के अध्यापक करीब बीस दिन से अवकाश पर होने के कारण नहीं आ रहे हैं। इस कारण इन दोनों विषयों की पढ़ाई नहीं हो पा रही है जबकि सितंबर के दूसरे सप्ताह से हमारी एस.ए. 1 की परीक्षा शुरू हो रही है।

आपसे प्रार्थना है कि विज्ञान एवं गणित विषयों की अतिरिक्त कक्षाएँ आयोजित करवाने की कृपा करें ताकि हम छात्र परीक्षा में अनुत्तीर्ण होने से बच सकें।

सधन्यवाद
आपका आज्ञाकारी शिष्य
संचित सिंह
IX डी अनु. 20
17 अगस्त, 20XX

कार्यालयों को लिखे जाने वाले पत्रों का प्रारूप
CBSE Class 9 Hindi A पत्र लेखन - 2

कार्यालय संबंधी प्रार्थना पत्र

10. रेल में यात्रा करते समय आपका सामान चोरी हो गया था। खोए हुए सामान की जानकारी देते हुए सुरक्षा आयुक्त, मंडल कार्यालय नई दिल्ली को पत्र लिखिए।
उत्तरः

सेवा में
सुरक्षा आयुक्त
मंडल कार्यालय
नई दिल्ली।
विषय-खोए हुए सामान के संबंध में।

महोदय

निवेदन यह है कि कल इलाहाबाद से नई दिल्ली आते समय प्रयागराज एक्सप्रेस में मेरा सूटकेस चोरी हो गया है। गाज़ियाबाद स्टेशन के बाद चोरी की यह घटना घटी। उस समय मैं अगले कोच में मित्र से मिलने चला गया था। काले रंग के इस वी.आई. पी. सूटकेस में मेरा पहचान पत्र, डायरी, पाँच हज़ार रुपये नकद तथा कुछ अन्य आवश्यक कागजात थे।

आपसे प्रार्थना है कि इस खोए सूटकेस की सूचना दर्ज कर इसे वापस दिलाने में मेरी मदद करें और सूटकेस मिलते ही नीचे दिए गए पते पर सूचित करने का कष्ट करें।

धन्यवाद
भवदीय
मोहन शर्मा
ए. 7/312
सेक्टर 9. द्वारका
दिल्ली।
05 सितंबर, 20XX

11. अपने क्षेत्र के डाकिए को सम्मानित करने के लिए मुख्य डाक अधीक्षक प्रधान डाकघर नई दिल्ली को पत्र लिखिए जिसमें उसके किसी उल्लेखनीय कार्य का उल्लेख हो।
उत्तरः

सेवा में
मुख्य डाक अधीक्षक
प्रधान डाकघर
कनाट प्लेस, नई दिल्ली।
विषय-क्षेत्र के डाकिए को सम्मानित करने के संबंध में।

महोदय

निवेदन यह है कि मैं नाईवाला बाग करोलबाग का निवासी हूँ। मैंने और आसपास के कई मित्रों ने बैंकिंग भर्ती बोर्ड लखनऊ
द्वारा विज्ञापित सहायक प्रबंधक पद हेतु आवेदन किया था, जिसका प्रवेशपत्र हमें नहीं मिल पाया था। हम परेशान एवं निराश हो चुके थे क्योंकि अगले दिन रविवार को ही परीक्षा थी। शाम को छह बजने वाले थे कि हमारे क्षेत्र का डाकिया आता दिखाई दिया। उसने घर आकर मुझे प्रवेश पत्र दिया। मैं भागकर अपने मित्रों को बुलाया। उसने प्रवेश पत्र दिए और कहा, “यह मुझे आज ही पाँच बजे के बाद मिला है, पर प्रवेश पत्र जैसा महत्त्वपूर्ण कागज कैसे रोक सकता था और देने चला आया। ड्यूटी के बाद भी ड्यूटी करना ऐसा सरकारी कर्मचारियों में कम ही देखने को मिलता है जिसके कारण हम परीक्षा दे सके।

मैं आपसे प्रार्थना करता हूँ कि इन्हें (श्री श्याम शरण) को उनकी कर्तव्यनिष्ठा के लिए सम्मानित करने की कृपा करें।

सधन्यवाद
भवदीय
मंजीत कुमार
27/3 नाईवाला बाग
करोलबाग दिल्ली
09 सितंबर 20XX

12. स्टेट बैंक ऑफ इंडिया की मानसरोवर गार्डेन शाखा के प्रबंधक को प्रार्थना पत्र लिखिए जिसमें खाता खोलने का अनुरोध किया गया हो।
उत्तरः

सेवा में
प्रबंधक महोदय
भारतीय स्टेट बैंक
शाखा-मानसरोवर गार्डेन
दिल्ली।
विषय-नया खाता खोलने के संबंध में।

महोदय

विनम्र निवेदन यह है कि मैं भारतीय स्टेट बैंक आफ इंडिया की इस शाखा में खाता खुलवाना चाहता हूँ। सरकार द्वारा छात्रों को प्रदत्त की जाने वाली आर्थिक सहायता अब विद्यालयों में नकद न देकर सीधे उनके खाते में स्थानांतरित की जाएगी। इस कारण मेरा भी खाता होना आवश्यक है। इसके लिए मैं आवश्यक कागजात भी जमा करा हूँ।
आपसे प्रार्थना है कि आवश्यक कार्यवाही पूरी करते हुए इस बैंक में मेरा भी खाता खोलने की कृपा करें। मैं आपका आभारी रहूँगा।

सधन्यवाद
भवदीय
उत्कर्ष सिंह
सी-5/135
मानसरोवर गार्डेन
दिल्ली
06 सितंबर, 20XX

संलग्नक –

  • तीन फ़ोटो
  • आधार कार्ड (फोटो प्रति)
  • जन्मतिथि प्रमाणपत्र (फोटो प्रति)
  • राशन कार्ड (फोटो प्रति)
  • पिता जी का पहचान पत्र (फोटो प्रति)
  • बिजली का बिल (फोटो प्रति)

13. आप जिस नई कालोनी में रहते हैं, वहाँ से बस की कोई सुविधा नहीं है। इसका अनुचित फायदा प्राइवेट बस वाले उठाते हैं। क्षेत्रवासियों को होने वाली असुविधा का उल्लेख करते हुए वजीरपुर डिपो के महाप्रबंधक को पत्र लिखिए।
उत्तरः

सेवा में
श्रीमान महाप्रबंधक जी
दिल्ली परिवहन निगम
वजीरपुर डिपो, दिल्ली।
विषय-नई बस सेवा शुरू करने के संबंध में।

महोदय
निवेदन यह है कि मैं मुंडका से तीन-चार किलोमीटर दूर बसी कॉलोनी रानीखेड़ा का निवासी हूँ। यहाँ से दिल्ली परिवहन निगम की किसी बस का परिचालन नहीं किया जाता है। इससे इस कॉलोनी और इससे आगे बसी कॉलोनियों के निवासियों को बड़ी परेशानी का सामना करना पड़ता है। सवेरे बच्चों को स्कूल भेजना हो या यहाँ से कामकाज पर जाना अत्यंत कठिन काम हो गया है। हम निवासियों की इस विवशता का फायदा प्राइवेट वाहन वाले उठाकर मनचाहा किराया वसूल रहे हैं। यहाँ से बस सेवा शुरू करके आप इस समस्या से मुक्ति दिला सकते हैं।

आपसे प्रार्थना है कि इस कॉलोनी से नई बस सेवा शुरू करने की कृपा करें।

सधन्यवाद
भवदीय
रोहताश सिंह
3/28 बी,
रानीखेड़ा दिल्ली।
10 सितंबर, 20XX

14. अपने क्षेत्र में पार्क विकसित करने के लिए उत्तरी नगर निगम के मुख्य उद्यान निरीक्षक को पत्र लिखिए।
उत्तरः

सेवा में
उद्यान निरीक्षक महोदय
उत्तरी दिल्ली नगर निगम
टाउन हाल, दिल्ली-10006
विषय-क्षेत्र में पार्क विकसित करने के संबंध में।

महोदय

निवेदन यह है कि मैं बुराड़ी के कौशिक एन्क्लेव का निवासी हूँ। इस कॉलोनी को बसे हुए पंद्रह साल से अधिक बीत गया है। यहाँ पार्क के लिए जो जगह छोड़ी गई थी, वह आज भी वैसी ही बदहाल स्थिति में पड़ी हुई है। उपेक्षा के कारण पार्क की यह ज़मीन कूड़ेदान में परिवर्तित होती जा रही है। इस कारण बदबू और गंदगी फैल रही है। इस जगह को यदि पार्क में विकसित कर दिया जाए तो इसका सौंदर्य बढ़ने के साथ-साथ यहाँ के लोगों को सुबह-शाम घूमने और समय बिताने की जगह भी मिल जाएगी।

अतः आपसे प्रार्थना है कि खाली पड़ी इस जगह को पार्क के रूप में विकसित करने का कष्ट करें।

सधन्यवाद
भवदीय
नगेंद्र सिंह
सी-128, गली नं. 4
कौशिक एन्क्लेव, बुराड़ी
दिल्ली।
15 जुलाई 20XX

15. आपका मोबाइल फ़ोन चलती बस में चोरी हो गया है। इसकी सूचना देते हुए अपने क्षेत्र के थानाध्यक्ष को पत्र लिखिए।
उत्तरः

सेवा में
थानाध्यक्ष महोदय
उत्तम नगर, दिल्ली
विषय-चोरी हुए मोबाइल फ़ोन के संबंध में

महोदय

निवेदन यह है कि आज सवेरे पीरागढ़ी से उत्तमनगर रुट संख्या 883 की बस से जाते समय मुझे सीट नहीं मिली, इसलिए खड़े-खड़े उत्तम नगर जाना पड़ा। जनकपुरी डिस्टिक सेंटर पर कुछ लोग बस में चढ़े जिनमें चार-पाँच युवक भी थे। उन्होंने आते ही आगे बढ़ते हुए धक्का-मुक्की शुरू कर दी। इसी बीच वे अगले गेट से उतर गए। बस चली ही थी कि मैंने जेब में हाथ डाला तो मोबाइल फ़ोन गायब था। काले रंग की 5.5 इंच वाला यह फ़ोन रेडमी नोट फोर माडल का था।

आपसे प्रार्थना है कि इसकी प्राथमिक सूचना दर्ज करते हुए आवश्यक कार्यवाही करने की कृपा करें और मिलने पर नीचे दिए गए पते पर सूचित करने की कृपा करें।

सधन्यवाद
भवदीय
दीक्षांत शर्मा
सी-3/512
उत्तम नगर, दिल्ली 28 अगस्त, 20XX

(3) शिकायती पत्रों का प्रारूप
CBSE Class 9 Hindi A पत्र लेखन - 3
CBSE Class 9 Hindi A पत्र लेखन - 4
16. आपके क्षेत्र में सफाई की बदहाल स्थिति के कारण लोगों का जीना कठिन होता जा रहा है। इस ओर ध्यान आकर्षित करते हुए अपने क्षेत्र के प्रचार अधिकारी को पत्र लिखिए।
उत्तरः

315/7B
जीवन विहार
नई दिल्ली
10 जुलाई, 20XX

क्षेत्रीय स्वास्थ्य अधिकारी
उत्तरी, दिल्ली नगर निगम
टाउन हॉल, दिल्ली-110006
विषय-क्षेत्र में सफाई की बदहाल स्थिति के संबंध में।

महोदय

मैं आपका ध्यान अपने क्षेत्र जीवन विहार की सफाई की दयनीय स्थिति की ओर आकर्षित करवाना चाहता हूँ। इस क्षेत्र में सफाई के लिए नियुक्त कर्मचारी सप्ताह में एक या दो दिन ही आते हैं और सफ़ाई के नाम आधा-अधूरा काम करके जाने की जल्दबाजी में रहते हैं। वे सड़क के किनारे ही कूड़े का ढेर लगा देते हैं जिसे आवारा जानवर बिखरा देते हैं और उस पर मल-मूत्र त्यागते हैं। इससे मक्खी-मच्छरों की बाढ़-सी आ गई है। मलेरिया के बढ़ते मरीज इसके प्रमाण हैं।

आपसे प्रार्थना है कि आप इस मामले में व्यक्तिगत रुचि लेकर क्षेत्र में सफाई की व्यवस्था में सुधार लाने हेतु आवश्यक कदम उठाएँ।

धन्यवाद
भवदीय
दीपचंद शाक्य

17. आपके क्षेत्र में डेंगू, मलेरिया और स्वाइन फ्लू जैसी बीमारियाँ पैर पसारने लगी हैं। अस्पतालों में रोगियों की भरमार लगी है। चिकित्सीय सुविधाओं के अभाव की ओर ध्यान दिलाते हुए स्वास्थ्य मंत्री को पत्र लिखिए।
उत्तरः

सेवा में
माननीय स्वास्थ्य मंत्री
दिल्ली सरकार, दिल्ली
10 सितंबर, 20XX
विषय-डेंगू, मलेरिया आदि बीमारियों के इलाज की सुविधा की कमी के संबंध में

महोदय,

निवेदन यह है कि बरसात का मौसम अपने साथ कई बीमारियाँ भी लेकर आता है। साफ़-सफ़ाई और देख-रेख के अभाव में ये बीमारियाँ जानलेवा बन जाती हैं। हमारे क्षेत्र में जगह-जगह पानी भरने और गंदगी के कारण मच्छर-मक्खियों की भरमार हो गई है। इससे क्षेत्र में डेंगू, मलेरिया और स्वाइन फ्लू जैसी बीमारियाँ पैर पसारने लगी हैं। इनके इलाज के लिए अस्पतालों में मरीजों की लंबी-लंबी लाइनें लगी हुई हैं। इन अस्पतालों में चिकित्सीय सुविधाओं जैसे डॉक्टर, दवाएँ, बेड आदि की कमी के कारण कई लोग असमय मौत का शिकार बन चुके हैं। लोगों की जान बचाने के लिए चिकित्सीय सुविधाएँ बढ़ाने की तुरंत आवश्यकता है।

आपसे प्रार्थना है कि आप इस क्षेत्र के अस्पताल का आकस्मिक निरीक्षण कर वास्तविक स्थिति को जानें और चिकित्सीय सुविधाएँ बढ़ाने के लिए तुरंत निर्देश दें।

धन्यवाद सहित
भवदीय
अमन वर्मा
सी-921/3
सीमापुरी दिल्ली।

18. अपने क्षेत्र के पुलिस अधीक्षक को पत्र लिखिए जिसमें नगर में बढ़ती चोरियों पर चिंता व्यक्त की गई हो तथा उनकी रोकथाम हेतु आवश्यक कदम उठाने का अनुरोध किया गया हो।
उत्तरः

सेवा में
पुलिस अधीक्षक
गाजियाबाद, उत्तर प्रदेश
15 अक्टूबर, 20XX
विषय-मोहल्ले में बढ़ती चोरियों के संबंध में।

महोदय

मैं आपका ध्यान अपने मोहल्ले पूजा कॉलोनी में बढ़ती चोरी की घटनाओं की ओर आकर्षित करवाना चाहता हूँ।
इस मुहल्ले में चोरों की सक्रियता और चोरी की बढ़ती घटनाओं के कारण यहाँ रहने वालों की रातों की नींद और दिन का चैन छिन गया है। रविवार को यहाँ की बाज़ार में बंदी होने एवं बिजली न होने का फायदा चोरों ने उठाया और कई दुकानों के शटर काटकर चोरी कर ली। उसी रात दो घरों के रोशनदान की जाली काटकर चोर कमरे में घुस आए और लोगों को बंधक बनाकर जमकर लूटपाट की। यहाँ मोबाइल फ़ोन छीनना, चैन, पर्स झपटकर भागने की घटनाएँ सामान्य बात बन चुकी हैं। पुलिस भी इन घटनाओं के प्रति उदासीन नज़र आती है।

आपसे प्रार्थना है कि इस मामले में तुरंत हस्तक्षेप करते हुए पुलिस गस्त बढ़ाने एवं अन्य आवश्यक कदम उठाने की कृपा करें।

सधन्यवाद
भवदीय
करतार सिंह
27-C पूजा कालोनी
गाजियाबाद, उत्तर प्रदेश।

19. अपने क्षेत्र के विद्युत प्रदाय संस्थान के महाप्रबंधक को पत्र लिखिए जिसमें परीक्षा के दिनों में बिजली की बार-बार कटौती से उत्पन्न समस्या का वर्णन हो।
उत्तरः

परीक्षा भवन
नई दिल्ली
15 सितंबर 20XX
महाप्रबंधक महोदय
एन.डी.पी.एल.
शालीमार बाग, दिल्ली।
विषय-बिजली कटौती से होने वाली परेशानी के संबंध में

महोदय

मैं आपका ध्यान अपने मुहल्ले सिंगलपुर गाँव में बिजली की बार-बार की जा रही अघोषित कटौती की ओर आकर्षित करवाना
चाहता हूँ।

यहाँ पिछले एक महीने से बिजली की आँख मिचौली जारी है। बिजली कब आएगी और चली जाएगी, इसका पता नहीं है। सबसे ज्यादा समस्या तो सुबह-शाम होने वाली कटौती से होती है। इधर हम छात्रों की एस.ए.वन की परीक्षाएँ आने वाली हैं। इसकी तैयारी में बिजली के बिना बहुत परेशानी हो रही है। इससे पूर्व भी हम कई बार मौखिक शिकायत कर चुके हैं, पर इसका कोई परिणाम नहीं निकला।

आपसे प्रार्थना है कि आप इस मामले में व्यक्तिगत रुचि लें तथा हम छात्रों का भविष्य और लोगों को होने वाली परेशानियों पर सहानुभूतिपूर्वक विचार करते हुए शीघ्र उचित कदम उठाएँ।

सधन्यवाद
भवदीय
कनिष्क

20. महानगर टेलीफ़ोन निगम लिमिटेड के महाप्रबंधक को पत्र लिखिए जिसमें आपकी कॉलोनी में नेटवर्क खराब रहने तथा अचानक बढ़कर आए बिल का उल्लेख किया गया हो।
उत्तरः

परीक्षा भवन
नई दिल्ली
15 दिसंबर, 20XX
महाप्रबंधक महोदय
महानगर टेलीफ़ोन निगम लिमिटेड
कनाट प्लेस, दिल्ली।
विषय-नेटवर्क खराब रहने तथा अचानक बढ़े बिल के संबंध में।

महोदय

मैं आपका ध्यान अपने नागिया पार्क मुहल्ले की एम.टी.एन.एल. सेवा की कमी और अचानक बढ़कर आए बिल की ओर आकर्षित करवाना चाहता हूँ।
एम.टी.एन.एल. की सेवा में पिछले महीने से कमी आनी शुरु हो गई जिससे फोन सुनना और फोन करना कठिन हो गया है। फोन पर आवाज़ आते-आते बंद होना, खड़खड़ाहट होना जैसी बात आम हो गई है। पता नहीं क्यों इस सेवा का नेटवर्क इतना खराब हो गया। इधर हमें जो बिल दिया गया है, उसे आठ से दस गुना तक बढ़ा दिया गया है। यह समस्या हमारी ही नहीं बल्कि पूरे नागिया पार्क की है। क्षेत्रीय कार्यालय में हमारी समस्या सुनने को कोई तैयार नहीं है। इस बढ़े बिल ने हम उपभोक ताओं का चैन छीन लिया है।।

अतः आपसे प्रार्थना है कि इस मामले में तुरंत आवश्यक कदम उठाएँ एवं सेवा में सुधार करते हुए दुबारा वास्तविक बिल प्रदान करवाएँ।

सधन्यवाद,
भवदीय
कुलदीप सैनी

21. आपने सैमसंग कंपनी का महँगा मोबाइल फ़ोन खरीदा पर आपको वैसी सेवाएँ एवं सुविधाएँ नहीं मिल पा रही हैं जिसके लिए आपने खरीदा था। उसकी कमियों का उल्लेख करते हुए ‘यंग जनरेशन टेलीकॉम’ मैनेजर को पत्र लिखिए।
उत्तरः

ए.75/5
मंदिर मार्ग, हर्षविहार
दिल्ली।
10 जुलाई 20XX
मैनेजर
यंग जेनरेशन टेलीकॉम
भजनपुरा दिल्ली।
विषय-मोबाइल फ़ोन की सेवाओं में कमी के संबंध में।

महोदय

मैं आपका ध्यान उस मोबाइल फ़ोन की ओर आकर्षित करवाना चाहता हूँ जिसके संबंध में सेल्स ब्वाय बड़ी-बड़ी बातें कर रहा था।
उसकी बताई खूबियों से आकर्षित होकर मैने सैमसंग का यह फ़ोन रु. 17999 में खरीदा है। दुख की बात यह है कि यह फ़ोन एक सप्ताह बाद से ही गरम होने लगा है। इसमें हैंग होने की समस्या भी बराबर आ रही है। यह फ़ोन अब सुविधा की जगह असुविधाजनक बन गया है।

आपसे प्रार्थना है कि इस फ़ोन की आप स्वयं जाँच करवाएँ ताकि आप इसकी स्थिति स्वयं जान सकें तथा इसे बदलकर आप दूसरा मोबाइल फ़ोन प्रदान करने की कृपा करें।

धन्यवाद
भवदीय
अक्षित कुमार

संपादकीय पत्र

संपादक के नाम लिखे जाने वाले पत्र का प्रारूप
CBSE Class 9 Hindi A पत्र लेखन - 5
CBSE Class 9 Hindi A पत्र लेखन - 7

संपादकीय पत्रों के उदाहरण

22. निरंतर बढ़ती महँगाई से जनता को हो रही परेशानियों की ओर सरकार का ध्यान आकर्षित कराने के लिए किसी दैनिक समाचार के संपादक को पत्र लिखिए।
उत्तरः

परीक्षा भवन
नई दिल्ली
10 सितंबर, 20xx
संपादक महोदय
दैनिक जागरण
एफ-62-63, सेक्टर-62
गौतम बुद्ध नगर (उ.प्र.)
विषय-महँगाई से उत्पन्न समस्याओं के संबंध में।

महोदय

आपके सम्मानित एवं लोकप्रिय पत्र के माध्यम से मैं सरकार का ध्यान निरंतर बढ़ती महँगाई से उत्पन्न लोगों की परेशानियों की ओर आकर्षित कराना चाहता हूँ।

अगस्त महीने से फल एवं सब्जियों के दामों में जो वृद्धि शुरू हुई, वह रुकने का नाम ही नहीं ले रही है। दालों के दाम पहले से ही आकाश छू रहे हैं। आटा, चीनी, चावल, मसाले सभी के दाम बढ़ते जा रहे हैं। इस महँगाई की सबसे अधिक मार जनता को झेलनी पड़ रही है, जिससे उसकी कमर टूट रही है। इस वर्ग के लिए दाल-रोटी मिलना भी मुश्किल होता जा रहा है। रही-सही कसर जमाखोर पूरी कर रहे हैं। सरकार भी उदासीन रवैया अपनाए हुए है।

आपसे प्रार्थना है कि इसे आप अपने समाचार-पत्र में छापने की कृपा करें, ताकि सरकार का ध्यान इस ओर आकृष्ट हो तथा वह महँगाई रोकने के लिए आवश्यक कदम उठाए।

धन्यवाद
भवदीय
सगुन शर्मा

23. मिलावट एवं जमाखोरी की समस्या की ओर सरकार एवं अधिकारियों का ध्यान आकर्षित करते हुए किसी समाचार संपादक को पत्र लिखिए।
उत्तरः

परीक्षा भवन
नई दिल्ली
18 सितंबर, 20XX
संपादक महोदय
नवभारत टाइम्स
बहादुर शाह जफ़र मार्ग
नई दिल्ली
विषय-बढ़ती मिलावट एवं जमाखोरी के संबंध में।

महोदय

मैं आपके सम्मानित एवं लोकप्रिय पत्र के माध्यम से जमाखोरी एवं मिलावट की समस्या की ओर सरकार एवं अधिकारियों का ध्यान आकर्षित कराना चाहता हूँ।

इन दिनों मिलावट एवं जमाखोरी की समस्या दिन दूनी-रात चौगुनी गति से बढ़ रही है। शुद्ध सामान मिलना दुर्लभ हो गया है। नैतिकता एवं ईमानदारी की कसमें खाने वाले दुकानदार लगभग हर खुले सामान में मिलावट करके खुद मोटा मुनाफा कमाते हैं पर जनता के स्वास्थ्य के साथ खिलवाड़ करते हैं। ये दुकानदार आवश्यक वस्तुओं की जमाखोरी कर महँगाई बढ़ाते हैं। मिलावट एवं जमाखोरी रोकने का दायित्व जिन अधिकारियों पर है, वे आँखें बंद किए बैठे हैं।

कृपया इसे अपने समाचार पत्र में स्थान दें, ताकि सरकार एवं अधिकारी इसे रोकने की दिशा में प्रयास करें।

धन्यवाद
भवदीय
कृतज्ञ वर्मा

24. आपके मुहल्ले को शहर से जोड़ने वाली सड़क पर जगह-जगह गड्ढे हो. गए हैं जिससे आए दिन दुर्घटनाएं होती रहती है। इस ओर ध्यान आकर्षित कराते हुए किसी दैनिक समाचार पत्र के संपादक को पत्र लिखिए।
उत्तरः

A 120/4
सेक्टर 19, द्वारका
दिल्ली 18 अगस्त, 20XX
संपादक महोदय
दैनिक जागरण
एफ-62-63, सेक्टर 62
गौतमबुद्ध नगर (उ. प्र.)
विषय-सड़क के गड्ढों के संबंध में।

महोदय

आपके सम्मानित एवं लोकप्रिय पत्र के माध्यम से मैं सरकार का ध्यान सड़क पर हुए उन गड्ढों की ओर ले जाना चाहता हूँ जो दुर्घटना का कारण बन रहे हैं।

हमारे मुहल्ले से रिंग रोड को जोड़ने वाली सड़क पर जगह-जगह गड्ढे हो गए हैं। ये गड्ढे एक-दो नहीं बल्कि हज़ारों की संख्या में हैं। इनके कारण सड़क पर चलना मुश्किल हो जाता है। बरसात के दिनों में इनमें पानी भर जाने से स्थिति और भी खराब हो जाती है। यहाँ आए दिन दुर्घटनाएं होती रहती हैं। स्थानीय नेता इस समस्या को अनदेखा किए बैठे हैं।

आपसे प्रार्थना है कि आप इसे अपने समाचार पत्र में छापने का कष्ट करें, ताकि संबंधित अधिकारियों का ध्यान इस ओर जाए और वे इसे सुधारने हेतु कदम उठाएँ।

सधन्यवाद
भवदीय
गजराज सिंह

25. आपने देखा कि आपके शहर की मुख्य सड़क पर लगी लाइटें खराब हो चुकी हैं। इस कारण रात में सड़क पर अँधेरा रहता है। इस ओर ध्यान आकर्षित कराने हेतु किसी समाचार संपादक को पत्र लिखिए।
उत्तरः

बी-124/4
गली नं. 5, संतनगर
दिल्ली
14 दिसंबर, 20XX
संपादक महोदय
पंजाब केसरी
वजीरपुर इंडस्ट्रियल एरिया
रिंग रोड, दिल्ली।
विषय-रिंग रोड पर लगी लाइटों के खराब होने के संबंध में।

महोदय

आपके सम्मानित एवं लोकप्रिय पत्र के माध्यम से मैं लोक निर्माण विभाग (सड़क) के अधिकारियों का ध्यान खराब पड़ी लाइटों की ओर ले जाना चाहता हूँ।

रिंग रोड पर लोगों के सुचारु आवागमन हेतु लाइटें लगवाई गई थीं, पर इनकी मरम्मत पर ध्यान न दिए जाने से ये लाइटें कब की खराब हो चुकी हैं। इससे इस मुख्य सड़क पर अँधेरा बना रहता है। यह अँधेरा लूटपाट और दुर्घटना का कारण बन रहा है। जिन अधिकारियों पर इसकी देख-रेख का जिम्मा हैं, वे उदासीन बने बैठे हैं।

आपसे प्रार्थना है कि इसे अपने समाचार पत्र में छापने का कष्ट करें ताकि संबंधित अधिकारी एवं कर्मचारी इस ओर ध्यान दें और इनकी मरम्मत हेतु आवश्यक कदम उठाएँ।

सधन्यवाद, भवदीय
कृपा शंकर वर्मा

(5) आवेदन पत्र ( नौकरी आदि के लिए) का प्रारूप
CBSE Class 9 Hindi A पत्र लेखन - 8
CBSE Class 9 Hindi A पत्र लेखन - 9

26. उत्तरी दिल्ली नगर निगम में प्राथमिक अध्यापकों के कुछ पद रिक्त हैं। इस पद हेतु अपनी योग्यता का उल्लेख करते हुए सहायक आयुक्त (शिक्षा) को आवेदन पत्र प्रस्तुत कीजिए।
उत्तरः

सेवा में
सहायक आयुक्त शिक्षा
उत्तरी दिल्ली नगर निगम
टाउन हाल, दिल्ली-110006
विषय-प्राथमिक अध्यापक पद हेतु आवेदन पत्र।

महोदय

11 सितंबर 20XX के नवभारत टाइम्स में प्रकाशित विज्ञापन के संबंध में मैं भी अपना आवेदन पत्र प्रस्तुत कर रहा हूँ, जिसका संक्षिप्त विवरण इस प्रकार है –
नाम – आलोक कुमार
पिता का नाम – मोहन सिंह
जन्म तिथि – 26 दिसंबर 1993
पत्र-व्यवहार का पता – A-115/4 सोमबाजार, रोड, संतनगर, दिल्ली।
संपर्क सूत्र – 011-2764……
शैक्षिक योग्यता
CBSE Class 9 Hindi A पत्र लेखन - 10
अनुभव-संत सुजान सिंह पब्लिक स्कूल में 1 वर्ष से प्राथमिक शिक्षक पद पर कार्यरत है।
घोषणा-उपर्युक्त विवरण मेरी जानकारी के अनुसार पूर्णतः सत्य है। आशा है कि मेरी योग्यता पर विचार कर आप सेवा का अवसर अवश्य देंगे।

सधन्यवाद
आवेदक – आलोक कुमार
हस्ताक्षर ……
13 सितंबर 20XX

27. पंजाब नेशनल बैंक लखनऊ में कंप्यूटर आपरेटर्स के कुछ पद रिक्त हैं। अपनी योग्यता का संक्षिप्त विवरण देते हुए मुख्य प्रबंधक को आवेदन पत्र लिखिए।
उत्तरः

सेवा में
मुख्य प्रबंधक
पंजाब नेशनल बैंक
लखनऊ, उत्तर प्रदेश
विषय-कंप्यूटर आपरेटर पद हेतु आवेदन पत्र ।

महोदय

02 सितंबर 20XX को ‘स्वतंत्र भारत’ समाचार पत्र में प्रकाशित विज्ञापन के जवाब में मैं उक्त पद हेतु अपना आवेदन-पत्र प्रस्तुत कर रहा हूँ, जिसका संक्षिप्त विवरण इस प्रकार है –
नाम – क्षितिज वर्मा
पिता का नाम – राम कृपाल वर्मा
जन्मतिथि – 10 अक्टूबर, 1994
पत्राचार का पता – 27/3 बी, स्टेशन रोड चारबाग, लखनऊ (उ.प्र.)
संपर्क सूत्र – 0536265…..

शैक्षिक योग्यता –
CBSE Class 9 Hindi A पत्र लेखन - 11

अनुभव-ग्रामीण बैंक गोमतीनगर में दो वर्ष से कंप्यूटर आपरेटर पद पर कार्यरत।
आशा है कि मेरी योग्यताओं पर विचार कर मुझे सेवा का अवसर अवश्य प्रदान करेंगे।

सधन्यवाद
आवेदक – क्षितिज वर्मा
हस्ताक्षर……
दिनांक 06 सितंबर, 20XX

28. प्रौढ़ शिक्षा निदेशालय द्वारा संचालित सायंकालीन कक्षाओं में पढ़ाने हेतु कुछ उत्साही युवकों की आवश्यकता है। अपनी योग्यता का विवरण देते हुए आवेदन पत्र प्रस्तुत कीजिए।
उत्तरः

सेवा में
निदेशक
प्रौढ़ शिक्षा निदेशालय
रोहतक हरियाणा
विषय-सायंकालीन कक्षाओं में पढ़ाने हेतु आवेदन पत्र ।

महोदय

07 जुलाई 20XX के समाचार पत्र से ज्ञात हुआ कि इस कार्यालय को कुछ ऐसे नवयुवकों की आवश्यकता है, जो सायंकालीन कक्षाओं में पढ़ा सके। इसके लिए मैं भी अपना आवेदन पत्र प्रस्तुत कर रहा हूँ।

नाम – सुनील शर्मा
पिता का नाम – राम स्वरूप शर्मा
जन्मतिथि – 15 नवंबर 1992
पत्राचार का पता – बी 322/4, रेलवे स्टेशन रोड रोहतक, हरियाणा।
संपर्क सूत्र – 0818397……

शैक्षिक योग्यता –
CBSE Class 9 Hindi A पत्र लेखन - 12

अनुभव-सैनी पब्लिक स्कूल रोहतक में तीन साल से प्राथमिक अध्यापक पद पर कार्यरत।
घोषणा-मैं सत्यनिष्ठा से कहता हूँ कि उपर्युक्त विवरण पूर्ण तथा सत्य है। यदि मुझे सेवा का अवसर प्रदान किया जाता है तो मैं अपनी सेवा से संतुष्ट एवं प्रसन्न रखूगा।

सधन्यवाद
आवेदक सुनील शर्मा
हस्ताक्षर …..
10 जुलाई, 20XX

29. दिल्ली विकास प्राधिकरण को कुछ कार्यालय सहायकों की आवश्यकता है। अपनी योग्यता का उल्लेख करते हुए प्राधिकरण के विकास अधिकारी को आप भी आवेदन पत्र प्रस्तुत कीजिए।
उत्तरः

सेवा में
विकास अधिकारी
दिल्ली विकास प्राधिकरण
दिल्ली।
विषय-कार्यालय सहायक पद हेतु आवेदन पत्र।

महोदय

15 अक्टूबर 20XX को प्रकाशित दैनिक समाचार पत्र हिंदुस्तान से ज्ञात हुआ कि आपके कार्यालय में कुछ सहायकों की आवश्यकता है। प्रार्थी भी अपना आवेदन पत्र प्रस्तुत कर रहा है, जिसका विवरण इस प्रकार है –

नाम – मयंक कुमार
पिता का नाम – प्रदीप कुमार
जन्मतिथि – 18 सितंबर, 1994
पत्राचार का पता – सी. 2/135 टैगोर, गार्डन, दिल्ली
संपर्क सूत्र – 981133…..

शैक्षिक योग्यता
CBSE Class 9 Hindi A पत्र लेखन - 13
अनुभव-छह महीने से प्राइवेट कंपनी में क्लर्क के रूप में कार्यरत। आशा है मेरी योग्यताओं पर विचार कर आप सेवा का अवसर प्रदान करेंगे।

सधन्यवाद
आवेदक – मयंक कुमार
हस्ताक्षर – ……
दिनांक – 18 अक्टूबर, 20XX

VI. अन्य पत्र

व्यावसायिक पत्र

30. किसी प्रकाशक को पत्र लिखकर सूचित कीजिए कि उनके द्वारा भेजी गई पुस्तकों का पार्सल अब तक नहीं मिला है।
उत्तरः

सेवा में
फ्रैंक एजुकेशनल प्रा. लि.
सेक्टर……
गौतमबुद्ध नगर (उ० प्र०)
विषय-पार्सल प्राप्त न होने के संबंध में।

महोदय

पिछले सप्ताह मैंने पचास पुस्तकों की सूची और बीस हज़ार रुपये का चेक आपके प्रकाशन के नाम पर भेजा था। इसके साथ भेजे गए पत्र में ये पुस्तकें शीघ्र ही डाक द्वारा भेजने का अनुरोध किया था। दुर्भाग्य से दस दिन बीत जाने पर भी वह पार्सल मुझे अब तक नहीं मिल पाया है।

आपसे अनुरोध है कि यदि आपको पार्सल अब तक न भेजा हो तो कृपया अविलंब भेजने का कष्ट करें।

धन्यवाद
भवदीय
चंद्रगुप्त पुस्तक भंडार
108, गांधी मैदान रोड
पटना (बिहार)
15 मार्च, 20XX

आमंत्रण पत्र –

31. आपके विद्यालय ने 14 सितंबर को हिंदी दिवस मनाने का फैसला किया है। इस कार्यक्रम में शिक्षा मंत्री दिल्ली सरकार
को आमंत्रित करने के लिए पत्र लिखिए।
उत्तरः

सेवा में
माननीय शिक्षा मंत्री
दिल्ली सरकार, दिल्ली।

माननीय महोदय

आपको यह सूचित करते हुए हमें अत्यंत हर्ष की अनुभूति हो रही है कि गतवर्ष की भांति इस वर्ष भी हमारे विद्यालय में हिंदी दिवस मनाने का फैसला लिया गया है। इसमें हिंदी से जुड़ा एक कवि सम्मेलन तथ अन्य सांस्कृतिक कार्यक्रम मनाएँ जाएँगे। विद्यालय प्रबंधन चाहता है कि इस अवसर पर मुख्य अतिथि के रूप में पधारकर कार्यक्रम की शोभा बढ़ाएँ।

कृपया अपनी स्वीकृति प्रदान कर हमें कृतार्थ करें।

सधन्यवाद।
भवदीय अनुराग कुमार
सचिव
विद्यालय प्रबंधन समिति
05 सितंबर, 20XX

अनौपचारिक पत्र –

अनौपचारिक पत्र का प्रारूप
CBSE Class 9 Hindi A पत्र लेखन - 14

अनौपचारिक पत्रों के उदाहरण

बड़ों को पत्र

1. आपको नए सत्र के आरंभ में पुस्तकें, ड्रेस तथा कापियाँ खरीदने के अलावा फ़ीस भी जमा करवाना है। इसका उल्लेख करते हुए रुपये मँगवाने के लिए अपने पिता को पत्र लिखिए।
उत्तरः

छात्रावास दयाल
दयालबाग, आगरा (उ०प्र०)
28 मार्च 20XX

पूज्य पिता जी
सादर चरण स्पर्श।

मैं यहाँ सकुशल रहकर आशा करता हूँ कि आप सभी लोग आनंद से होंगे।

पिता जी, कल घोषित हुए परिणाम से पता चला कि मैं कक्षा में प्रथम आया हूँ, यह जानकर आपको काफ़ी खुशी होगी। नए सत्र, की पढ़ाई 02 अप्रैल से शुरू हो जाएगी। इसके लिए मुझे नई किताब-कापियाँ और ड्रेस खरीदनी है। इसके अलावा छात्रावास की फ़ीस भी जमा करवानी है। इसके लिए मुझे पाँच हज़ार रुपये की आवश्कता है। किताब-कापियाँ खरीदकर मैं नए सत्र की पढ़ाई शुरू कर दूंगा। आप रुपये मेरे खाते में जमा करवा दीजिएगा ताकि मैं समय से फ़ीस जमा कर सकूँ।

पूज्य माता जी को प्रणाम और चाँदनी को स्नेह। शेष सब कुशल है। पत्रोत्तर की प्रतीक्षा में,

आपका प्रिय पुत्र
पुलकित

2. आप अपने विद्यालय के छात्र-छात्राओं के साथ बाढ़ पीड़ितों की मदद के लिए जाना चाहते हैं। इसके लिए अनुमति माँगते हुए अपने पिता जी को पत्र लिखिए।
उत्तरः

परीक्षा भवन
नई दिल्ली
25 जुलाई 20XX
पूज्य पिता जी
सादर चरण स्पर्श।

मैं यहाँ सकुशल रहकर आशा करता हूँ कि आप भी सकुशल होंगे। मैं आप सब की कुशलता हेतु प्रार्थना करता हूँ। पिता जी, आपको तो पता ही होगा कि आजकल फैज़ाबाद और उसके आसपास के क्षेत्र में बाढ़ आई हुई है। इससे वहाँ रहने वालों का जीवन संकट में पड़ गया है। इन बाढ़ पीड़ितों की मदद से हमारे विद्यालय से एक दल राहत सामग्री, दवाइयाँ और कपड़े लेकर जा रहा है। इसमें हमारी कक्षा के छात्र-छात्राएँ भी जा रहे हैं। मनुष्यता की सेवा के इस पावन काम में मैं भी हाथ बँटाना चाहता हूँ। इसके लिए आपकी अनुमति की आवश्यकता है।

पूज्या माता जी को चरण स्पर्श और शैली को प्यार। शेष सब ठीक है। पत्रोत्तर की प्रतीक्षा में,

आपका प्रिय पुत्र
चंदन कुमार

3. आप इस ग्रीष्मावकाश में विद्यालय के तरणताल में तैराकी सीखना चाहते हैं। इसकी फ़ीस के लिए रुपये तथा अनुमति माँगते हुए पत्र लिखिए।
उत्तरः

विवेकानंद छात्रावास
विवेकानंद पब्लिक स्कूल
जींद, हरियाणा।
05 मई, 20XX

पूज्य पिता जी
सादर चरणस्पर्श।

मैं यहाँ सकुशल रहकर आशा करता हूँ कि आप भी सकुशल होंगे।

पिता जी आपको यह जानकर अत्यंत प्रसन्नता होगी कि मैंने जिला स्तर पर नौवीं कक्षा में प्रथम स्थान प्राप्त किया है। यह
आपके आशीर्वाद का परिणाम है। अब मैं इस अवकाश में पढ़ाई करते हुए तैराकी सीखना चाहता हूँ। तैराकी सीखने की व्यवस्था विद्यालय के तरणताल में ही है। इस कार्य के लिए यहाँ कुशल प्रशिक्षण की व्यवस्था है जिसकी देखरेख में विद्यालय के कई छात्र तैराकी सीखते हैं। इसके लिए एक हज़ार रुपये तथा आपकी अनुमति चाहिए। आशा है आप इस बारे में निराश नहीं करेंगे।

पूज्य माता जी एवं चाची जी को प्रणाम तथा सोनू को प्यार। शेष कुशल है। पत्रोत्तर की प्रतीक्षा में,

आपका प्रिय पुत्र
अर्पित कुमार

4. छात्रावास में मिलने वाले भोजन और वहाँ की अन्य सुविधाओं के बारे में पत्र लिखकर अपनी माता को बताइए कि वे चिंताग्रस्त न हों।
उत्तरः

चंद्रगुप्त मौर्य छात्रावास
पटना, बिहार।
10 अप्रैल, 20XX
पूज्य माता जी
सादर चरण स्पर्श।

आपका भेजा पत्र कल शाम को मिला पत्र पढ़कर जाना कि जब से मैं छात्रावास में आया हूँ तब से आप काफ़ी चिंतित रहती हैं। माँ, सच तो यह है कि शुरू-शुरू में एक-दो दिन मेरा मन भी बड़ा उदास रहा, परंतु जल्दी ही एक-दो सहपाठी मेरे मित्र बन गए। इसके अलावा यहाँ मिलने वाले भोजन की गुणवत्ता बहुत ही अच्छी है। यहाँ स्वाद का ध्यान कम पौष्टिकता को अधिक महत्त्व दिया जा रहा है। इसके अलावा यहाँ का विशाल खेल का मैदान, पढ़ाई की उत्तम व्यवस्था तथा हम सभी को अपना समझने वाले गुरुजन हैं। इससे मेरा मन छात्रावास में लग गया है। अब आप बिल्कुल भी चिंता न करना। शेष सब ठीक
है।

पूज्य पिता जी को प्रणाम तथा संचिता को स्नेह। पत्रोत्तर की प्रतीक्षा में,

आपका प्रिय पुत्र
सुचित सैनी

5. अपनी माता जी को पत्र लिखकर बताइए कि उनके बताए नियमानुसार पढ़ाई करने से ही आप इतने अच्छे अंक ला सकते हैं।
उत्तरः

परीक्षा भवन
नई दिल्ली
08 मई 20XX
पूज्या माता जी
सादर चरण स्पर्श।

मैं यहाँ सकुशल रहकर आशा करता हूँ कि आप सब भी सकुशल होंगे। माँ आपको यह जानकर काफ़ी प्रसन्नता होगी कि मैंने अपनी कक्षा में दूसरा स्थान प्राप्त किया है, जबकि एस.ए-I में फेल होते-होते बचा था। यह सब आपके कुशल निर्देशन एवं सीख का परिणाम है। मैंने आपकी बातों पर ध्यान देकर सवेरे उठना शुरू किया और टाइम-टेबल बनाकर पढ़ाई शुरू कर दी। मैंने हर विषय की पढ़ाई के लिए बराबर समय दिया और कठिन प्रश्नों के उत्तर के लिए सहपाठियों एवं अध्यापकों की सहायता ली। परीक्षा के समय पाठ्यक्रम दोहराया। हाँ प्रश्नों के उत्तर लिखते समय आपकी बातों का बराबर ध्यान रखा। इसका परिणाम अब सामने है। आशा है कि आप भविष्य में भी ऐसी ही सहायता करती रहेंगी।

पूज्य पिता जी को चरण स्पर्श और सुरभि को स्नेह कहना। पत्रोत्तर की प्रतीक्षा में,

आपका प्रिय पुत्र
कृतज्ञ वर्मा

6. आपके चाचा-चाची चार धाम की यात्रा पर जा रहे हैं। उनकी सफल एवं मंगलमय यात्रा के लिए शुभकामनाएँ देते हुए पत्र लिखिए।
उत्तरः

परीक्षा भवन
नई दिल्ली
10 नवंबर, 20XX
पूज्य चाचा एवं चाची जी
सादर प्रणाम।

मैं यहाँ सकुशल रहकर आशा करता हूँ कि आप सब भी सकुशल होंगे। आपकी कुशलता हेतु ईश्वर से प्रार्थना करता हूँ। चाचा जी, यह जानकर अतीव हर्ष हुआ कि ईश्वर की असीम कृपा से आप चार धामं की यात्रा पर जा रहे हैं। वास्तव में यह पूरे परिवार के लिए हर्ष का विषय है कि आपको इस धार्मिक यात्रा के बहाने भ्रमण का सुअवसर मिल जाएगा। इससे एक पंथ दो काज वाली कहावत चरितार्थ हो जाएगी। इस यात्रा के लिए मैं बार-बार शुभकामनाएँ देता हूँ। ईश्वर करे कि आपकी यह यात्रा सफल एवं मंगलमय हो।

पत्रोत्तर में अपनी यात्रा के बारे में अवश्य लिखिएगा। पत्रोत्तर की प्रतीक्षा में,

आपका भतीजा
पुष्कर सोनी

7. अपने बड़े भाई की स्वास्थ्य संबंधी सलाह मानने के कारण आपका स्वास्थ्य सुधरता जा रहा है। इसके लिए उन्हें धन्यवाद देते हुए पत्र लिखिए।
उत्तरः

आनंद निकेतन छात्रावास
यपुर, राजस्थान।
10 दिसंबर, 20XX
आदरणीय भाई साहब
सादर प्रणाम।

मैं यहाँ सकुशल रहकर आशा करता हूँ कि आप सब भी सकुशल होंगे। इसके लिए मैं ईश्वर से यही प्रार्थना करता हूँ। भाई साहब, इस बार छात्रावास में आते ही मैंने आपके बताने के अनुसार प्रातः पाँच बजे उठना शुरू कर दिया हूँ। मैं आधे घंटे तक व्यायाम करता हूँ, दौड़ता हूँ। इसे मैंने प्रतिदिन का नियम बना लिया है। शाम को मैं एक-डेढ़ घंटे नियमित रूप से खेलों में भाग ले रहा हूँ। इससे मेरा आलस्य कम हो रहा है और पढ़ाई में मेरा मन लगने लगा है। अब मैं हर काम के लिए उत्साहित रहता हूँ। यह सबकुछ आपके सुझाव एवं निर्देशन के कारण संभव हो पाया है। इसके लिए आपको बार-बार धन्यवाद देता हूँ। भविष्य में भी आप ऐसा ही सहयोग बनाए रखेंगे ऐसी आशा है।

पूज्या माता एवं पिता जी को चरण स्पर्श तथा विभा को स्नेह। पत्रोत्तर की प्रतीक्षा में,

आपका अनुज
विकास विष्ट

समान उम्रवालों को पत्र

8. आपके मित्र ने दसवीं की बोर्ड परीक्षा में जिले में प्रथम स्थान प्राप्त किया है। इस खुशी में शामिल होते हुए उसे बधाई पत्र लिखिए।
उत्तरः

ए. 37/5, रामा मार्ग
आदर्श नगर, दिल्ली।
30 मई 20XX
प्रिय मित्र विनय
सप्रेम नमस्ते।

मैं यहाँ सकुशल रहकर तुम्हारी कुशलता हेतु ईश्वर से प्रार्थना करता हूँ।

मित्र! कल समाचार-पत्र में तुम्हारा नाम और फोटो देखकर सहसा विश्वास ही नहीं हुआ कि सचमुच में तुम्हीं हो। मित्र, जनपद भर में बोर्ड परीक्षा में प्रथम स्थान प्राप्त करना बड़ी उपलब्धि है। इस शानदार उपलब्धि पर मैं तुम्हें बार-बार बधाई देता हूँ। इस सफलता से तुमने अपने साथ-साथ परिवार, विद्यालय, गुरुजन और मित्रों को भी गौरवान्वित किया है। मैं ईश्वर से प्रार्थना करता हूँ कि भविष्य में भी वह तुम्हारा साथ दे और तुम इससे भी बढ़कर सफलता अर्जित करो। इस शानदार सफलता के लिए एक बार पुनः मेरी बधाई स्वीकार करो। अपने माता-पिता को मेरा प्रणाम कहना। पत्रोत्तर की प्रतीक्षा में,

तुम्हारा अभिन्न मित्र
मोहित

9. आपके मित्र का चयन अपने राज्य की अंडर नाइनटीन क्रिकेट टीम में हो गया है। उसे और परिश्रम करने की सलाह देते हुए शुभकामना-पत्र लिखिए।
उत्तरः

परीक्षा भवन
नई दिल्ली
27 नवंबर, 20XX
प्रिय मित्र सौरभ
सप्रेम नमस्ते!

मैं यहाँ सकुशल रहकर आशा करता हूँ कि तुम भी स्वस्थ एवं प्रसन्न होंगे।

मित्र! यह जानकर अतीव हर्ष हुआ कि तुम्हारा चयन अपने राज्य उत्तर प्रदेश की अंडर नाइनटीन क्रिकेट टीम में हो गया है। यह हम सबके लिए अत्यंत खुशी की बात है। मित्र! यूँ तो चयन होना ही अपने आप में एक विशिष्ट उपलब्धि है, पर अब तुम्हें और अधिक परिश्रम करने की आवश्यकता है। अब अपने खेल पर खूब ध्यान दो। इस परिश्रम का फल तुम्हें अवश्य मिलेगा। इस चयन से हम सबकी बहुत-सी आशाएँ जुड़ी हैं जो आगे साकार होंगी। एक अच्छा क्रिकेट खिलाड़ी बनने के लिए मेरी हार्दिक शुभकामनाएँ स्वीकार करो।

अपने माता-पिता को मेरा प्रणाम कहना। शेष मिलने पर,
तुम्हारा अभिन्न मित्र
पुरुषार्थ

10. अपने मित्र की दादी जी के आकस्मिक निधन पर शोक संवेदना-पत्र लिखिए। आप 37 B/4 महात्मा गांधी मार्ग साकेत दिल्ली निवासी विक्रम हो।
उत्तरः

37 B/4
महात्मा गांधी मार्ग
साकेत, दिल्ली
9 सितंबर 20XX
प्रिय मित्र नितिन
सप्रेम नमस्ते।

कल शाम को तुम्हारा भेजा पत्र मिला। पढ़कर बड़ा दुख हुआ कि तुम्हारी ममतामयी दादी जी का आकस्मिक निधन हो गया। इससे तुम्हें जो दुख हुआ होगा, उसे शब्दों में बाँधना कठिन है।

मित्र! तुम्हारी दादी अभी तो पूरी तरह स्वस्थ थी, पर उस प्रभु की लीला को कौन जान सकता है। जीवन-मरण उसी के हाथ में है। दादी जी का यह चिरवियोग बड़ा ही दुखदायी है। दुख की इस घड़ी में मैं तुम्हारे साथ हूँ। ईश्वर तुम्हें यह दुख सहन करने की शक्ति दे तथा दिवंगत दादी जी की आत्मा को शांति दे। दुख की इस घड़ी में तुम अपना साहस एवं धैर्य अवश्य बनाए रखना। शेष मिलने पर!

तुम्हारा अभिन्न मित्र
सार्थक

11. तमिलनाडु में रहने वाले अपने मित्र को दशहरा अवकाश के दौरान अपने शहर दिल्ली आने के लिए आमंत्रित करते हुए पत्र लिखिए।
उत्तरः

परीक्षा भवन
नई दिल्ली 1
5 सितंबर 20XX
प्रिय मित्र नागराजन
सप्रेम नमस्ते!

मैं यहाँ सकुशल रहकर आशा करता हूँ तुम भी सकुशल होंगे और तुम्हारी कुशलता हेतु कामना करता हूँ।

मित्र! तुम्हें यह जानकर हर्ष होगा कि हमारी एस.ए. I की परीक्षाएँ समाप्त होते ही दशहरा अवकाश शुरू हो जाएगा। यह भी अजीब संयोग है कि इधर परीक्षा खत्म और उधर छुट्टियाँ शुरू। मेरी इच्छा है कि इस बार की छुट्टियाँ तुम हमारे साथ देश की राजधानी दिल्ली में मनाओ और दशहरे की रामलीला का आनंद उठाओ। तुम्हारे साथ झाँकियाँ और रामलीला देखने का आनंद कई गुना बढ़ जाएगा। इस बार ये छुट्टियाँ 30 सितंबर से हो रही हैं। इनमें हम मेट्रो की वातानुकूलित यात्रा का आनंद भी उठाएँगे।

अपने माता-पिता को मेरा प्रणाम कहना। शेष अगले पत्र में,

तुम्हारा अभिन्न मित्र
केशव

12. आपके क्षेत्र में डेंगू और चिकनगुनिया फैल गया है। इससे यहाँ बीमारी की संभावना बढ़ गई है। इसका उल्लेख करते हए अपने उस मित्र को पत्र लिखिए जो चेन्नई से आपके पास आने वाला है।
उत्तरः

एफ 32/2B
नंदग्राम, गाज़ियाबाद
03 अक्टूबर, 20XX
प्रिय मित्र राजप्पा
सप्रेम नमस्ते।

मैं यहाँ सकुशल रहकर आशा करता हूँ कि तुम भी सकुशल होंगे और मैं ईश्वर से यही कामना भी करता हूँ।

मित्र! बरसात के मौसम में जगह-जगह जलभराव हो जाता है जिससे मच्छर-मक्खियाँ पनपते हैं। इस समय हमारे क्षेत्र की स्थिति भी कुछ ऐसी ही है। आजकल यहाँ डेंगू और चिकनगुनिया का प्रकोप जोरों पर है। ऐसे में अस्पताल में भरती होने वालों की संख्या में निरंतर बृद्धि होती जा रही है। सरकार द्वारा उठाए गए कदम इसे रोकने में अपर्याप्त सिद्ध हो रहे हैं। ऐसी स्थिति में यहाँ आना स्वास्थ्य पर प्रतिकूल प्रभाव डाल सकता है। स्थिति सामान्य होने के बाद मैं स्वयं सूचित कर दूंगा।

अंकल और आँटी को मेरा प्रणाम कहना। शेष अगले पत्र में,

तुम्हारा अभिन्न मित्र
अनुरूप आर्य

13. आप अपने मित्र के पास दिल्ली आए थे। उसके साथ कुछ समय बिताकर अपने घर लौट चुके हैं। इसके लिए धन्यवाद देते हुए उसे पत्र लिखिए।
उत्तरः

विद्यासागर छात्रावास
आर. के पुरम् चेन्नई।
10 जनवरी, 20XX
प्रिय मित्र आशुतोष
सप्रेम नमस्ते!

मैं यहाँ सकुशल रहकर आशा करता हूँ कि तुम भी स्वस्थ एवं प्रसन्न होंगे।

मित्र! यह पत्र मैं तुम्हें धन्यवाद देने के लिए लिख रहा हूँ। इस दिसंबर का अंतिम सप्ताह मैंने तुम्हारे साथ दिल्ली में बिताया। वह मेरे लिए अविस्मरणीय रहेगा। तुमने वहाँ के दार्शनिक स्थलों, प्रसिद्ध मंदिरों, सुंदर पार्कों के जो दर्शन कराए उनकी छवि अब भी ताज़ी हैं और ताज़ी रहेंगी। इतना ही नहीं तुमने दिल्ली के इतिहास तथा वहाँ के महत्त्वपूर्ण स्थलों की जो उपयोगी जानकारी दी है, उससे मेरा ज्ञानवर्धन ही नहीं अपितु मनोरंजन भी हुआ। इसके लिए मैं बार-बार धन्यवाद देना चाहता हूँ। गरमी की छुट्टियों मैं तुम चेन्नई आना। मैं तुम्हें वहाँ की सैर कराऊँगा।
अपने माता-पिता को मेरा प्रणाम कहना। शेष अगले पत्र में,

तुम्हारा अभिन्न मित्र
सुंदरम्
छोटों को पत्र

13. आपका छोटा भाई वाद-विवाद प्रतियोगिता में प्रथम आया है। उसे बधाई देते हुए पत्र लिखिए।
उत्तरः

परीक्षा भवन
नई दिल्ली
20 सितंबर 20XX
प्रिय सोनू
शुभाशीर्वाद।

हम सभी यहाँ सकुशल रहकर आशा करते हैं कि तुम भी छात्रावास में सकुशल होगे।

यह जानकर हम सभी को अत्यंत हर्ष हुआ कि तुमने वाद-विवाद प्रतियोगिता में प्रथम स्थान प्राप्त किया है। यह हम सब के लिए गर्व की बात है। हम सबको आश्चर्य हो रहा है कि जी.एस.टी. जैसे कठिन विषय पर भी तुमने अपनी बात जिस वाक्पटुता से कही उसने निर्णायक मंडल के सदस्य प्रभावित हुए बिना न रह सके। इतना ही नहीं तुमने तर्कों और आँकड़ों के मेल से अपनी बातों को तथ्यपूर्ण बना लिया। तुम्हारा यह प्रयास अत्यंत सराहनीय है। प्रतियोगिता में प्रथम पुरस्कार पाने के लिए मैं तुम्हें बार-बार बधाई देता हूँ। आशा है कि तुम भविष्य में भी इसी तरह का प्रयास जारी रखोगे।

माता एवं पिता जी की ओर से तुम्हें स्नेह। पत्रोत्तर की प्रतीक्षा में,

तुम्हारा अग्रज
सौम्य शर्मा

14. छोटे भाई को सफ़ाई का महत्त्व बताते हुए अपने आसपास साफ़-सुथरा रखने की प्रेरणा देते हुए पत्र लिखिए।
उत्तरः

राधा रमण छात्रावास
मथुरा उत्तर प्रदेश
15 सितंबर, 20XX
प्रिय अनुज रोशन
शुभाशीष।

कल सायं तुम्हारा पत्र मिला। यह जानकर दुख हुआ कि तुम तीन दिन तक ज्वर ग्रस्त रहने के बाद अब स्वस्थ हुए हो। अनुज, निश्चित ही तुमने छात्रावास में साफ़-सफ़ाई पर ध्यान नहीं दिया होगा और बीमार पड़ गए। यह बात निश्चित रूप से समझ लो कि सफ़ाई और स्वास्थ्य का अत्यंत घनिष्ठ संबंध है। सफ़ाई के प्रति लापरवाही होते ही बीमारियाँ आ घेरती हैं। इस कारण अपने तन की ही नहीं, आसपास की सफ़ाई करना भी आवश्यक हो जाता है। महात्मा गांधी ने तो यहाँ तक कहा है कि सफ़ाई ईश्वर का दूसरा नाम हैं। साफ़-सफ़ाई रखने से हम खुद ही नहीं औरों को भी बीमारियों से बचाते हैं। आशा है कि तुम अब साफ़-सफ़ाई पर ध्यान दोगे। शेष कुशल है।

माता-पिता की ओर से तुम्हें ढेर सारा प्यार। शेष अगले पत्र में,

तुम्हारा अग्रज
संतोष कुमार

15. अपने छोटे भाई को पत्र लिखिए, जिसमें पटाखों के नुकसान बताते हुए पटाखों के बिना दीपावली मनाने की सीख दी गई हो।
उत्तरः

परीक्षा भवन
नई दिल्ली
15 अक्टूबर 20XX
प्रिय अनुज मोहन
शुभाशीर्वाद!

हम सभी यहाँ सकुशल रहकर आशा करते हैं कि तुम भी स्वस्थ एवं प्रसन्न होंगे।

मोहन! याद है तुम्हें पिछली दीपावली पर ज़िद करके बहुत सारे पटाखे खरीद लिए थे। शाम को पटाखों को जलाते हुए तुम्हारा हाथ झुलस गया था। पटाखों से इसके अलावा और भी हानि होती है। इनसे निकला धुआँ वायु को ज़हरीला बनाता है जिससे स्वाँस की अनेक बीमारियाँ उत्पन्न होती है। इनसे उत्पन्न शोर से छोटे बच्चों, बूढों और मरीजों को विशेष परेशानी होती है। इसके अलावा कूड़ा चारों ओर बिखरा नज़र आता है। पटाखे जलाने से अनेक बच्चे जल जाते हैं और अस्पताल की शरण लेते हैं। मैं सलाह देता हूँ कि इस बार दीपावली पटाखों के बिना मनाओ और स्वस्थ रहने में सबकी मदद करो।

आशा ही नहीं विश्वास है कि तुम मेरी बात अवश्य मानोगे।

तुम्हारा अग्रज
मनोज मोहन

NCERT Solutions for Class 9 Hindi

The post CBSE Class 9 Hindi A पत्र लेखन appeared first on Learn CBSE.


CBSE Class 9 Hindi B संवाद लेखन

$
0
0

CBSE Class 9 Hindi B लेखन कौशल संवाद लेखन

संवाद शब्द ‘वाद’ मूल शब्द में ‘सम्’ उपसर्ग लगाने से बना है। संवाद का अर्थ है वार्तालाप या बातचीत। अर्थात् दो व्यक्तियों के बीच किसी विषय पर हुई बातचीत को संवाद कहते हैं। उनके मध्य हुई बातचीत को लिपिबद्ध करना ही संवाद-लेखन कहलाता है। अच्छा संवाद लेखन करते समय निम्नलिखित बातों का ध्यान रखना चाहिए-

  • संवाद सुनने वाले की उम्र, रुचि को ध्यान में रखना चाहिए।
  • वाक्यों का स्पष्ट, शुद्ध उच्चारण करना चाहिए। रोचकता और सहजता होनी चाहिए।
  • विषय पर पकड़ एवं संबद्धता होनी चाहिए।
  • मनोभावों को स्पष्ट करने के लिए विराम चिह्नों का प्रयोग आवश्यक है।

आइए संवाद लेखन के कुछ उदाहरण देखते हैं

प्रश्नः 1.
आजकल महँगाई बढ़ती ही जा रही है। इससे परेशान दो महिलाओं की बातचीत को संवाद के रूप में लिखिए।
उत्तर:
रचना – अलका बहन नमस्ते! कैसी हो?
अलका – नमस्ते रचना, मैं ठीक हूँ पर महँगाई ने दुखी कर दिया है।
रचना – ठीक कहती हो बहन, अब तो हर वस्तु के दाम आसमान छूने लगे हैं।
अलका – मेरे घर में तो नौकरी की बँधी-बधाई तनख्वाह आती है। इससे सारा बजट खराब हो गया है।
रचना – नौकरी क्या रोज़गार क्या, सभी परेशान हैं।
अलका – हद हो गई है कोई भी दाल एक सौ बीस रुपये किलो से नीचे नहीं है।
रचना – अब तो दाल-रोटी भी खाने को नहीं मिलने वाली।
अलका – बहन कल अस्सी रुपये किलो तोरी और साठ रुपये किलो टमाटर खरीदकर लाई। आटा, चीनी, दाल, चावल मसाले दूध सभी में आग लगी है।
रचना – फल ही कौन से सस्ते हैं। सौ रुपये प्रति किलो से कम कोई भी फल नहीं हैं। अब तो लगता है कि डाक टर जब लिखेगा तभी फल खाने को मिलेगा।
अलका – सरकार भी कुछ नहीं करती महँगाई कम करने के लिए। वैसे जनता की भलाई के दावे करती है। जमाखोरों पर कार्यवाही भी नहीं करती है।
रचना – नेतागण व्यापारियों से चुनाव में मोटा चंदा लेते हैं फिर सरकार बनाने पर कार्यवाही कैसे करे।
अलका – गरीबों को तो ऐसे ही पिसना होगा। इनके बारे में कोई नहीं सोचता।

प्रश्नः 2.
यमुना की दुर्दशा पर दो मित्रों की बातचीत को संवाद के रूप में लिखिए।
उत्तर:
अजय – नमस्कार भाई साहब, शायद आप दिल्ली के बाहर से आए हैं।
प्रताप – नमस्कार भाई, ठीक पहचाना तुमने, मैं हरियाणा से आया हूँ।
अजय – मैं भी अलवर से आया हूँ। तुम यहाँ कैसे?
प्रताप – दिल्ली आया था। सोचा सवेरे-सवेरे यमुना में स्नान कर लेता हूँ पर
अजय – कल मेरा यहाँ साक्षात्कार था और आज कुछ और काम था। मैं भी यहाँ स्नान के लिए आया था।
प्रताप – इतनी गंदी नदी में कैसे नहाया जाए?
अजय – मैंने भी यमुना का बड़ा नाम सुना था, पर यहाँ ती उसका उल्टा निकला।
प्रताप – इसका पानी तो काला पड़ गया है।
अजय – फैक्ट्रियों और घरों का पानी लाने वाले कई नाले इसमें मिल जाते हैं न।
प्रताप – देखो, वे सज्जन फूल मालाएँ और राख फेंककर पुण्य कमा रहे हैं।
अजय – इनके जैसे लोग ही तो नदियों को गंदा करते हैं।
प्रताप – सरकार को नदियों की सफ़ाई पर ध्यान देना चाहिए।
अजय – केवल सरकार को दोष देने से कुछ नहीं होने वाला। हमें खुद सुधरना होगा।
प्रताप – ठीक कहते हो। यदि सभी ऐसा सोचें तब न।
अजय – यहाँ की शीतल हवा से मन प्रसन्न हो गया। अब चलते हैं।
प्रताप – ठीक कहते हो। अब हमें चलना चाहिए।

प्रश्नः 3.
बढ़ती गरमी और कम होती वर्षा के बारे में दो मित्रों की बातचीत का संवाद-लेखन कीजिए।
उत्तर:
रवि – रमन, कैसे हो?
रमन – मत पूछ यार गरमी से बुरा हाल है।
रवि – गरमी इसलिए बढ़ गई है क्योंकि वर्षा भी तो नहीं हो रही है।
रमन – 24 जुलाई भी बीतने को है पर बादलों का नामोनिशान भी नहीं है।
रवि – मेरे दादा जी कह रहे थे, पहले इतनी गरमी नहीं पड़ती थी और तब वर्षा भी खूब हुआ करती थी।
रमन – ठीक कह रहे थे तुम्हारे दादा जी। तब धरती पर आबादी कम थी परंतु पेड़-पौधों की कमी न थी।
रवि – वर्षा और पेड़ पौधों का क्या संबंध?
रमन – पेड़-पौधे वर्षा लाने में बहुत सहायक हैं। जहाँ अधिक वन हैं वहाँ वर्षा भी खूब होती है। इससे गरमी अपने आप कम हो जाती है।
रवि – फिर तो हमें भी अपने आसपास खूब सारे पेड़-पौधे लगाने चाहिए।
रमन – और हरे-भरे पेड़ों को कटने से बचाना भी चाहिए।
रवि – इस गरमी के बाद वर्षा ऋतु में खूब पौधे लगाएँगे।
रमन – यही ठीक रहेगा।

प्रश्नः 4.
कक्षा-IX में प्रवेश लेने आए छात्र और प्रधानाचार्य के मध्य बातचीत का संवाद लेखन कीजिए।
उत्तर:
छात्र – नमस्ते सर। क्या मैं अंदर आ सकता हूँ।
प्रधानाचार्य – नमस्ते। आ जाओ। क्या बात है?
छात्र – जी, मुझे नौवीं कक्षा में प्रवेश चाहिए।
प्रधानाचार्य – आठवीं कक्षा तुमने कौन-से विद्यालय से उत्तीर्ण की है?
छात्र – जी, राजकीय सर्वोदय बाल विद्यालय ………………. से।
प्रधानाचार्य – क्या तुम अपना अंक-पत्र लाए हो?
छात्र – जी हाँ, यह रहा मेरा अंक-पत्र।
प्रधानाचार्य – तुम्हारे ग्रेड तो अच्छे हैं, पर तुम यहाँ प्रवेश क्यों लेना चाहते हो?
छात्र – मेरे पिता जी का स्थानांतरण अभी यहीं हुआ है। यह विद्यालय मेरे आवास से सबसे निकट है।
प्रधानाचार्य – कोई और कारण?
छात्र – जी मुझे अंग्रेजी माध्यम से पढ़ाई करनी है ताकि मैं ग्यारवीं में विज्ञान वर्ग में प्रवेश ले सकूँ।
प्रधानाचार्य – यह फार्म भरो और मिस्टर वर्मा से मिलो। वे तुम्हारा टेस्ट लेंगे।
छात्र – जी धन्यवाद।

प्रश्नः 5.
परीक्षा भवन में जाने से आधा घंटा पहले दो सहपाठियों में हुई बातचीत का संवाद लेखन कीजिए।
उत्तर:
अमर – अरे विनय! सारी तैयारी कर लिया?
विनय – हाँ अमर मैंने तो सारा पाठ्यक्रम दोहरा लिया है।
अमर – मैंने तो रात में देर तक जगकर पढ़ाई की परंतु पाठ्यक्रम पूरा न हो सका।
विनय – तूने पाइथागोरस प्रमेय के सवाल किए हैं?
अमर – नहीं विनय, मेरा तो मन घबरा रहा है। कहीं प्रश्न पत्र पूरा हल न कर पाया तो।
विनय – इस तरह दिल छोटा नहीं करते। चल जल्दी से देख, यह रहा सूत्र और इस पर आधारित सवाल।
अमर – यार एक बार और समझा दे।
विनय – ठीक है। अच्छा कुछ और?
अमर – एक बार मुझे हीरोन के सूत्र के बारे में बता दे।
विनय – यह भी आसान है। यह रहा हीरोन का सूत्र।
अमर – इस पर आधारित कोई सवाल समझा दे न।
विनय – यह देख सवाल। ऐसे करते हैं।
अमर – धन्यवाद विनय। चल अब अंदर चलते हैं। घंटी बज रही है।
विनय – बेस्ट आफ लक।

प्रश्नः 6.
शोर के कारण पढ़ाई में उत्पन्न हो रही बाधा पर दो छात्रों के मध्य हुए संवाद का लेखन कीजिए।
उत्तर:
नमन – नमस्कार अजय! कैसे हो?
रमन – नमस्कार नमन ! मैं ठीक हूँ। परीक्षा की तैयारी कैसी चल रही है?
नमन – रमन तैयारी कर तो रहा हूँ, पर अच्छी तरह नहीं हो पा रही है।
रमन – क्या बात है, तबीयत तो ठीक है ना।
नमन – तबीयत तो एक दम ठीक है पर ……………….
रमन – पर क्या?
नमन – मेरी कॉलोनी में दो धार्मिक स्थल है जिससे वहाँ शोर होता रहता है।
रमन – क्या लोगों की ज़्यादा भीड़-भाड़ होती है वहाँ ?
नमन – लोगों की भीड़ तो कम पर वहाँ तेज़ आवाज़ में लाउडस्पीकर बजता रहता है।
रमन – इस बारे में सोसायटी के लोग मिलकर पुजारी से बात क्यों नहीं करते हैं।
नमन – कई बार बात की पर लगता है, दोनों पुजारियों में जैसे लाउडस्पीकर बजाने की प्रतियोगिता हो रही है।
रमन – उन्हें बताओ कि रात दस बजे के बाद लाउडस्पीकर बजाने पर प्रतिबंध है।
नमन – अब तो लगता है कि उनके विरुद्ध थाने में शिकायत करनी पड़ेगी क्योंकि इसमें हमें नींद नहीं आती है और हमारे काम प्रभावित हो रहे हैं।
रमन – अवश्य, क्योंकि इसका संबंध सभी के स्वास्थ्य से है।

प्रश्नः 7.
अध्यापिका और गृहकार्य न करके आने वाले छात्र के बीच हुई बातचीत का संवाद लेखन कीजिए।
उत्तर:
अध्यापिका – मोनू, अब तुम अपनी कॉपी निकालकर तैयार रहो।
मोनू,- जी मैम।
अध्यापिका – जल्दी ढूढों, तुम्हारा नंबर आ गया है।
मोनू – मैम! लगता है कॉपी तो घर रह गई।
अध्यापिका – तुमने काम किया ही न होगा।
मोनू – नहीं मैम, काम तो किया था।
अध्यापिका – पिछले सप्ताह भी तो तुमने यही बहाना किया था।
मोनू – ध्यान आ गया मैम, कल मैं घरवालों के साथ एक विवाह-पार्टी में चला गया और रात में देर से लौटा था।
अध्यापिका – तो काम पूरा करके पार्टी में जाना था।
मोनू – सोचा था, मैम कि आकर कर लूँगा पर समय ही नहीं मिला।
अध्यापिका – तुम झूठ बोलना भी सीखते जा रहे हो। यह अच्छी बात नहीं। कल अपने पिता या माँ को साथ लेकर आना।
मोनू – मैम एक आखिरी मौका दे दीजिए, प्लीज!

प्रश्नः 8.
वनों की अंधाधुंध कटाई पर चिंता प्रकट करते हुए दो मित्रों के मध्य हुए संवाद (बातचीत) को लिखिए।
उत्तर:
पुनीत – नमस्ते सुमित! कहाँ थे छुट्टियों में?
सुमित – नमस्ते पुनीत! इन छुट्टियों में मैं अपने नाना जी से मिलने चला गया था।
पुनीत – तुम्हारे नाना जी गाँव में रहते हैं क्या?
सुमित – हाँ पुनीत! वहाँ का हरा-भरा वातावरण छोड़कर आने को मन ही नहीं कर रहा था।
पुनीत – अच्छा रहा तुम हरे-भरे वातावरण का आनंद उठा आए।
सुमित – पुनीत, तुम दिल्ली में ही थे या कहीं गए थे।
पुनीत – मैं भी अपने चाचा के पास आगरा गया था।
सुमित – वहाँ ताजमहल देखकर बड़ा आनंद आया होगा न?
पुनीत – ताजमहल देखने के आनंद से अधिक दुख वहाँ कटते पेड़ों को देखकर हुआ। जहाँ कभी हरे-भरे पेड़ हुआ करते थे अब घर बनते जा रहे हैं।
सुमित – यहाँ दिल्ली से तो जैसे हरियाली गायब ही हो गई है।
पुनीत – कुछ लोग वनों को काटकर अब वहाँ रेत, सीमेंट, कंकरीट और लोहे के मकानों के जंगल खड़े करते जा रहे
सुमित – जलवायु परिवर्तन, बढ़ती गरमी, बाढ़ आना ये सब वनों के कटने के दुष्परिणाम हैं।
पुनीत – हमें लोगों को इसके प्रति जागरूक करना होगा ताकि वनों की कटाई रुक सके।
सुमित – तुम्हारे इस अभियान में मैं और मेरे मित्र भी साथ देंगे।

प्रश्नः 9.
गरमी की ऋतु में पानी की कमी से उत्पन्न समस्या से परेशान दो महिलाओं की बातचीत का संवाद लेखन कीजिए।
उत्तर:
गीता – अरे सीमा! क्या बात है कुछ परेशान-सी दिख रही हो।
सीमा – क्या बताऊँ, गीता, न कल दिन में पानी और न रात में।
गीता – गरमी आते ही बिजली की तरह ही पानी का संकट शुरू हो जाता है।
सीमा – बिजली न आने पर जैसे-तैसे झेल भी लेते हैं परंतु पानी के बिना बड़ी परेशानी होती है।
गीता – आखिर परेशानी क्यों न हो नहाना, धोना, खाना बनाना आदि काम पानी से ही तो होते हैं।
सीमा – अब तो गरमी भी अधिक पड़ने लगी है! इससे नदियाँ तक सूख जाने लगी हैं। आखिर इन्हीं नदियों का पानी शुद्ध करके शहरों में घर-घर भेजा जाता है।
गीता – पिछले सप्ताह मैंने देखा था कि मजदूरों की बस्ती में कई नल खुले थे, दो-तीन की टोटियाँ टूटी थी, जिनसे पानी बहता जा रहा था।
सीमा. – पानी की यही बरबादी तो जल संकट को जन्म दे रही है। हमें पानी की बरबादी अविलंब बंद कर देना चाहिए।

प्रश्नः 10.
नोटबंदी से उत्पन्न समस्या से परेशान दो लोगों की बातचीत को संवाद रूप में लिखिए।
उत्तर:
मोहन – नमस्ते गोपी! लगता है काफ़ी पहले लाइन में लगने आ गए थे।
गोपी – कल दस बजे के बाद आया था। दिन भर लाइन में लगा रहा और शाम को खाली हाथ जाना पड़ा।
मोहन – इस नोटबंदी ने हम जैसों की कमर तोड़ दी है। जानते हो सब्जियाँ खरीदने वाली सौ रुपये की सब्जी खरीदकर पाँच सौ का पुराना नोट तुड़ाना चाहती है।
गोपी – मेरी तो कल दिहाड़ी मारी गई थी। लाइन में लगने से काम पर न जा सका था और आज तो भगवान ही मालिक है।
मोहन – अब तो फेरी लगाने पर रोटी भर के लिए भी पैसे कमाना मुश्किल हा गया है।
गोपी – ठेकेदार तो नोटबंदी का बहाना करके न पिछली मज़दूरी दे रहा है और न नया काम करवा रही है।
मोहन – पर सरकार कहती है कि इसका फायदा बाद में मिलेगा।
गोपी – ऐसे तो भुखमरी की स्थिति आ जाएगी।
मोहन – हमारे कई जाननेवाले मजदूर यहाँ से गाँव जा चुके हैं।
गोपी – सरकार ने हम गरीबों का तनिक भी ध्यान नहीं रखा। वह तो अच्छे दिनों का वाब दिखा रही है।
मोहन – उम्मीद रखो, कुछ दिन में शायद हालात ठीक हो जाएँ।
गोपी – इसी उम्मीद पर तो जिंदा हूँ।

प्रश्नः 11.
बस अड्डे पर टिकट विक्रेता और यात्री के मध्य संवाद का लेखन कीजिए।
उत्तर:
यात्री – नमस्ते, बाबू जी।
क्लर्क – नमस्कार। बताइए, क्या चाहिए आपको?
यात्री – जी, मुझे लखनऊ के लिए दो टिकट चाहिए।
क्लर्क – आप मुझे आठ सौ रुपए दीजिए।
यात्री – आप तो बहुत ज़्यादा किराया बता रहे हैं। इतना तो रेलगाड़ी का भी नहीं है।
क्लर्क – यह बस भी तो ए.सी. वाली है न।।
यात्री – मुझे बिना ए.सी. वाली बस का टिकट दे दीजिए।
क्लर्क – बिना ए.सी. वाली बस अभी एक घंटे पहले गई है। अब आज लखनऊ जाने वाली ऐसी कोई बस नहीं है।
यात्री – पर इसका किराया भी तो बहुत अधिक है।
क्लर्क – थोड़ी ही दूर पर रेलवे स्टेशन है। आप ट्रेन के बारे में पता करके चले जाइए।
यात्री – वहीं से तो लौटकर आया हूँ। लखनऊ जाने के लिए आज कोई रेलगाडी नहीं है।
क्लर्क – जल्दी बताओ, टिकट दूँ। तुम्हारे पीछे दूसरा यात्री आ गया है।
यात्री – आप दो टिकट दे दीजिए। ये लीजिए हज़ार का नोट।
क्लर्क – ये रहे आपके दो टिकट और शेष दो सौ रुपए।

प्रश्नः 12.
कुछ छोटे बच्चे अधिकांश समय टी.वी. देखते रहते हैं। इसका दुष्परिणाम उन्हें भुगतना पड़ता है। इसी विषय पर दो महिलाओं के बीच हुई बातचीत को संवाद के रूप में लिखिए।
उत्तर:
रचना – अरे सरिता! बेटो को लेकर बाज़ार गई थी क्या?
सरिता – नहीं रचना, मैं इसे लेकर डॉक्टर के पास गई थी।
रचना – डॉक्टर के पास! क्यों, क्या हुआ इसे? यह तो स्वस्थ दिखाई पड़ रहा है।
सरिता – इसकी आँखें चेक करवानी थी। इसकी मैम कहती है कि छोटे अक्षर पढ़ने में परेशानी होती है।
रचना – पर ऐसा क्यों?
सरिता – यह स्कूल से आते ही बैग फेंककर कार्टून देखने लगता है। यह कई-कई घंटे टीवी देखता रहता है।
रचना – अधिक टीवी देखने का आँखों पर बुरा असर पड़ता है।
सरिता – डॉक्टर ने इसका वजन भी अधिक बताया है।
रचना – टीवी देखने से मोटापा तो बढ़ता ही है।
सरिता – अब इसके साथ कड़ाई से पेश आना होगा।
रचना – कड़ाई नहीं प्यार से इसे समझाओ और डॉक्टर की राय मानो।
सरिता – धन्यवाद रचना।

प्रश्नः 13.
पुत्र अपने जन्मदिन पर उपहारस्वरूप मोबाइल फ़ोन चाहता है। इस संबंध में पिता-पुत्र के मध्य संवाद लिखिए।
उत्तर:
पुत्र – पिता जी, आने वाली पंद्रह तारीख को मेरा जन्मदिन है। याद है न आपको?
पिता – अरे! मैं अपने बेटे का जन्मदिन कैसे भूल सकता हूँ।
पुत्र – पिता जी, इस बार मैं अपने दो-तीन नए दोस्तों को भी बुलाऊँगा।
पिता – यह तो और भी अच्छा रहेगा। अरे हाँ! जानते हो, इस बार मैंने उपहार में देने के लिए क्या सोचा है?
पुत्र – नहीं मुझे कैसे पता?
पिता – मैं तुम्हें ‘हमारे वैज्ञानिक’ नामक पुस्तक देना चाहता हूँ।
पुत्र – पर पिता जी, इस बार मुझे अच्छे किस्म वाला मोबाइल फ़ोन चाहिए।
पिता – बेटा, मोबाइल फ़ोन से तुम्हारी पढ़ाई में व्यवधान उत्पन्न होगा। इसके अलावा तुम इसका दुरुपयोग भी कर सकते हो।
पुत्र – मेरे और मित्र तो फ़ोन लाते हैं।
पिता – विद्यालयों में विद्यार्थियों को फ़ोन लाने पर रोक है। ज़रा सोचो, अध्यापिका पढ़ा रही हो और फ़ोन की अलग-अलग ध्वनियाँ आने लगें, तो कैसा रहेगा?
पुत्र – आप ठीक कह रहे हैं। इससे पढ़ाई में व्यवधान उत्पन्न होगा। आप मुझे उपहार में पुस्तक ही दीजिएगा।
पिता – ये हुई न राजा बेटे वाली बात।

प्रश्नः 14.
बिना हेलमेट के पहने बाइक चला रहे युवक और ट्रैफिक इंस्पेक्टर के बीच हुई बातचीत का संवाद लेखन कीजिए।
उत्तर:
इंस्पेक्टर – इधर … इधर रोको अपनी बाइक को।
युवक – बताइए क्या बात है?
इंस्पेक्टर – बताइए! अच्छा, तुम्हारा हेलमेट कहाँ है? ।
युवक – मुझे ज्यादा दूर नहीं जाना था, सो बिना हेलमेट के निकल आया।
इंस्पेक्टर – तुम लाल बत्ती की परवाह किए बिना क्यों चले आ रहे हो?
युवक – मुझे थोड़ा जल्दी पहुँचना था।
इंस्पेक्टर – सड़क सुरक्षा के नियमों का उल्लंघन करना कहाँ तक उचित है। चलो डी.एल. दिखाओ।
युवक – वह तो मैं घर भूल आया।
इंस्पेक्टर – फिर तो तुम्हारा चालान ज़रूर का,गा। निकालो 1200 रुपये।
युवक – आज छोड़ दीजिए भविष्य में ऐसा नहीं करूँगा।
इंस्पेक्टर – यह रहा तुम्हारा चालान।
युवक – थोड़ी-सी लापरवाही तो भारी पड़ गई। लीजिए ये रुपये।

प्रश्नः 15.
‘कार्टून फ़िल्मों का बच्चों पर प्रभाव’ विषय पर दो महिलाओं के मध्य हुए संवाद को लिखिए।
उत्तर:
रमा – नमस्ते सुमन! आज काफ़ी दिन बाद पार्क में आई हो?
सुमन – नमस्ते रमा! आना तो मैं रोज़ चाहती हूँ पर ……………….
रमा – पर क्या? आलस्य रोक लेता है?
सुमन – नहीं वो बात नहीं है? वो मेरा बेटा काव्य है न ………………
रमा – पहले तो तुम उसके साथ शाम को ज़रूर ही आ जाती थी, पर अब काव्य ने क्या कर दिया?
सुमन – आजकल वह स्कूल से आते ही टेलीविजन से चिपक जाता है। वह कार्टून देखते हुए ही नाश्ता करता है, अपनी पढ़ाई का काम करता है।
रमा – आजकल यह प्रवृत्ति बच्चों में तो बढ़ रही है, जो चिंता का विषय है।
सुमन – इन कार्टून कार्यक्रमों के कारण बच्चों में लड़ने-झगड़ने और हिंसक प्रवृत्ति अपनाने की दुष्प्रवृत्ति पनप रही है।
रमा – ठीक कहती हो, अमेरिका में एक बच्चे ने अपने छोटे भाई के साथ ही हिंसक व्यवहार किया था।
सुमन – काव्य को भी मार-धाड़ वाले कार्टून ही अच्छे लगते हैं। समझ में नहीं आता क्या करूँ? मन तो करता है कि टी.वी. ही बेच दूँ।
रमा – अरे नहीं। सारे कार्टून बुरे नहीं होते हैं। बहुत-सी कार्टून फिल्में शिक्षाप्रद एवं मनोरंजक होती हैं। तुम उसे ऐसे कार्टून देखने के लिए प्रेरित करो।
सुमन – धन्यवाद! अच्छा अब चलते हैं।

प्रश्नः 16.
गरमी की छुट्टी बिताकर लौटे दो मित्रों की बातचीत का संवाद लेखन कीजिए।
उत्तर:
शरद – नमस्ते अमर! कैसे हो?
अमर – नमस्ते! शरद मैं तो ठीक हूँ। कहो इस बार कहाँ घूमने गए थे।
शरद – मैं अपने दादा-दादी के पास इलाहाबाद घूमने गया था।
अमर – तुम्हारे दादा-दादी शहर में ही रहते हैं क्या?
शरद – नहीं शरद वहाँ से आठ किलोमीटर दूर एक गाँव में।
अमर – फिर तो गाँव में बड़ा मज़ा आया होगा!
शरद – ताज़ी हवा, खुला वातावरण, गंगा नदी का किनारा, कलरव करते पक्षी, ताजे फल और सब्जियाँ सब कुछ बहुत अच्छा था वहाँ।
अमर – प्रकृति के करीब जाने का अपना अलग ही आनंद है।
शरद – पर अमर तुम कहाँ गए थे, इन छुट्टियों में?
अमर – मैं देहरादून गया था।
शरद – इलाहाबाद में तो बड़ी गरमी थी, पर देहरादून में इतनी गरमी न रही होगी।
अमर – असली मज़ा तो मसूरी में आया। वहाँ गरमी न थी।
शरद – यही तो अपने देश का मज़ा है, कही गरमी है तो कहीं सरदी।
अमर – तभी तो अपना भारत महान है।

प्रश्नः 17.
बूढ़ा किसान अपने खेतों के पास की खाली ज़मीन में पौधे लगा रहा था। उसके और बालक अक्षय के बीच हुई बातचीत को संवाद रूप में लिखिए।
उत्तर:
अक्षय – राम-राम काका। इस धूप में आप यहाँ क्या कर रहे हैं।
किसान – बेटा राम-राम। आज इन फ़सलों को पानी लगाना है तो …।
अक्षय – पर आप तो खेत के किनारे गड्ढे क्यों खोद रहे हैं ?
किसान – इन गड्ढों में पौधे लगाऊँगा और फ़सलों के साथ-साथ इन्हें भी पानी दे दूंगा।
अक्षय – ये कौन-कौन से पौधे हैं ?
किसान – इनमें नीम शीशम बरगद, आम, जामुन आदि के पेड़ हैं।
अक्षय – इससे तुम्हारे खेत के किनारे छाया रहेगी और तुम्हें धूप नहीं लगेगी न।
किसान – हाँ बेटा और आने-जाने का फल भी मिलेगा।
अक्षय – पर आप तो बूढ़े हो गए हो। आप तो इन पेड़ों के फल खाने के लिए शायद ही जिंदा रहें।
किसान – बेटा मैंने तो दूसरों के लगाए पेड़ों के फल खाए हैं। अब मेरे लगाए पेड़ों से दूसरों को फल मिलेंगे।
अक्षय – मैं भी बड़ा होकर परोपकार करूँगा।
किसान – यह तो बहुत अच्छा होगा बेटा! जुग जुग जियो।

प्रश्नः 18.
बढ़ती रिश्वतखोरी के संबंध में दो मित्रों के मध्य संवाद लिखिए।
उत्तर:
केशव – नमस्ते अनुज! क्या हालचाल है?
अनुज – नमस्ते केशव! सब ठीक-ठाक है।
केशव – इस धूप में कहाँ से चले आ रहे हो? अनुज
अनुज – बिजली के दफ्तर से।
केशव – क्या समस्या आ गई?
अनुज – केशव पिछले चार बार से दफ़्तर वाले तीन-चार गुना तक बिल भेज रहे हैं। जब भी जाओ, कह देते हैं कि इस बार जमा करा दो, अगली बार ठीक हो जाएगा।
केशव – उन्होंने बिल ठीक कर दिया?
अनुज – नहीं केशव। कल कार्यालय से एक आदमी आया था। उसने कुछ कागज, पुराने बिलों की फोटोप्रति ली और ऑफिस आने को कहा।
केशव – क्या आज वह मिला फिर?
अनुज – फिर क्या, आज वही व्यक्ति दफ्तर में मिला और बिल ठीक करने के पाँच हजार रुपए रिश्वत माँग रहा है।
केशव – अनुज हर दफ़्तर का यही हाल है। बिना कुछ दिए कोई काम होता ही नहीं।
अनुज – मैं तो रिश्वत दूंगा नहीं। मैं इसकी शिकायत करूँगा।
केशव – तुमने ठीक ही सोचा है। मैं भी तुम्हारा साथ दूंगा।
अनुज – जी धन्यवाद।

प्रश्नः 19.
परीक्षा भवन से निकले दो छात्रों की बातचीत का संवाद-लेखन कीजिए।
उत्तर:
विजय – अरे संजय! कैसे हुआ तुम्हारा पेपर?
संजय – मज़ा आ गया। पेपर तो बहुत अच्छा था।
विजय – पर इस बार अपठित अंश में बहुविकल्पी प्रश्नःःः तो थे ही नहीं।
संजय – अब प्रश्नःःःों के उत्तर देने का तरीका बदल जा गया है।
विजय – कब से हुआ यह सब?
संजय – लगता है कि जब अध्यापक ने हमें बताया और समझाया था तब तुम नहीं थे कक्षा में।
विजय – तभी तो मैं कहूँ ये पेपर कैसे उल्टा-सीधा आ गया।
संजय – याद है उन दिनों तुम खूब छुट्टियाँ करके घर बैठ जाया करते थे।
विजय – उसका परिणाम अब भुगतना पड़ रहा है। यह तो अच्छा रहा कि पाठ्यपुस्तकें मैं ढंग से पढ़कर आया था वरना पास होने के भी लाले पड़ जाते।
संजय – इस परीक्षा के बाद अब नए सिरे से पढ़ाई में जुट जाओ और सभी तरह के बदलावों को अध्यापक एवं सहपाठियों से समझ लो ताकि ऐसी स्थिति न आए।
विजय – संजय अब ऐसा ही करूँगा।

प्रश्नः 20.
मिलावट की समस्या पर दो महिलाओं के मध्य हुए संवाद को लिखिए।
उत्तर:
विभा – नमस्ते उमा बहन! कहाँ से आ रही हो?
उमा – नमस्ते विभा! मैं बाज़ार से आ रही हूँ।
विभा – अरे हाँ! आज तो तुमने कुछ ज़्यादा ही खरीदारी कर रखी है।
उमा – खरीदारी नहीं विभा, कल नौकर को बाज़ार भेज दिया सामान लाने। उस दुकानदार ने रेट तो पूरे लिए पर नौकर को मिलावटी सामान पकड़ा दिए।
विभा – फिर क्या तुमने?
उमा – पहले तो चावलों से कंकड़-पत्थर, चने से गेहूँ, मटर कुछ साफ़ करने की कोशिश की, पर मिलावट के कारण साफ़ करना मुश्किल हो रहा था।
विभा – तुम्हें सामान को वापस कर देना चाहिए था। ये दुकानदार ज्यादा लाभ कमाने के चक्कर में चावल में कंकड़, अरहर में खेसारी की दाल, धनिया में बुरादा, घी में डालडा आदि मिलाते हैं।
उमा – उसी सामान को वापस करने गई थी। मैंने जब उपभोक्ता फोरम जाने की धमकी दी, तब उसने सामान वापस किया।
विभा – फिर ये सामान?
उमा – ये सामान में सरकारी केंद्रीय भंडार से ला रही हूँ।
विभा – यह समझदारी वाली बात हुई।
उमा – धन्यवाद! अच्छा अब चल जाए।

प्रश्नः 21.
बढ़ती बेरोजगारी से दुखी दो मित्रों की बातचीत का संवाद-लेखन कीजिए।
उत्तर:
अनिल – अरे अमित! यह थैला लटकाए कहाँ से आ रहे हो?
अमित – अनिल, मैं रोज़गार दफ़्तर से आ रहा हूँ।
अनिल – क्या नौकरी के रोज़गार दफ्तर से बुलावा आ गया है?
अमित – नहीं अनिल, अपना रजिस्ट्रेशन करवाने गया था, पर लाइन इतनी लंबी थी कि लौट आना पड़ा।
अनिल – यह लाइन वाली समस्या तो हर जगह की होती जा रही है।
अमित – जनसंख्या जो इतनी ज़्यादा होती जा रही है। इससे बेरोज़गारी बढ़ती जा रही है।
अनिल – माँ-बाप इतनी महँगी फ़ीस देकर पढ़ाई करवाते हैं और फिर काम न मिलने पर कितनी निराशा होती है।
अमित – बेरोज़गारी के कारण युवाशक्ति अवांछित गतिविधियों में शामिल होने लगती है।
अनिल – भूखा व्यक्ति आखिर हर काम करने पर उतार आता है।
अमित – मैं तो नौकरी करने के बजाय अपना रोज़गार लगाऊँगा ताकि कुछ और लोगों को भी काम दे सकूँ।
अनिल – काश ऐसा हर नवयुवक सोचता तो स्थिति ही कुछ और होती।

प्रश्नः 22.
शहरों में बढ़ते प्रदूषण के संबंध में दो मित्रों के मध्य संवाद को लिखिए।
उत्तर:
अंकुर – नमस्ते! क्या मैं आपका नाम जान सकता हूँ?
प्रताप – जी, मेरा नाम प्रताप है।
अंकुर – लगता है कहीं बाहर से दिल्ली आए हो।
प्रताप – जी, मैं शिमला के एक छोटे-से गाँव से आया हूँ, यहाँ परीक्षा देने।
अंकुर – फिर तो तुम्हारा गाँव हरा-भरा और प्रदूषण मुक्त होगा।
प्रताप – ठीक कहा आपने। यहाँ दिल्ली की हवा और वहाँ की हवा में भी बहुत अंतर है।
अंकुर – वहाँ खूब सारे पेड़ हैं और यहाँ ज़हर उगलती फैक्ट्रियाँ, ईंट, सीमेंट और कंक्रीट के जंगल हैं।
प्रताप – वहाँ की नदियों का पानी निर्मल है, जिसे देखते ही पीने का मन होता है।
अंकुर – तुम यहाँ की यमुना को देखो। कितनी गंदी हो चुकी है। इसके पानी को पीना तो दूर नहाने का भी मन नहीं करता।
प्रताप – आखिर ऐसा क्यों?
अंकुर – यहाँ नालों और फैक्ट्रियों का दूषित जल नदी में मिल जाता है।
प्रताप – यहाँ शोर-शराबा भी तो है?
अंकुर – यहाँ मोटर-गाड़ियों के हॉर्न, उनका शोर कभी-कभी कान बंद करने को विवश कर देता है।
प्रताप – इसका कारण क्या हो सकता है?
अंकुर – इसका कारण स्पष्ट रूप से बढ़ती जनसंख्या है।

प्रश्नः 23.
दुकान पर सामान खरीदने आए ग्राहक और दुकानदार के मध्य हुई बातचीत को संवाद रूप में लिखिए।
उत्तर:
ग्राहक – लाला जी राम-राम!
दुकानदार – बताइए क्या-क्या आपको?
ग्राहक – अभी तो आप 250 ग्राम वाली चाय का एक पैकेट दे दीजिए।
दुकानदार – यह लो बहुत ही अच्छी चाय है। पीकर याद करोगे।
ग्राहक – कितने रुपये दूं?
दुकानदार – जी अस्सी रुपये।
ग्राहक – पर इसका असली दाम तो आपने खुरचकर मिटा दिया है। यह स्टिकर क्यों लगा है।
दुकानदार – अब यही इसका नया दाम है। लेना हो लो … ।
ग्राहक – दाम तो नया है पर चाय तो पुरानी वाली है। इसका दाम भी पुराना वाला ही आपको लेना चाहिए। लेते हैं पुराना दाम या करूँ सौ नंबर पर फोन?
दुकानदार – फ़ोन मत कीजिए। मुझे पैंसठ रुपये दे दीजिए।
ग्राहक – अब आप भविष्य में ऐसा नहीं करेंगे।
दुकानदार – जी ठीक है।

प्रश्नः 24.
गांधी आश्रम पर कपड़ा खरीदने आए युवक से विक्रेता की जो बातचीत हुई होगी, उसका संवाद लेखन कीजिए।
उत्तर:
युवक – नमस्ते सर, मुझे अपने लिए कपड़ा चाहिए। किस काउंटर पर जाऊँ?
कर्मचारी – आप दाहिनी ओर के काउंटर-7 पर जाएँ।
युवक – मुझे शर्ट के लिए कपड़ा चाहिए।
कर्मचारी – यहाँ उत्तम कोटि के खादी वस्त्र मिलेंगे।
युवक – वह चेकवाला कपड़ा दिखाएँ और दाम बताए।
कर्मचारी – यह बहुत अच्छा कपड़ा है। इसका दाम चार सौ रुपये प्रति मीटर है।
युवक – चार सौ रुपये प्रति मीटर यानी दो मीटर के ₹ 800। यह तो बहुत महँगा है।
कर्मचारी – पर इन कपड़ों को पहनने का आनंद अलग है। इनकी क्वालिटी उत्तम है।
युवक – मैं कोई और कपड़ा खरीद लूँगा।
कर्मचारी – पर खादी का यह कपड़ा हमारे मन में देश प्रेम और देशभक्ति की भावना बढ़ाता है। यह हमारे देश में ही बना है।
युवक – फिर तो आप दो कमीजों का कपड़ा दे दीजिए।
कर्मचारी – जी धन्यवाद। देश को आप जैसे युवाओं की ज़रूरत है।

NCERT Solutions for Class 9 Hindi

The post CBSE Class 9 Hindi B संवाद लेखन appeared first on Learn CBSE.

CBSE Class 9 Hindi B विज्ञापन लेखन

$
0
0

CBSE Class 9 Hindi B लेखन कौशल विज्ञापन लेखन

विज्ञापन शब्द की रचना ‘ज्ञापन’ शब्द में ‘वि’ उपसर्ग लगाने से हुई है, जिसका शाब्दिक अर्थ है-विशेष जानकारी देना। अर्थात किसी वस्तु की बिक्री बढ़ाने के लिए उस वस्तु के गुणों का प्रचार-प्रसार करना ही विज्ञापन कहलाता है। विज्ञापन का उद्देश्य होता है-उत्पादक द्वारा अपनी वस्तुएँ खरीदने के लिए लोगों को आकर्षित और लालायित करना तथा उन्हें बेचकर मोटा मुनाफ़ा कमाना। आज जिधर भी देखो, विज्ञापन किसी न किसी वस्तु का गुणगान करते नज़र आते हैं। टेलीविजन के चैनेल, समाचार पत्र-पत्रिकाएँ, होर्डिंग्स, साइन बोर्ड, बस स्टैंड, दीवारें मेट्रो, बस तथा वाहनों की दीवारें आदि पर विज्ञापन दिखाई देते हैं। अब फ़िल्मों के बीचबीच में इतने विज्ञापन आने लगे हैं कि पता ही नहीं लगता कि हम विज्ञापन देख रहे हैं या फ़िल्म। इनके अलावा कुछ विज्ञापन सरकारी एजेंसियों द्वारा लोगों को जागरूक करने के लिए प्रसारित किए जाते हैं। राष्ट्रीय एकता, जनसंख्या वृद्धि रोकने संबंधी, रोगों से बचाव, मद्यपान न करने संबंधी तथा समय-समय पर मच्छर-मलेरिया रोकने संबंधी विज्ञापन इसी कोटि में आते हैं।

विज्ञापन-लेखन करते समय निम्नलिखित बातों का ध्यान रखना चाहिए।

  1. कम से कम शब्दों में विज्ञापन होना चाहिए।
  2. शब्दों में गागर में सागर भरने की क्षमता होनी चाहिए।
  3. भाषा सरस, रोचक तथा प्रभावपूर्ण होनी चाहिए।
  4. विज्ञापन की भाषा काव्यात्मक हो तो बेहतर रहता है।
  5. विज्ञापन की भाषा में मुहावरे तथा सूक्तियों का प्रयोग होने से भाषा साधारण लोगों की समझ में भी आ जाती है।
  6. रंगीन, आकर्षक एवं बड़े चित्र को जगह अवश्य देनी चाहिए।
  7. प्रस्तुतीकरण सबसे अलग एवं नवीन हो।
  8. उपभोक्ताओं को आकर्षित करने वाले शब्दों का प्रयोग करना चाहिए।

विज्ञापन-लेखन के कुछ उदाहरण-

1. विराट मोबाइल फ़ोन बनाने वाली कंपनी के लिए एक विज्ञापन तैयार कीजिए।
CBSE Class 9 Hindi B विज्ञापन लेखन -1

2. ‘प्लाजा’ कार कंपनी के लिए एक विज्ञापन तैयार कीजिएं।
CBSE Class 9 Hindi B विज्ञापन लेखन -2 - Copy

3. ‘मोहित’ बैग बनाने वाली कंपनी के लिए एक विज्ञापन तैयार कीजिए।
CBSE Class 9 Hindi B विज्ञापन लेखन -3

4. ‘ज्योति घड़ी’ बनाने वाली कंपनी के लिए एक विज्ञापन तैयार कीजिए।
CBSE Class 9 Hindi B विज्ञापन लेखन -4 - Copy

5. ‘रॉयल टेलीविज़न’ बनाने वाली कंपनी के लिए विज्ञापन तैयार कीजिए।
CBSE Class 9 Hindi B विज्ञापन लेखन -5

6. ‘उत्सव’ छाता बनाने वाली कंपनी के लिए विज्ञापन तैयार कीजिए।
CBSE Class 9 Hindi B विज्ञापन लेखन -6 - Copy

7. ‘शीतल’ ए.सी. कंपनी के लिए विज्ञापन तैयार कीजिए।
CBSE Class 9 Hindi B विज्ञापन लेखन -7

8. ‘सुविधा’ वाशिंग मशीन बनाने वाली कंपनी के लिए विज्ञापन तैयार कीजिए। आधी आबादी को आराम पहुँचाने सुविधा वाशिंग मशीन आ गई है
CBSE Class 9 Hindi B विज्ञापन लेखन -8

9. ‘चित्रा’ पेंसिल बनाने वाली कंपनी के लिए विज्ञापन तैयार कीजिए।
CBSE Class 9 Hindi B विज्ञापन लेखन -9

10. ‘शक्ति प्राश’ नामक फूड सप्लीमेंट बनाने वाली कंपनी के लिए विज्ञापन तैयार कीजिए।
CBSE Class 9 Hindi B विज्ञापन लेखन -10

11. ट्रैक्टर बनाने वाली कंपनी ‘सूमो’ के लिए ट्रैक्टर की विशेषताएँ बताते हुए विज्ञापन तैयार कीजिए।
CBSE Class 9 Hindi B विज्ञापन लेखन -11

12. ‘विद्यार्थीप्राश’ बनाने वाली कंपनी के लिए विज्ञापन तैयार कीजिए। क्या आपका बच्चा पढ़ाई में कमज़ोर है ? क्या वह पढ़ाई में पिछड़
CBSE Class 9 Hindi B विज्ञापन लेखन -12

13. ‘गणित’ विषय के लिए होमट्यूशन पढ़ाने वाली संस्था ‘मल्होत्रा ट्यूटोरियल्स’ के लिए विज्ञापन तैयार कीजिए। समाधान है
CBSE Class 9 Hindi B विज्ञापन लेखन -13

14. आप अपना मकान बेचना चाहते हैं। उसके मूल्य एवं अन्य खूबियों का उल्लेख करते हुए एक विज्ञापन तैयार कीजिए।
CBSE Class 9 Hindi B विज्ञापन लेखन -14

15. ‘सुमन’ साड़ियों की बिक्री बढ़ाने हेतु एक विज्ञापन तैयार कीजिए।
CBSE Class 9 Hindi B विज्ञापन लेखन -15

16. ‘विद्यार्थी पुस्तक भंडार’ में पुस्तकों की बिक्री बढ़ाने हेतु एक विज्ञापन तैयार कीजिए। । विद्यार्थी पुस्तक भंडार
CBSE Class 9 Hindi B विज्ञापन लेखन -16

17. आपकी गली में तनु ब्यूटी पार्लर खुला है। इसके लिए विज्ञापन तैयार कीजिए।
CBSE Class 9 Hindi B विज्ञापन लेखन -17

18. आपके पड़ोस में ‘गोपाल डेयरी’ खुल गई है। इसके लिए विज्ञापन तैयार कीजिए।
CBSE Class 9 Hindi B विज्ञापन लेखन -18

19. ‘उत्तर प्रदेश पर्यटन विभाग’ पर्यटकों के लिए कई आकर्षक योजनाएँ लाया है। उनके संबंध में एक विज्ञापन तैयार कीजिए।
CBSE Class 9 Hindi B विज्ञापन लेखन -19

20. ‘राजस्थान पर्यटन विभाग’ की आय बढ़ाने के लिए एक विज्ञापन तैयार कीजिए।
CBSE Class 9 Hindi B विज्ञापन लेखन -20

21. ‘समीर’ पंखा बनाने वाली कंपनी की बिक्री बढ़ाने के लिए विज्ञापन तैयार कीजिए।
CBSE Class 9 Hindi B विज्ञापन लेखन -21

22. ‘हरियाली’ पौधशाला में विविध प्रजातियों के पौधे छूट के साथ उपलब्ध हैं। उनकी बिक्री बढ़ाने वाला विज्ञापन तैयार कीजिए।
CBSE Class 9 Hindi B विज्ञापन लेखन -22

23. ‘गंगाजल’ नामक बोतल बंद पानी उपलब्ध कराने वाली कंपनी की बिक्री के लिए विज्ञापन तैयार कीजिए।
CBSE Class 9 Hindi B विज्ञापन लेखन -23

24. ‘सौम्या शैंपू’ बनाने वाली कंपनी की बिक्री बढ़ाने के लिए एक विज्ञापन तैयार कीजिए।
CBSE Class 9 Hindi B विज्ञापन लेखन -24

25. आपकी गली में ही ‘हॉली हार्ट’ प्ले स्कूल खुल गया है। इसके लिए विज्ञापन तैयार कीजिए।
CBSE Class 9 Hindi B विज्ञापन लेखन -25

26. ‘सेंचुरी क्रिकेट अकादमी’ में प्रवेश बढ़ाने के लिए एक विज्ञापन तैयार कीजिए।
CBSE Class 9 Hindi B विज्ञापन लेखन -26

27. ‘रूप निखार’ साबुन बनाने वाली कंपनी साबुनों की बिक्री बढ़ाना चाहती है। इसके लिए विज्ञापन तैयार कीजिए।
CBSE Class 9 Hindi B विज्ञापन लेखन -27

28. ‘रोशनी’ मोमबत्ती बनाने वाली कंपनी अपनी बिक्री बढ़ाने के लिए विज्ञापन बनवाना चाहती है। इस कंपनी के लिए आप विज्ञापन लेखन कीजिए।
CBSE Class 9 Hindi B विज्ञापन लेखन -28

29. ‘आस्था’ अगरबत्तियाँ बनाने वाली कंपनी के लिए विज्ञापन तैयार कीजिए।
CBSE Class 9 Hindi B विज्ञापन लेखन -29

30. ‘रफ़्तार’ ट्रेवेल्स एवं ट्रेवेल एजेंसी को ड्राइवर्स की आवश्यकता है। आवश्यक योग्यताओं का उल्लेख करते हुए विज्ञापन तैयार कीजिए।
CBSE Class 9 Hindi B विज्ञापन लेखन -30

NCERT Solutions for Class 9 Hindi

The post CBSE Class 9 Hindi B विज्ञापन लेखन appeared first on Learn CBSE.

Dr. Abdul Kalam Scholarship | Application Form 2019, Eligibility, Awards

$
0
0

Abdul Kalam Scholarship: Abdul Kalam Scholarship is not only offered by a single organization but also named after India’s former president. It is also offered by various state governments, educational institutions and other organizations to commemorate Dr. APJ Abdul Kalam’s memories and help achieve the mission of making India a 100% literate country by providing students with educational opportunities. All deserving and needy candidates are offered financial support by the scholarships or fellowships in order to complete their education.

Under this system of Abdul Kalam fellowship, Jamia Millia Islamia’s 50 learners obtain tuition reimbursement pursuant to an Rs. 10,000 total per pupil per year. In addition, 50% of the complete grants are intended for woman learners.

We discussed the significant information of all the scholarships designated after APJ Kalam in this paper. The catalog of these scholarships/fellowships can be found here along with their registration duration, benefits, registration, and implementation method. The chart of all the scholarships designated Abdul Kalam Scholarship is given below.

Abdul Kalam Scholarship – Time Table

Scholarship NameProviderApplication Period
APJ Abdul Kalam Scholarship, KeralaDepartment of Minority Welfare, Government of KeralaAugust to November
Dr. APJ Abdul Kalam ScholarshipsJamia Millia IslamiaDecember to January
Abdul Kalam Technology Innovation National FellowshipScience & Engineering Research Board (SERB)December
President APJ Abdul Kalam Postgraduate FellowshipUniversity of South Florida (USF)January to March
Dr. APJ Abdul Kalam IGNITE AwardsNational Innovation FoundationApril to August
Fulbright Kalam Climate FellowshipUnited States-India Educational Foundation (USIEF)January to July
Dr. Abdul Kalam International Undergraduate ScholarshipsUniversity of SydneyDuring Semester-I in August and Semester-II in May
Dr. APJ Abdul Kalam Summer Training ProgramAcademy of Scientific & Innovative Research (AcSIR)December to January

Abdul Kalam Scholarship – Rewards

Applicants are chosen for any of the Abdul Kalam Scholarships financed by various organizations to obtain economic support to achieve a good education. Find below the benefits information provided by each of the above-described scholarships/fellowships to the learners.

Abdul Kalam Scholarship – Individual Award Details

Each scholarship of Abdul Kalam listed in this article has its own application procedure. It is possible to apply most of these scholarships through the online application system. The table below tells you how to apply for these scholarships and where to apply before the end of their deadline.

Each scholarship/fellowship referred to in this article requires different criteria for eligibility. School students, undergraduates or postgraduates can make use of these schemes. The conditions of eligibility that applicants must fulfill in order to benefit from scholarship benefits are set out below.

Applicants selected for any of the Abdul Kalam Scholarships funded by various organizations receive financial support to pursue a quality education. Find below the details of the rewards offered by each of the above-mentioned scholarships/fellowships to the students.

APJ Abdul Kalam Scholarship, Kerala

Application Process: Interested applicants can register for this scholarship internet through the state government’s Minority Welfare Scholarships Portal.

Eligibility

  • This is a minority scholarship and can, therefore, be implemented by learners from minority groups like Muslim, Christian, Sikh, Buddhist, Parsi, and Jain.
  • The applicant’s estimated household earnings must not exceed Rs. 8 Lakh.
  • The applicant must be the Kerala state’s indigenous.
  • The candidate must take a merit-based government / assisted provincial certificate classes.
  • Candidates of the BPL are provided priority while disbursing this grant.

Reward: The chosen learners earn an Rs. 6000 annual scholarship to pursue a 3-year diploma path in government/assisted polytechnics.

Dr. APJ Abdul Kalam Scholarships, Jamia Millia Islamia

Application Process: Applicants can register by downloading the request type from Jamia Millia Islamia’s university portal and submitting it to the Dean’s cabinet after completing it and attaching the required documents.

Eligibility

  • The candidates must be Jamia Millia Islamia’s bonafide teachers and study periodic UG/PG classes.
  • The candidates must have at least 75 percent participation.
  • The candidates must have either completed the committee of school 12 or acquired a degree of UG.
  • The applicants’ annual household earnings shall not exceed Rs. 2 Lakh.

Reward: Under this system of Abdul Kalam fellowship, Jamia Millia Islamia’s 50 learners obtain tuition reimbursement pursuant to an Rs. 10,000 total per pupil per year. In addition, 50% of the complete grants are intended for woman learners.

Dr. Abdul Kalam International Undergraduate Scholarships

Application Process: Students can register the internet through Sydney University’s internet implementation scheme.

Eligibility

  • This scholarship is available for international learners.
  • For any UG program at the Faculty of Engineering and Information Technologies at Sydney University, the applicants must have an unconditional grant.
  • The applicants must have finished either an Australian Year 12 diploma or an ATAR equal of at capped 98 global junior secondary diploma.

Reward: This scholarship includes only one year 50 percent of the chosen applicant’s education charges.

Dr. APJ Abdul Kalam Summer Training Program

Application Process: You can apply for this program directly through AcSIR’s formal portal.

Eligibility

  • Applicants enrolled in the first-year MSc program (pre-MSc degree category minimum first-class) are qualified.
  • The candidates may also qualify for a GPA rating of 6.0 on a scale of 10 in the fifth to pre-final year of Integrated MSc program or equal.
  • GPA rating of 8.0 on a scale of 10 or GPA 7.0 on a scale of 10 for CFT Is at B.E/B. Tech or GATE Score of 600 or NET (Engineering) rank of 1 to 100 may also extend to first-year learners of ME / MTech program or similar.
  • Applicants are qualified to register for a BE / B.Tech program or equal to PA 8.0 on a scale of 10 or GPA 7.0 on a scale of 10.
  • Also eligible are the third to the pre-final year students of the Integrated ME / MTech or equivalent program (GPA 8.0 on a scale of 10 OR GPA 7.0 on a scale of 10 for CFTIs or GATE Score alone or NET).

Reward

  • For this program, a maximum of 20 applicants is chosen and obtain an Rs. 25,000 worth grant for a duration of 2 months.
  • Candidates also receive an Rs. 5000 transport subsidy.

Abdul Kalam Technology Innovation National Fellowship

Application Process: Applicants can submit their application/appointment through their organizations/institutions heads.

Eligibility

  • The candidates must be Indians employed in India at reputable publicly funded organizations in various engineering capabilities.
  • Bachelor’s degree and skilled professional qualification must be held by the candidates.
  • As on the eve of the grant of this grant, the applicants must have at least 5 years of work remaining in the mother organization.

Reward

  • The applicants earn a grant amounting to Rs. 25,000 per month under this Abdul Kalam grant.
  • The chosen applicants also receive an annual Rs. 15 Lakh research grant to be used for design, study and development operations, including consumables, recruitment of resources, national/international science transport, etc.
  • The housing centers are also provided an expense quantity of Rs. 1 Lakh per year.

President APJ Abdul Kalam Postgraduate Fellowship

Application Process: Please submit your finished request type by email to KalamFellowship@usf.edu.

Eligibility

  • The candidates must have completed their Indian university degrees.
  • Before submitting their fellowship application, the applicants must have submitted the University of South Florida’s graduate application form and received the university ID code.
  • In the previous topics, candidates must be prepared to follow a Ph.D. degree.

Reward

  • For a total duration of 4 years, the chosen applicants receive a tuition fee waiver.
  • The pupils are also awarded a USD 22,000 stipend.
  • The University of South Florida also charges full tuition (up to 12 courses per class) for USF graduate-level credit hours that work towards the graduate degree of the student.

Fulbright Kalam Climate Fellowship

Application Process: Through the gateway of the Institute of International Education, applicants can register the internet.

Eligibility

  • The candidates are required to reside in India and Indian people.
  • The candidates ‘ educational records must be outstanding.
  • The applicants must have English language skills.
  • Candidates applicants for doctoral studies must be enrolled in an Indian Ph.D. program and have undertaken appropriate studies in the appropriate sector, particularly in resource detection in India and the United States.
  • Applicants for postdoctoral research must have earned a Ph.D. degree in the previous four years.

Rewards: The chosen applicants are qualified for various advantages under this grant provided by USIEF, such as J-1 visa assistance, round-journey airfare between India and the US, a monthly stipend, skilled compensation and much more.

Dr. APJ Abdul Kalam IGNITE Awards

Application Process: Students can apply online or by email/post for this award.

Eligibility

  • The students up to Class 12 can apply this Abdul Kalam award.
  • The applicants must not be over the age of 17-18.
  • The applicants who have the maximum age limit out of school are 17 years old.

Reward: In an exhibition, the students get to show their project. The students also receive the train’s travel expenses.

FAQ’s on Abdul Kalam Scholarship

Question 1.
How many different types of scholarships are available under the Abdul Kalam scholarship?

Answer:
A total of 8 different types of scholarships are available under the Abdul Kalam scholarship.

Question 2.
Is there a different process to be followed for all the scholarships?

Answer:
Yes, there is a different application process, eligibility criteria, and rewards for each of these scholarships.

Scholarships for Students

The post Dr. Abdul Kalam Scholarship | Application Form 2019, Eligibility, Awards appeared first on Learn CBSE.

ST Scholarship | Eligibility, Benefits, Application, and Selection Process 

$
0
0

ST Scholarship: ST Scholarship is an initiative of the Ministry of Tribal Affairs, Government of India. This scholarship is awarded only to the scheduled tribe (ST) category students. This scholarship offers financial support to ST students to complete their studies at both school and college level. This scholarship initiated to encourage ST students for higher education. This scholarship empowers the ST students from the deprived section of people to undertake education. This Scholarship helps ST students from lower-income families to pursue their education.

ST Scholarship organized by Govt. of India to offer an award to ST students. Those students who are not able to fulfill their dreams due to financial hurdles. Application for this Scholarship releases on the website of the National Scholarship portal. The Ministry of Tribal Affairs is implementing various types of ST Scholarships. Those are the Pre-matric Scholarship, Post-matric Scholarship, Rajiv Gandhi National Fellowship, and Overseas Scholarship. Read the article below to know the complete information about ST Scholarship.

Scholarships Under ST Scholarship

Scholarship Division of the Ministry of Tribal Affairs offers financial support to individuals/ Institutes/ States according to schematic norms. The Ministry of Tribal Affairs is implementing the below-given scholarships for ST students in the country.

Scholarship TypesApplication DatesNumber of Scholarships
Post-matric ScholarshipMay to June
Pre-matric ScholarshipMay to June
Rajiv Gandhi National FellowshipJuly to October667
National Overseas ScholarshipDecember in the academic year20

Post-Matric Scholarship (Class XII Onwards)

This scholarship provides financial help to ST students studying at post matriculation level to complete their education. This scheme is centrally funded and implemented by the State Government. These scholarship awards are fully funded by the Central Government. Whereas administered by the State Government. Students who fulfill the eligibility criteria can apply for this scholarship. Application for this scholarship is available online on the National Scholarship portal.

Eligibility

  • Candidates who belong to ST category can apply for this scholarship.
  • Candidates’ family income should not be more than 2 Lakh per annum.
  • Candidates must be pursuing post matriculation and post-secondary courses in recognized Schools/ Institutes / Universities.

Post-Matric ST Scholarship Awards

The scholarship awarded to ST students as per the courses they are enrolled into is given below:

CoursesScholarship Allowance
Day ScholarsHostellers
Group I

Degree and postgraduate courses such as M.Phil., Ph.D. in engineering, technology, medicine, planning, architecture, and so forth.
Commercial Pilot License
Postgraduate diploma courses in medicine and management
CA/ ICWA/ CS/ ICFA etc.
MPhil, Ph.D., research programs
LLM

Rs.1200Rs.500
Group II

Graduate and postgraduate courses leading to a diploma, certificate, and degree in LLB, Nursing, BFS, and so forth.
Postgraduate courses which are not covered in Group – I

Rs.820Rs.530
Group III – all other courses which are not covered in Group I and IIRs.570Rs.300
Group IV- Post-matriculation level non-degree courses like Polytechnic, ITI, and so forth.Rs.380Rs. 230

Application Process

  • Candidates can apply online for this scholarship from May to June on the official website.
  • Candidates should visit the National Scholarship portal.
  • Candidates should upload the following relevant documents along with the application:
  • Attested copies of diploma, degree certificates, and mark sheets
  • Passport size photograph
  • Caste certificate & family income declaration of the candidate.
  • Submit the filled application along with the documents to the Head of the Institution.

Pre-Matric Scholarship (Class IX and X)

Pre-Matric ST Scholarship offers financial support to the pre-matric students of Class 9th and 10th. This scholarship helps ST students in improving the post-matric level of education and excel in their careers. This also helps the students from the lower-income family and reduce dropouts at this level. The State Government will award this scholarship to the students in which they are domiciled.

Eligibility

  • Candidates must belong to the Scheduled Tribe category
  • Candidate’s family income should not be more than 2 Lakh per annum.
  • Candidates must not get any other scholarships from other Governments.
  • Candidates must be a regular and full-time student in a recognized Government School or Central/ State Board of Secondary Education.

Pre-Matric Scholarship Awards

The value of scholarship awarded to the ST students under this scheme are given below:

ScholarshipDay ScholarsHostellers
Ad-hoc grants and BooksRs.750/-Rs.1000/-
Scholarship for 10 monthsRs.150/-Rs.350/-

Allowance for Disabled Students

  • Monthly Reader Allowance – Rs.160/-
  • Monthly Transport Allowance – Rs.160/-
  • Monthly Helper Allowance – Rs.160/-
  • Monthly Escort Allowance – Rs.160/-
  • Monthly coaching Allowances to retarded students – Rs. 240/-

Application Process

  • Candidates can apply online for the pre-matric scholarship from May to June on the official website.
  • Candidates should visit the National Scholarship Portal to apply for the pre-matric scholarship.
  • Make sure to upload the following relevant documents along with the application:
  • Candidates caste certificate
  • Family income certificate and domicile certificate
  • Candidates bank account details
  • Aadhaar number and passport size photo
  • Finally, submit the filled application along with the documents to the Head of the Institution.

Selection Process: All the eligible students must belong to the ST category to avail of this scholarship.

Rajiv Gandhi National Fellowship for Higher Education

The Rajiv Gandhi National Fellowship (RGNF) scheme is funded and implemented by the Ministry of Tribal Affairs, Government of India. This fellowship offers financial support to ST students for pursuing higher education. The scheme is open to only ST students.

This fellowship encourages the ST students from the lowest literacy section of society to undertake higher education in the country. This scheme helps to students who are pursuing regular and full-time M.Phil. and Ph.D. courses. These courses include Sciences, Humanities, Social Sciences, Engineering, and Technology. There are 667 seats reserved for this scholarship.

Financial Assistance and Tenure of Fellowship for ST Students

The financial assistance and tenure of fellowship for ST students under this scheme are given below:

CoursesDuration of Fellowship in YearsAmount
Fellowship in Science, Social Science, Humanities, and Technology2 for JRF
3 for SRF
Rs. 25,000/- for the initial 2 years for JRF
Rs. 28,000/- per annum for remaining tenure
Contingency A in Social Science, and Humanities2 for JRF
3 for SRF
Rs. 10,000/- for the initial 2 years for JRF
Rs. 20,500/- for remaining tenure
Contingency B in Engineering, Science, and Technology2 for JRF
3 for SRF
Rs. 12,000/- per annum for the initial 2 years
Rs.25,000/- per annum for the remaining three years
HRA for all subjectsAs per rules of the University/College

Eligibility

  • ST category candidates must have passed the post-graduation course.
  • Candidates must be registered in regular and full-time M.Phil./ Ph.D. courses in the UGC recognized Universities/ Institutes/ Colleges.
  • The candidate’s family income should not be more than 4.5 Lakh per annum.
  • Transgender candidates are also eligible to apply for this fellowship.

Application Process

  • Candidates can apply online for RGNF from July to October on the official website of UGC.
  • Candidates should visit the website ugc.ac.in.
  • Candidates should fill the application in 2 parts. One is personal details and the second is academic qualifications and research experience.
  • Make sure to upload the following scanned copy of the documents along with the application:
  • Post Graduation certificates and mark sheets in PDF or JPG format.
  • Submit the filled application along with the documents and take a hard copy for future usage.

National Overseas Scholarship

This scholarship offers financial support to students pursuing higher studies abroad. Higher studies should be in Masters, Ph.D. or research level programs. There are 20 scholarships reserved for ST students. 30% of total scholarships are reserved for female candidates. The awardees must come to India after completing the studies.

The duration can be extended for a maximum of 2 years with permission from the Indian Government and Indian Mission abroad. There are 4 passage grants for ST students who have received a merit scholarship. These grants are for post-graduate studies, research or training abroad from a foreign government. The cost of these passages is not provided.

Course and Number of Scholarships Awarded

CoursesNumber of Fellowships
Economics/ Finance7
Applied Science3
Management7
Science3
Engineering7
Agriculture5
Humanities and Social Sciences5
Medicine5

Eligibility

  • Ph.D. students must have passed the previous year examination with 55% marks.
  • Ph.D. students should possess 5 years of research/ training/ professional experience in the concerned fields.
  • Masters students should possess 2 years of relevant research/ training/ professional experience in the concerned field.
  • Masters students must have passed the previous year examination with 55% marks.
  • Candidate’s age must be less than 35 years.
  • Candidates’ family income should be less than 6 Lakh per annum.

National Overseas Allowance and Other Benefits

The maintenance allowance and other benefits prescribed under the scheme are given below:

AllowancesOther Benefits
US – annual maintenance allowance for all level of courses is USD 15,400Visa fees, medical expenses, poll tax, incidental journey expenses, travel costs, and so forth.
UK – annual maintenance allowance for all level of courses is GBP 9,900

Application Process

  • The deadline to apply for this scholarship is generally in December of the academic year.
  • Candidates should visit the Ministry of Tribal Affairs online portal to download the application form.
  • Candidates should enter the correct and valid information in the application.
  • Make sure to attach the following relevant documents along with the application form:
  • Caste certificate and family income certificate of the candidate
  • Domicile certificate of the candidate
  • Candidates bank details
  • Candidates aadhaar number and passport size photo with signature
  • Submit the filled application along with the relevant documents to apply for this scholarship.

FAQ’s on ST Scholarship

Question 1.
Why should the ST Scholarship?

Answer:
ST Scholarship offers financial support to ST students to complete their studies at both school and college level.

Question 2.
Is ST Scholarship available for all category candidates?

Answer:
No, this scholarship is awarded only to the scheduled tribe (ST) category students.

Question 3.
What are the different types of ST Scholarships?

Answer:
The Ministry of Tribal Affairs is implementing different types of ST Scholarships for ST students in the country. These are the Pre-matric Scholarship, Post-matric Scholarship, Rajiv Gandhi National Fellowship, and Overseas Scholarship.

Question 4.
What is the deadline to submit the National Overseas Scholarship application?

Answer:
The deadline for submitting this scholarship application is the last week of December of the academic year.

Question 5.
What should be the candidate’s age limit to apply for the National Overseas Scholarship?

Answer:
Candidate’s age must be less than 35 years to apply for the National Overseas Scholarship.

Hope this article will help you to know more about ST Scholarship. For any queries related to ST Scholarship, leave it in the comment box to get in touch with us.

Scholarships for Students

The post ST Scholarship | Eligibility, Benefits, Application, and Selection Process  appeared first on Learn CBSE.

Savitribai Phule Scholarship | Eligibility, Benefits, and Application Process 

$
0
0

Savitribai Phule Scholarship: Savitribai Phule Scholarship is an initiative of the Social Justice and Special Assistance Department, Government of Maharashtra. The scheme is available to only the girl students of SC/VJNT/SBC categories. The backward class girl students of Maharashtra state can apply for this scholarship. This scholarship offers financial support to economically backward girl candidates. So that these candidates can enroll and complete their education. Maharashtra Education Department provides financial help to girls for improving education. This scholarship initiated to encourage girls for education.

Application for this Scholarship releases on the website of Aaple Sarkar DBT portal. Candidates can also apply through their respective school to avail the scholarship benefits. Candidates can fill the application and apply in the last week of December 2019. The article below provides complete information about Savitribai Phule Scholarship.

Savitribai Phule Scholarship

Savitribai Phule Scholarship helps girl students from lower-income families to continue their schooling. This scholarship is for the girls who belong to reserved categories. Also, girls under the poverty line can avail the benefits of this scholarship. This scheme empowers the girl candidates from the deprived section of the society to undertake education.

The objective of this scholarship is to provide an award in the form of financial support to SC/SBC/VJNT students. This scholarship organized by Maharashtra Government for the SC/SBC/VJNT girl students who are not able to fulfill their dreams. These students can complete their schooling under this scheme without any financial hassle.

Savitribai Phule Scholarship Overview

ParticularsDetails
Conducting BodyThe Social Justice and Special Assistance Department of the Government of Maharashtra State
Scholarship NameSavitribai Phule Scholarship
AwardsThe selected candidates get the amount of up to Rs.100 per month
Deadline of Savitribai Phule Scholarship Application SubmissionLast Week of December 2019
Educational QualificationClass 5th to 10th
Scholarship TypePre Matric Scholarship
Applicable StateMaharashtra
Websitemahadbtmahait.gov.in

Scholarships for Students

Savitribai Phule Scholarship 2019 – Eligibility Criteria

Candidates must fulfill the following eligibility criteria to apply for Savitribai Phule Scholarship

  • Only girl candidates of Maharashtra are eligible for this scholarship.
  • Candidates must belong to SC, VJNT or SBC category.
  • Candidates must be studying in 5th to 10th class in recognized Government schools.
  • Candidates must have passed the previous year’s examination.

Savitribai Phule Scholarship 2019 – Selection Process

The Selections for Savitribai Phule Scholarship are made on the basis of merit. Girl candidates will be shortlisted based on their performance and hardworking skills. The final selection depends on the decision of the Assistant Commissioner of Social Welfare. The Assistant Commissioner has the right to cancel the scholarship without any notice. The Assistant Commissioner verifies the application and shortlists the candidates for scholarship. The shortlisted girl candidates will receive the amount as per the sanctioned fund. The scholarship amount will be transferred directly into the shortlisted candidate’s bank account.

Savitribai Phule Scholarship 2019 – Benefits

The shortlisted candidates get financial help from the Govt. of Maharashtra to complete their school education. Refer the below award details of Savitribai Phule Scholarship

CategoryScholarship AmountDuration
Students studying in class 8th to 10thRs.100 per month10 months
Students studying in class 5th to 7thRs.60 per month10 months

Savitribai Phule Scholarship 2019 – Application Process

The application for this Scholarship releases on the official website of Aaple Sarkar DBT portal. It is a unique platform run by the Govt. of Maharashtra to assist its people for receiving the benefits of different schemes. But, candidates should have an Aadhar number link to this portal to apply through this portal. Candidates can fill and submit the application in the last week of December 2019. Candidates can apply for this scholarship in both online and offline mode. Candidates must read the instructions carefully before filling this scholarship application. Make sure to keep ready all the relevant documents before filing the application. Refer to the below points of online as well as the offline application for this scholarship.

Online Application Process

  • Visit the official website mahadbtmahait.gov.in.
  • Click on “Pre Matric Scholarship” to apply for Savitribai Phule Scholarship.
  • Click on “New Applicant Registration” appeared on the home page.
  • Enter the Aadhar number and authenticate Aadhaar by selecting OTP. Fill the necessary details to create user Id and password to login to this portal.
  • Log in to this portal and complete the registration process. Then, apply online for this scholarship.
  • Fill the scholarship application with accurate and necessary details in capital letters.
  • Recheck the filled application to avoid rejection by higher authorities during verification.
  • Upload scanned copies of the relevant documents to apply online for this scholarship.
  • Then, submit the filled application to be selected for Savitribai Phule Scholarship.

Offline Application Process

  • The eligible girl candidates can apply for this scholarship only through their respective school.
  • Interested candidates have to contact directly to their school Principal or Head Master.
  • After this, the HeadMaster will select the list of eligible candidates.
  • Then, submit the list to the Assistant Commissioner of Social Welfare of the respective district for sanction.

Savitribai Phule Scholarship 2019 – Checklist of Documents

Candidates must attach the relevant documents along with the application. The following are the list of relevant documents to apply for Savitribai Phule Scholarship.

  • Candidates caste certificate issued by the competent authority
  • Attested copies of mark sheets of last appeared examination
  • A photocopy of the first page of the candidate’s bank passbook
  • A photocopy of Aadhar Card
  • Xerox copy of domicile certificate
  • Candidates one recent passport size photograph
  • Age proof (class 10th mark sheets or birth certificate)
  • Residential Proof (Electricity Bill/ Voter ID/ Aadhar Card/ Ration Card/ Driving License etc.)

Savitribai Phule Scholarship 2019 – Renewal

Candidates who have already registered for Savitribai Phule Scholarship must renew it every year. Candidates application renewal depends on the performance in each year’s annual exam. Candidates must have passed the previous year’s examination to renew the scholarship. If the candidate’s performance is satisfactory then they will be considered to avail the benefits of this scholarship. Candidates can modify the existing application and submit for renewal.

Savitribai Phule Scholarship 2019 – Contact Details

In case of any queries related to this scholarship contact on the below details:

Phone022-22025251/022-22843665/220231652
AddressSocial Justice and Special Assistance Department, Annex Building, 1st Floor, Mantralaya, Hutatma Rajguru Chowk, Nariman Point, Mumbai-400032, Maharashtra
Email IDmin.socjustice@maharashtra.gov.in, mahadbt.helpdesk@maharashtra.gov.in

FAQ’s on Savitribai Phule Scholarship 2019

Question 1.
Why should apply for Savitribai Phule Scholarship?

Answer:
This scholarship offers financial support to economically backward girl candidates. So that these candidates can enroll and complete their education. This scholarship initiated to encourage girls for education. So, girls who desire to fulfill their dreams should apply for this scholarship.

Question 2.
Is Savitribai Phule Scholarship available for all category candidates?

Answer:
No, this scheme is available for the girl candidates of SC/VJNT/SBC categories.

Question 3.
Is Savitribai Phule Scholarship open to all states?

Answer:
No, Savitribai Phule Scholarship is open to only girl candidates of Maharashtra State.

Question 4.
What is the deadline to submit the Savitribai Phule Scholarship application?

Answer:
The deadline for submitting the application is the last week of December 2019.

Question 5.
How much amount will be paid to the shortlisted candidates?

Answer:

  • Candidates studying in class 5th to 7th will get the scholarship amount of Rs. 60/- per month for 10 months. However,
  • Candidates studying in class 8th to 10th will get the scholarship amount of Rs. 100/- per month for 10 months.

Hope this article will help you to know more about Savitribai Phule Scholarship. For any queries related to Savitribai Phule Scholarship, leave it in the comment box to get in touch with us.

The post Savitribai Phule Scholarship | Eligibility, Benefits, and Application Process  appeared first on Learn CBSE.

Rajiv Gandhi National Fellowship | Eligibility, Awards, Application, and Documents 

$
0
0

RGNF Scheme: Rajiv Gandhi National Fellowship (RGNF) scheme initiated by the Ministry of Social Justice and Empowerment and the Ministry of Tribal Affairs. The scheme is open to Scheduled Castes and Scheduled Tribe students. The RGNF for SC students is formulated and funded by the Ministry of Social Justice and Empowerment. Whereas the RGNF for ST students is formulated and funded by the Ministry of Tribal Affairs. This fellowship offers support to SC and ST candidates who desire to pursue higher studies such as regular and full-time M.Phil. and Ph.D. degrees. These degrees can be pursued in Science, Humanities, Social Sciences, Engineering, and Technology. Candidates who belong to SC and ST categories can only apply for this fellowship.

Application for RGNF releases on the official website of University Grants Commission (UGC). Candidates can fill the application and apply for RGNF in the last week of October 2019. The article below provides complete information about Rajiv Gandhi National Fellowship.

Rajiv Gandhi National Fellowship 2019

Rajiv Gandhi National Fellowship encourages SC and ST students from lower-income families to pursue higher education. RGNF helps the candidates from the deprived section of the society to undertake advanced studies and research. The objective of this fellowship is to provide fellowships in the form of financial support to SC/ST students. These students can complete higher education without financial hassle. The RGNF is also available for those SC/ST students who study M.Phil., Ph.D., Engineering and Technology courses every year. There are 2000 seats available for SC and 667 seats for ST candidates every year for all the subjects. Govt. of India has reserved 3% fellowships for disability candidates who belong to SC categories.

Rajiv Gandhi National Fellowship Overview

ParticularsDetails
Conducting BodyMinistry of Social Justice and Empowerment for Scheduled Caste
Ministry of Tribal Affairs for Scheduled Tribe
Fellowship NameRajiv Gandhi National Fellowship
AwardsThe selected candidates get the amount of Rs.28,000/- per month along with contingency grant, HRA, and allowances for PH and blind candidates
Last Date for Submission of RGNF ApplicationLast Week of October 2019
Websitehttps://www.ugc.ac.in/

Rajiv Gandhi National Fellowship 2019 – Eligibility Criteria

Candidates must fulfill the following eligibility criteria to apply for RGNF

  • Candidate must belong to SC and ST categories.
  • Candidate should have passed the postgraduate examination.
  • Candidates must be registered themselves for full-time M.Phil/Ph.D. degrees.
  • Candidates register as regular students in University/Institute/Colleges recognized by UGC under Section 2 (f) of the UGC Act/ICAR.
  • Candidate belongs to the transgender category are also eligible.

Rajiv Gandhi National Fellowship 2019 – Selection Process

The Selections for RGNF made on the basis of merit and as per the procedures of the Commission. The Commission scrutinizes and selects the received applications. The final selection depends on the decision of the Commission. The Commission has the right to withdraw/cancel the award without conveying any reason. After these formalities, candidates Joining Report will be duly signed by the supervisor/head of department. Then, it will be sent to the UGC through the Registrar/ Director/ Principal. The first-year admissible grant releases on receipt of the Joining Report and other required documents. Otherwise, the concerned university will be informed to release the grant.

This grant releases from the funds which have sanctioned for this purpose by the Commission. Thereon, the fellows may get the amount from the institutions/bank after getting the UGC approval/award letter. Also, other expenses from the grants paid by the Commission in conformity with the rules. The payment of the fellowships will be transferred directly into the bank account of the beneficiaries.

Rajiv Gandhi National Fellowship 2019 – Terms and Conditions

  • The duration of RGNF scheme is initially for two years.
  • The work of the Fellow will be evaluated by a Committee of three members before the completion of this duration. These three committee members are the Head of the Department, Supervisor and one external subject expert. These members are to be constituted by the concerned Department/ University/ College.
  • The tenure will be extended further based on the research work satisfaction. If the work is satisfactory, then tenure will be extended for 3 more years.
  • The tenure will be extended under the enhanced emoluments of the Rajiv Gandhi National Senior Research Fellowship (RGNSRF).
  • The recommendation of the Committee for upgrading the level of RGNSRF will be submitted to the UGC.
  • The work status and the time spent on fellowships of any agency other than the UGC will not be accepted. The fellowship information from UGC only considers improving the value of fellowship.
  • The fellowship may be withdrawn in case of the dissatisfaction of the work. Also, if the candidate fails in any of the examinations related to Ph.D.
  • In case the work for the first two years is not satisfied, an extra year will be given to a candidate for improvement.
  • In this duration, the candidate will be designated as Rajiv Gandhi National Junior Research Fellow (RGNJRF).
  • Again the work will be evaluated after completion of 2nd year of the tenure.
  • If found any improvement, then the Fellow will get two more years under the RGNSRF.
  • So, the total period of fellowship (RGNJRF and RGNSRF) is for five years, with no more provision of extension.

Rajiv Gandhi National Fellowship 2019 – Awards

RGNF aims to improve the low literacy levels of the SC/ST categories through higher education. The shortlisted candidates get financial help from the Govt. to pursue M.Phil./Ph.D. degrees under RGNF. Refer the below award details of RGNF

ParticularsDetails
Fellowship AmountRGNF JRF (Junior Research Fellow) candidates receive a fellowship amount of Rs. 25,000/- per month for the initial two years.
RGNF JRF candidates promoted to SRF (Senior Research Fellow) receive a fellowship amount of Rs.28,000/- per month for the remaining three years.
Contingency grant for Humanities and Social SciencesAn amount of Rs.10,000/- per annum for the initial two years.
An amount of Rs.20,500/- per annum for remaining tenure.
Contingency grant for Science, Engineering, and TechnologyAn amount of Rs.12,000/- per annum for the initial two years.
An amount of Rs.25,000/- per annum for the remaining three years.
Escorts/Reader Assistance to PH CandidatesAn amount of Rs.2,000/- per month
Departmental Assistance for providing infrastructure to the host institutionAn amount of Rs.3,000/- per annum for each student
House Rent Allowance (HRA)As per rules of the University/Institution/Colleges
MedicalAs per rules of the University/Institution/Colleges

Rajiv Gandhi National Fellowship 2019 – Application Process

The application for RGNF releases online on the official website of UGC. Candidates can fill and submit the application in the last week of October 2019. Candidates can apply for this scholarship through the UGC website. It is a platform run by Govt. of India for coordinating, deciding and maintaining teaching standards, examination and research in University. Candidates must read the instructions carefully before filling RGNF application. Make sure to keep ready all the relevant documents prior to fill the RGNF application form. Refer to the below points to apply for RGNF Scholarship.

  • Visit the official website of UGC, New Delhi, India.
  • Click on the “e-SARTS” link appeared on the under “Quick Links” of the home page.
  • Check the National Fellowship for Scheduled Caste Students details appeared under Fellowship section.
  • Similarly, check the National Fellowship for Higher Education of ST Students details.
  • Candidates can click on the “See Details Info” to apply for the desired fellowship.
  • After that, click on the “Apply Now” to open the application form.
  • Fill the correct and necessary information as per the RGNF instruction.
  • Recheck the filled application to avoid rejection by the RGNF committees during verification.
  • Upload the required documents along with the RGNF application.
  • Finally, submit the RGNF filled application along with the documents.

Rajiv Gandhi National Fellowship 2019 – Checklist of Documents

Candidates must upload the scanned copy of essential documents along with the application. The following are the list of essential documents to apply for RGNF.

  • Candidates caste certificate only in pdf, jpg or gif format issued by the competent authority
  • Specific disability certificate, if required
  • Post-graduation mark sheet only in pdf, jpg or gif format
  • Certificate from the Head of the Department /Institution stating that the required facilities will be provided by the Institution/College/University if the candidate gets selected for RGNF

Contact Details

In case of any queries related to RGNF send the queries to Email ID – sosa3ugc@gmail.com

FAQ’s on Rajiv Gandhi National Fellowship 2019

Question 1.
What is Rajiv Gandhi National Fellowship?

Answer:
Rajiv Gandhi National Fellowship also known as RGNF is an initiative of Government. It offers support to SC and ST candidates for pursuing full-time M.Phil. and Ph.D. degrees.

Question 2.
Is RGNF open to all category candidates?

Answer:
No, RGNF scheme is open to only Scheduled Castes and Scheduled Tribe category candidates.

Question 3.
What are the streams offered to pursue M.Phil and Ph.D. degrees?

Answer:
The degrees can be pursued in Science, Humanities, Social Sciences, Engineering and Technology streams.

Question 4.
What is the last date to submit the RGNF application?

Answer:
The last date for submitting RGNF application is the last week of October 2019.

Question 5.
How many seats are available for SC and ST candidates in RGNF?

Answer:
There are 2000 seats available for SC and 667 seats for ST candidates. However, 3% fellowships reserved for PH candidates who belong to SC categories.

Hope this article will help you to get more information about RGNF 2019. If you have queries related to RGNF, then leave it in the comment box to get in touch with us.

The post Rajiv Gandhi National Fellowship | Eligibility, Awards, Application, and Documents  appeared first on Learn CBSE.

Pragati Scholarship | Eligibility, Application, Rewards

$
0
0

AICTE Pragati Scholarships 2019-20: Pragati Scholarship is an MHRD Scheme being implemented by AICTE strived at assisting the Advancement of Girls pursuing Technical Education. Education is one of the most important means of enabling women with the knowledge, skill, and self-confidence required to join fully in the development process. This is an effort to allow young women for further education and prepare for a successful future by “Empowering Women through Technical Education”

All India Council Technical Education (AICTE) welcomes online applications for Pragati Scholarship Schemes for the academic year 2019. The scholarship will be given to only two girl children of a family for pursuing her undergraduate professional/diploma courses. In a year total of 4000 scholarship will be offered to the selected girl students  (2000 for Degree and 2000 for Diploma). The Last Date to apply for the Pragati Scholarship is tentatively on the 2nd week of October 2019.

Pragati Scholarship Eligibility Criteria

The details of the conditions required to apply for the Pragati scheme are given here. Candidates should make sure they fulfill all the criteria before applying for the scholarship.

  • Up to Two Girls per family can apply.
  • Family income should not be more than Rs. 8 Lakhs per Annum (in case of a married girl, the income of parents/in-laws, whichever is higher will be considered)
  • Students Admitted for Diploma/Undergraduate Degree Level Programmes/Courses in AICTE Approved Institutions.
  • Only for the students admitted in the first year of their Degree/Diploma for the academic year 2017-18.
  • The selection of the candidate will be made on merit on the basis of qualifying examination to peruse the respective Technical Degree/Diploma course from any of the AICTE approved institutions.

Pragati Scholarship Application Process

Check the complete step by step procedure to apply for Pragati Scholarship.

  • Go to AICTE Portal @ https://www.aicte-pragati-saksham-gov.in/ to register for Pragati Scheme.
  • Click on “Register Here” to create a new profile. Enter the details and click on “Register”.
  • A confirmation e-mail link will be sent to the registered e-mail ID. Please click on the link to confirm your registration.
  • Login again at https://www.aicte-pragati-saksham-gov.in/ with the registered e-mail Id/User name and Password to fill the online application form.
  • Enter personal details, details of family and income, details of the Institute in which Admission is taken for first-year Degree or Diploma course along with fee paid details, SSC/10th Standard and HSC/12th Standard details, details of the Aadhar seeded Savings Bank Account.
  • Attach the scan copies of all the necessary documents in jpg./pdf./png. Formats.
  • Submit the application form.
  • Take the print out for future reference.

Documents Required for Pragati Scholarship

Find here all the necessary documents required to apply for the Pragati scholarship. Students have to keep ready the scan copy of all these documents before they start filling up the application form.

  • SSC/10th Standard Mark sheet.
  • HSC/12th Standard Mark sheet.
  • Annual Family Income Certificate for the financial year 2016-17 as per Annexure-I (or) in the prescribed format issued by Tehsildar or above competent Authority.
  • Admission letter issued by the Centralized Admission Authority to first-year Degree/Diploma courses for the academic year 2017-18.
  • Tuition fee paid receipt for the academic year 2017-18.
  • Scanned Copy of Aadhar Seeded Bank Pass Book showing the name of the student, Account number, IFSC code and Photograph pasted at appropriate place duly signed by Manager with the rubber stamp of Bank affixed.
  • Certificate Issued by the Director/Principal/HOD as per Annexure-II
  • Certificate issued by Competent Authority in case of candidates applying for Saksham Scheme (Only for Physically Disabled Candidates having more than 40% disability) as per the State Government Format.
  • Attested Scanned copy of SC/ST/OBC Certificate, in case applying for SC/ST/OBC category.
  • Declaration by parents duly signed stating that the information provided by their child is correct and will refund the Scholarship amount if found false at any stage a per Annexure – III
  • Aadhar Card
  • Candidate Photograph.
  • Candidate Signature.

Pragati Scholarship Benefits and Rewards

The amount of scholarship offered by the Pragati scheme and who will get the amount, other benefits are given to the students along with scholarship reward, and other related details are provided here.

  • The AICTE Pragati Scholarship rewards a total of 4000 scholarships to girl students, in which 2000 scholarships are given to students pursuing diploma and 2000 to degree students.
  • The selected scholars will receive a tuition fee of INR 30,000 or the actual tuition fee amount, whichever is less.
  • The selected scholars also become eligible to get INR 2000 per month for 10 months as incidental charges each year.
  • In case of Tuition fee waiver/reimbursement, Students are eligible to get an amount of Rs. 30,000/- for the purchase of Books/Equipment/Softwares/ Laptop/Desktop/Vehicle/Fee paid towards competitive examination application forms/exam.
  • Fees paid for competitive exams, application forms/exam fees for all examinations related to higher education/employment.

Pragati Scholarship Reservation

  • A reservation of 15% for SC candidates, 7.5% for ST candidates and 27% for OBC candidate/applicant is provided by AICTE.
  • Candidates falling under these categories have to provide their caste certificate at the time of document submission while applying for the scheme.

Important Instructions for Pragati Scholarship

  • Only the scholars who have taken admission in the first year of their degree/diploma program in the current academic year can apply for this scholarship.
  • The scanned copies of the applicant’s passport size photo and signature must be uploaded in .jpg/.pdf/.png format while filling the application form. The file size of the photo should not be more than 200 kb and signature should not be more than 50kb.
  • Students who got admission through management quota are not eligible for the AICTE Pragati Scholarship.
  • Candidates must have a general savings account of their own in a bank. They cannot use the FRILL/Minor/Joint account.
  • Direct Benefit Transfer (DBT) is used to directly transfer the scholarship amount into the selected candidate’s bank account.
  • The scholarships for degree and diploma are transferable in event of non-availability of eligible applicants in any of the degree/diploma level programs.

FAQ’s on Pragati Scholarship

Question 1.
What is Pragati Scholarship?

Answer:
Pragati Scholarship is a government scholarship scheme implemented by the All India Council for Technical Education (AICTE). Under this scholarship scheme, a total of 4000 scholarships are disbursed every year among meritorious girl students for pursuing technical education.  Tuition Fee of Rs. 30,000/- or at actual, whichever is less and Rs.2000/- per month for 10 months as incidentals charges each year.

Question 2.
What is Saksham Scholarship?

Answer:
Saksham is an MHRD Scheme being implemented by AICTE aimed at providing encouragement and support to specially-abled children to pursue Technical Education. This is an attempt to give every young student, who is otherwise specially-abled, the opportunity too studies further and prepare for a successful future. Tuition Fee of Rs. 30,000/- or at actual, whichever is less and Rs.2000/- per month for 10 months as incidentals charges each year.

Question 3.
Is AADHAAR Card Mandatory for PRAGATI (or) SAKSHAM Scholarship?

Answer:
Yes. Aadhaar card and Aadhaar seeded bank account in the name of the candidate are mandatory for submitting an online application under Pragati/Saksham schemes.

Question 4.
How the scholarship will be disbursed through College/Institute?

Answer:
The scholarship will be disbursed only through Direct Benefit Transfer (DBT) directly in the bank account of the student. The student must have a General Saving Account in the bank. (FRILL/Minor/Joint account will not be accepted).

The post Pragati Scholarship | Eligibility, Application, Rewards appeared first on Learn CBSE.


National Fellowship for OBC | Application, Award, Key Date, Eligibility

$
0
0

National Fellowship for OBC 2019-20: OBC’s National Fellowship is a UGC grant system financed under the Government of India by the Ministry of Social Justice & Empowerment. This UGC grant is for the advantage of Other Backward Class (OBC) applicants. The unemployed OBC applicants are significantly endorsed under this National Fellowship for OBC students to conduct sophisticated studies and study aspiring to M.Phil. And Ph.D. in Science, Humanities, Social Science and Engineering & Technology. This UGC scholarship is given annually to a maximum of 300 applicants.

National Fellowship for OBC – Highlights

ParticularsDetails
Award detailsFellowship amount up to Rs. 28,000 per month along with contingency grant and assistance from escorts/reader for candidates with different skills
Application period*October-December
EligibilityCandidates must be from the OBC Category. Must pursue M.Phil./Ph. D. Degree No more than Rs. 6 Lakh Annual revenue
Application processThrough the official website of UGC

National Fellowship for OBC – Awards

This UGC fellowship provides financial support to a total of 300 candidates selected according to the eligibility criteria. Under OBC’s National Fellowship, the fellowship’s duration is 5 years, divided into the Junior Research Fellowship (JRF) and Senior Research Fellowship (SRF). List of selected candidates for national fellowship for OBC will be published at the official website.

JRF’s work is evaluated by a three-member committee comprising Department Head, Supervisor, and an external subject expert, and if the work is found to be satisfactory, the tenure of the candidates is further extended for a period of 3 years under the SRF’s enhanced rewards. National Fellowship for OBC Amount is Rs.25000 for first two years.

In addition, the amount of the fellowship is transferred directly to the awarded account via e-payment under DBT. In addition, the amount of the fellowship is transferred directly to the awarded account via e-payment under DBT. Find the quantity of the OBC National Fellowship as given to the applicants chosen below.

National Fellowship for OBC – Award details

ParticularsDetails
Fellowship amountA monthly grant sum of Rs.25,000 for the first two years to JRF A grant sum of Rs.28,000 per month for the following commitment to SRF
Contingency grant for Humanities & Social Sciences studentsAn annual quantity of Rs. 10,000 for the original two years Rs. 20,500 per year for the following three years
Contingency grant for Science and Engineering & Technology studentsRs. 12,000 per year for the original two years Rs. 25,000 per year for the following three years
Escorts/Reader assistance to physically & visually challenged candidatesRs. 2000 per month
HRAAs per the norms of the Government of India

National Fellowship for OBC – Application Period

The National OBC Fellowship’s application period varies from year to year. Its application period, however, generally ranges from October to December. The request launched in the first week of October for the educational era of 2018-19, with the date dropping in the last week of December. Furthermore, applicants wanting to take advantage of this UGC grant must bear in mind that the timeline listed here is timely and may alter according to the membership provider’s path.

National Fellowship for OBC – Eligibility Criteria

OBC’s National Fellowship enables aspirants to study M.Phil full-time. Or doctoral degree program without UGC NET JRF or UGC-CSIR NET. Under this UGC scholarship system, 300 applicants are chosen annually for economic advantages. The information of qualifying circumstances to be fulfilled by candidates in an attempt to benefit from the advantages of this grant is given below.

  • To qualify for this UGC grant, candidates must be homeless and adhere to the OBC group.
  • The postgraduate examination must have been taken by the candidates.
  • The candidates ‘ annual household earnings from all accounts must not exceed INR 6 Lakh.
  • Applicants must be enrolled for periodic M.Phil./Ph.D. full-time. Science, Humanities, Social Science or Engineering & Technology program from universities/colleges/institutions recognized by UGC, State/Central Government or nationally significant institutions.
  • As part of this UGC scholarship, transgender applicants are also permitted to register. The reservation is made in accordance with the Indian government’s standards.

National Fellowship for OBC – Application Process

UGC enables candidates to register the internet through its formal portal to render the registration method simpler and more effective for candidates. Follow the step-by-step manual below to complete your OBC National Fellowship request type.

  • First, applicants must check and surf down the formal UGC portal to press the’ vision all’ button on the’ Scholarship / Fellowships ‘ section. A range of UGC’s scholarships and grants is shown.
  • Go to the’ Fellowship’ section and press the’ Apply Now’ key provided under the OBC Candidate National Fellowship.
  • The candidates must closely study the guidance and tap on ‘Continue to register.’
  • In the registration form, complete the necessary information. Upon completion of the enrollment method, a registered identification is produced. The registration ID helps download the finished request form’s PDF file once it has been presented.
  • The candidates must provide personal details, email data, educational details, details of the institute, study job descriptions, etc. while carrying out the application. Other information such as the information of the Supervisor, the details of the scholarship and the current job are also needed.
  • Lastly, the applicants must check the request type filled in. Go to the statement chapter once you are fulfilled. Carefully read it and tap on ‘I agree’ to submit the application.

National Fellowship for OBC – Key Documents

While registering for this UGC grant, there are some significant papers that the candidates must maintain prepared with them. The range of important papers that applicants should have with them when carrying out the National Fellowship request document for OBC is highlighted below.

  • Applicant’s caste license Income register of the applicant’s relatives issued by the competent authority
  • Applicant’s license picture (color) and registration
  • Mark of the master’s degree diploma given by Head of Department/Institution indicating that the required equipment will be supplied by the College/University/Institution in the event that applicants are chosen for this grant.

National Fellowship for OBC – Selection Procedure

The candidates receiving this UGC fellowship are selected based on merit and according to the UGC procedure. However, 3% of the complete spaces are allocated for people with disabilities (PwD) in the OBC class.

National Fellowship for OBC – Conditions

There are a couple of terms and conditions that applicants need to search for if they want to take advantage of this UGC scholarship. See below the table of National Fellowship terms and conditions for OBC applicants.

  • Under this UGC scholarship, the institution’s applicants can be supplied with appropriate single-seat hostel facilities.
  • OBC’s National Fellowship award is liable in addition to public holidays for a maximum of 30 days of leave per year. They also qualify for maternity/paternity leave in accordance with the Indian government’s standards.
  • Within 3 months of awarding the grant, the applicants must attend the class.
  • A copy of their Award Letter and Joining Report together with a photo, address and contact number in the prescribed format must also be submitted to the bank’s designated branch.
  • The applicants chosen must also send a’ Continuation Certificate’ to the assigned office every three months.
  • The applicants must also send their progress report to the office allocated to the UGC after completing one year of the scholarship.

FAQ’s on National Fellowship for OBC

Question 1.
In which field is the national fellowship being offered to students?

Answer:
This fellowship is for the students aspiring to M.Phil. And Ph.D. in Science, Humanities, Social Science and Engineering & Technology.

Question 2.
In total, how many scholarships are offered under national fellowship?

Answer:
This UGC scholarship is given annually to a maximum of 300 applicants.

Question 3.
How is the amount in this fellowship transferred to students?

Answer:
The amount of the fellowship is transferred directly to the awarded account via e-payment under DBT.

Scholarships for Students

The post National Fellowship for OBC | Application, Award, Key Date, Eligibility appeared first on Learn CBSE.

Prime Minister’s Special Scholarship Scheme (PMSSS) | Dates, Eligibility, Application Procedure

$
0
0

PMSSS 2019: PMSSS (Prime Minister’s Special Scholarship Scheme) also is known as J&K Scholarship, meant for students of Jammu and Kashmir who wants to acquire higher education in different fields such as, Engineering and Technology, Nursing, Pharmacy, Architecture, Hotel Management and Catering Technology, Crafts and Design, Arts, Medical and General education. The scheme is available for Diploma passed students also who want to be graduate in Engineering and Technology as Lateral Entry (2nd year BE/B.Tech).

J&K or PMSSS is an elite scholarship opportunity provided to students who are a domicile of Jammu and Kashmir. Commenced in the year 2011, the scheme focusses on improving job opportunities for the youth of the state by giving them financial support, so that they can proceed with higher studies at an undergraduate level outside J&K. 5000 fresh scholarships are provided by the government every year, for studying in the best academic institutions outside J&K. Conducted by the All India Council for Technical Education, the AICTE PMSSS is administered within the state by the J&K government.

Get complete information related to PMSSS eligibility conditions to qualify for different scholarship schemes, application procedure, what is the last date to apply. Also, know the method through which scholarship will be paid to students and the selection process.  In this article, all the important detail related to J and K scholarship such as the dates, eligibility criteria, application procedure, award details, selection procedure, etc. are given here.

PMSSS Important Dates

Before reviewing information about the PMSSS, you should know the important dates related to it. The scholarship supports applications from eligible students every year within a mentioned time duration. Here, all the vital dates related to the JK scholarship for the academic session 2019-20 is given.

ParticularsDates
Online registration starting date10th April 2019
Last date to register online for class 12 passed/ appearing students31st May 2019
The last date to register online for diploma passed students (lateral entry in engineering stream only)18th June 2019
Document verification of registered candidates at facilitation centers (online)16th April 2019 to 31st May 2019
16th April 2019 to 14th May 2019 (for lateral entry students)
Choice filling option for students whose documents have been verified22nd April 2019 to 31st May 2019
20th April 2019 to 14th May 2019 (for lateral entry students)
Release of provisional merit list on the portal11th June 2019
Up to 10th June 2019 (for lateral entry students)
Document re-verification at facilitation centers (Offline)11th June 2019 to 13th June 2019
Release of final merit list on the portal (Online)18th June 2019
14th June 2019 (for lateral entry students)
The first round of counseling (provisional seat allotment) OnlineYet to be announced
The second round of counseling (provisional seat allotment)_OnlineYet to be announced

Scholarships for Students

PMSSS Eligibility Criteria

To apply for Prime Minister’s Special Scholarship Scheme, students need to meet certain requirements which mainly depend on their citizenship, academic qualification and total income of the family. Know about details of eligibility criteria, necessarily required for this scholarship here.

  • Candidates should be a domicile of J&K State
  • Candidates should have Passed 10+2 Exam from JKBOSE or CBSE affiliated schools located in J&K
  • Candidates should have Passed 10+3 Diploma from J&K State Polytechnic to get admission directly in the second year in the stated list of Colleges against the vacant seats of the previous year
  • Overall Family income of the candidates should not exceed Rs. 8.00 Lakh per annum

Candidates who are not eligible

  • Seeking their studies within open universities.
  • Already undergoing financial support under this plan or any other scholarship scheme.
  • Getting admission within the management quota.
  • Persevering studies at the postgraduation level.
  • Procured admission to a college other than the designated colleges.
  • Getting admission within NGOs/Agents.
  • Annual family earnings are greater than INR 8 Lakh per annum.

PMSSS Application Procedure

Every year, AICTE welcomes on its official website online applications filled by eligible candidates. If applicants fulfill all the criteria needed for PMSSS, they can apply by following the below steps:

  • Go the AICTE PMSSS official website, www.aicte-jk-scholarship-gov.in
  • To apply for PMSSS, click on the registration button.
  • Fill in all the mandatory details.
  • Submit the registration form.
  • Once registration is done, candidates will receive an SMS/email on their registered mobile number/email for verification.
  • Click the verification link and complete the registration process.
  • After the successful registration, candidates have to log in via the AICTE PMSSS website.
  • Keep the scanned copies of all the supporting documents in jpg/png format with the file size not more than 2 MB.
  • Once logged in, read the brochure carefully, tick mark the declaration and click on the “Proceed Further” button.
  • Fill the required details in the application form such as personal information, address, family income, educational details, etc.
  • Upload all the scanned copies of supporting documents and preview the application form.
  • Provide the details of preferences and grievance centre details.
  • Now submit the application by clicking on “Submit Application”.
  • After submitting the application, applicants should take the print out of the application form and visit the facilitation center along with all supporting documents in original for verification.
  • Present the photocopies of all the certificates to the facilitation center at the time of verification.
  • Once the verification is done, the applicants have to collect a copy of their document verification report.

Documents Required for PMSSS

  • Passport-sized photograph (preferred size – 200 X 230 pixels)
  • Scanned signature (preferred size – 140 X 60 pixels)
  • SSC/Class 10th mark sheet
  • Domicile/Day Scholar certificate issued by the government of J&K
  • Aadhaar card (if available)
  • Family income certificate issued by the Tehsildar or equivalent
  • Category/Caste certificate issued by the competent authority of state government (if applicable)
  • Certificate of disability (for Persons with Disabilities – PWD)

Note: Aadhar Card is essential for smooth DBT (Direct Benefit Transfer) through Public Financial Management System (PFMS).

PMSSS – List of Facilitation Center for Documents Verification

Candidates are required to get their uploaded documents verified with the originals at the nearest Facilitation Centres set-up by the Government of J&K and once it is verified and uploaded with the stamp on Facilitation Centres portals, the choice filling window will be activated for only. The choice filling window will be blocked on the last date, after that, no change will be allowed in order of preference. After document verification from the facilitation center, the candidate has to collect a copy of the document verification report without fail.

Facilitation Centers for J&K PMSSS 2019-20 Jammu Division

  • Government G.M. Science College, Jammu – 01912578189, principalggm@gmail.com
  • Government College for Women Gandhi Nagar, Jammu – 01912435158, principal@gcwgandhinagar.com
  • Government Degree College, Boys (Kathua) – 01922234315
  • Government Degree College, Bani – 01921268025, gdcbani08@gmail.com
  • Government Degree College, Boys Udhampur – 01992270239, 01992270289, principal@gdcudhampur.in
  • Government Degree College, Rajouri – 01962262510, gdcrajouri@gmail.com
  • Government Degree College, Poonch – 01965220231, degreecollegepoonch@gmail.com
  • Government Degree College GDC, Doda – 01996233552, principalgdcdo@yahoo.in
  • Government Degree College, Reasi – 019912455905, principalgdcreasi20@gmail.com
  • Government Degree College, Ramban – 01998266779, principalgdcramban@gmail.com
  • Government Degree College, Banihal – 01998255906, gdc.banihal@yahoo.com
  • Government Degree College, Kishtwar – 01995259513, gdckishtwar@gmail.com
  • Government Degree College, Bhaderwah – 01997244155, Principalmarwah@gmail.com
  • Government Degree College, Marwah – NA, gdcrspura@gmail.com
  • Government Degree College, R.S.Pura – 01923252958
  • Government Degree College, Samba – 01923241044, gdcsamba90@gmail.com
  • Government Degree College, Mendhar – 01965226793, principalgdcmendhar@gmail.com

Facilitation Centers for J&K PMSSS 2019-20 Kashmir Division

  • Sri Pratap College, Srinagar – 09142476828, 09142476804, spcsgr1905@gmail.com
  • Government Degree College, Anantnag – 01932222308, principal@gdcboysang.ac.in
  • Government College (Women), M.A. Road, Srinagar – 09142478259, gcwmaroad@yahoo.in
  • Government Degree College, Bijbehara – 01932233263, principalbijbehara@gmail.com
  • Government Degree College (Boys), Sopore – 0195422262, soporecollege@gmail.com
  • Government Degree College (Boys), Baramulla – 01952234214, principal@baramullacollege.com
  • Government Degree College, Pattan – 01954231244, gdcpattan@yahoo.com
  • Government Degree College (Boys), Kupwara – 01955252155, kcollege786@gmail.com
  • Government Degree College, Kulgam – 01931260177
  • Government Degree College (SAM), Budgam – 01951256444, principalsamdc5@gmail.com
  • Government Boys College, Pulwama – 01933241250, gdcpulwama@gmail.com
  • Government Degree College, Shopian – 01933260204, spncollege@hotmail.com
  • Government Degree College, Tral – 01933251278, gdctral@rediffmail.com
  • Government Degree College, Bandipora – 01957226033, degreecollegebpr@gmail.com
  • Government Degree College, Dooru – 01932230157, principal.gdcdooru@gmail.com
  • Government Degree College Ganderbal – 01942416854, info@gdcganderbal.org
  • Government Degree College, Chrari-sharief – 0195125340, gdc.chrarisharief@gmail.com

PMSSS Selection Process

Prime Ministers Special Scholarship Scheme conducted by AICTE is totally a merit-based scholarship that evaluates the marks obtained by students in class 12th for the selection. Apart from this, there are many other conditions on the basis of which choices are made for different streams. These conditions include

  • Only those candidates will be selected who have secured admission to a degree course after clearing an entrance examination administered by either the central government or state government.
  • To get admission to MBBS or BDS courses, the students have to qualify the NEET (CBSE) examination.
  • To get admission to the Architecture course, the students need to qualify for the NATA examination conducted by the Council of Architecture.
  • Caste and category reservations are also available where 8% of the total seats in each stream are held for SC applicants, 11% reservation is for ST applicants, 25% reservation is for SEBC applicants and remaining seats are for General category students.

PMSSS Statistics

Let us know the PMSSS scholarship got by students in the previous year’s data as per the Jammu and Kashmir scheme. With the help of these scholarships, J and K government is able to help many students for higher studies. The table given here presents the statistics for the past 6 years.

Academic SessionNumber of Students AdmittedNumber of Students Reported
2017-1834002706 + 516 (Diploma Lateral Entry)
2016-1738182272
2015-1641021410
2014-1529571726
2013-1445853981
2012-1337753675

General Instructions To Applicants of PFMSSS

  • Candidates should guarantee the uploading of joining report on the AICTE portal, on or before the last date of joining. It will be later verified by the Institute, after which the DBT application will be opened by the eligible candidates.
  • Candidates who fail to upload their joining report on the portal of AICTE on or before the last date of joining, their seats will be canceled automatically under the scheme and will be given to the students in the next/subsequent rounds of counseling (if any).
  • Candidates are required to report for admission at designated University / Institution on the date specified as per the counseling dates along with the original documents/testimonials required at the time of admission.
  • Nodal Officer may direct the Candidates during his/her stay in the college campus.
  • Candidates are suggested to visit the AICTE website for all PMSSS updates.
  • No new registrations shall be provided in next/subsequent rounds of counseling
  • Candidates must bring the original and one set of self-attested photocopies of the documents at the time of reporting for final admission in allotted University / Institute.
  • Candidates are advised to refer the fee structure and other details of the Institute before choosing the seat.

FAQ’s on PFMSSS

Question 1.
Who is eligible for PMSSS?

Answer:

  • Must be the State subject of J&K.
  • Must have passed 10+2 examination from J&K Board or CBSE school located in J&K.
  • Must have passed 10+3 Diploma from J&K State Polytechnics eligible for admission in the direct second year in the prescribed list of colleges on the vacant seats of Engg category for batch 2018-19.
  • The family income from all sources for the Financial Year 2018- 19 should not be more than Rs. 8.0 Lakh per annum.
  • Only those willing to study outside the State of J&K.

Question 2.
How to apply for PMSSS?

Answer:
Log on to AICTE Website and follow the steps to apply for “Online Registration for scholarship” and follow the instructions on how to fill the online application.

  • Open AICTE Website via the link: https://www.aicte-india.org/
  • Go to the PMSSS tab on the home page.
  • Further, click on the tab of A/Y 2019-20 and register yourself by following the instructions.

Question 3.
What is the PMSSS amount for various streams?

Answer:

StreamAcademic Fee and No. of Scholarships
General DegreeUp to Rs.30,000 – 2070 Scholarships
Engineering Degree/ProfessionalUp to Rs.1.25 Lakh – 2830 Scholarships
Medical/BDS or equivalent Medical StreamsUp to Rs.3.00 Lakh – 100 Scholarships
Total No. of Scholarships5000

Maintenance charges limits – Uniformly 1 Lakh For all

Question 4.
What documents are required at the facilitation center for verification?

Answer:
Original Documents to be produced at the time of Verification at Facilitation Center are as

  • Xth Standard Mark sheet
  • XIIth Standard Mark sheet(if any)
  • Diploma Certificate (for lateral entry only)
  • Domicile Certificate  Aadhaar Card (if any)
  • Family Income Certificate (Issued by Tehsildar or equivalent)
  • Valid Caste certificate for SC/ST/SEBC for claiming the reservation as per State Govt. Policy
  • Valid disability Certificate (if any)

The post Prime Minister’s Special Scholarship Scheme (PMSSS) | Dates, Eligibility, Application Procedure appeared first on Learn CBSE.

DTE Scholarship 2019 Maharashtra | Dates, Eligibility, Awards, Application Process, Documents 

$
0
0

DTE Maharashtra Scholarship 2019: Directorate of Technical Education (DTE), Maharashtra released the notification for DTE Scholarship 2019. DTE Scholarship scheme has initiated for the economically weak, and state minority communities students. DTE, Maharashtra offers Scholarship to the candidates studying in higher education or Post HSC courses. This scholarship provides financial help to economically weak and minority communities students for pursuing professional and technical courses. Candidates who are domiciled in Maharashtra State can only apply for this scholarship scheme.

Application for DTE scholarship will be available on the official website of DTE, Maharashtra. Candidates can fill the application form and apply for this scholarship on or before 28th February 2020. The article below provides the information that is the date eligibility, awards, and so forth about DTE Scholarship 2019.

DTE Scholarship 2019

DTE Scholarship 2019 is one of the initiatives of DTE, Maharashtra. It encourages lower-income students for pursuing professional and technical courses. This scholarship helps economically backward students to complete higher education without financial hassle. This scholarship was initiated with the help of the mahaDBT mahait team by the Minority Department of DTE, Maharashtra. This scholarship is also available for different religion students. Those who study diploma, degree and engineering courses every year.

Different religion students including Muslim, Christians, Buddhists, and so forth avail this scholarship. DTE, Maharashtra is an exceptionally flexible and universally competitive higher education institute. It is responsive to the institutional, and socio developmental needs of the Maharashtra people. It is working towards economic growth and technical manpower development to come across the needs of the industry. DTE Scholarship mainly consists of the following two scholarships.

Detailed List of DTE Scholarship in Maharashtra

Scholarship nameProviderApplication Timeline
Rajarshi Chhatrapati Shahu Maharaj Shikshan Shulkh Shishyavrutti YojnaDirectorate of Technical Education (DTE), Maharashtra StateSeptember – January
Dr. Panjabrao Deshmukh Vasatigruh Nirvah Bhatta YojnaDirectorate of Technical Education (DTE), Maharashtra StateSeptember – January

Scholarships for Students

DTE Scholarship Overview

ParticularsDetails
Conducting BodyDirectorate of Technical Education (DTE), Maharashtra
Scholarship NameDTE Scholarship
Applicable StateMaharashtra
Websitewww.dtemaharashtra.gov.in

DTE Scholarship 2019 – Important Dates

EventsImportant Dates
Start of DTE Scholarship ApplicationSeptember 4, 2019
Deadline to Submit DTE Scholarship ApplicationFebruary 28, 2020

DTE Scholarship 2019 – Eligibility Criteria

Candidates must fulfill the following eligibility criteria to apply for DTE Scholarship:

  • Candidate must be an Indian national.
  • Candidate should be the domicile or permanent resident of Maharashtra state.
  • Candidates from outside the state are not eligible.
  • Candidate should be “Bonafide Student of Institute”. Also, admitted for the professional and technical courses.
  • Candidates from the deemed university are not eligible for this scholarship.
  • Candidates must be admitted through the Centralized Admission Process (CAP).
  • Candidates cannot avail any other scholarship while applying for this scholarship.
  • Only 2 children from a family are allowed for the benefit of the scheme for the current academic year.
  • Candidates annual family income should not be more than 8 Lakhs.
  • The minimum 50% attendance in the previous semester is required for this scholarship.
  • Candidates should not have a gap of two or more years during the course duration.

DTE Scholarship 2019 – Selection Process

The Selections for DTE scholarship are made on the basis of Merit. The final selection considers academic merit prescribed for each selected courses. The financial needs of the candidates also take into account for the selection. State Govt. is responsible for the selection of candidates after verifying the application. Candidates are required to link their Aadhar number to the Bank account to avail the benefits of this scholarship. The scholarship amount will be transferred directly to the selected candidate’s bank account. The AapleSarkar DBT portal will be used to transfer the scholarship amount.

DTE Scholarship 2019 – Awards

There are variable awards agreed to the permanent resident students of Maharashtra under DTE scholarship. These awards can be accessed through the mahaDBT portal. Refer to the below award details that a candidate can avail under these 2 DTE scholarships.

Rajarshi Chhatrapati Shahu Maharaj Shikshan Shulkh Shishyavrutti Yojna

  • Candidates of this scholarship will receive 50% of the tuition and exam fees.

Dr. Panjabrao Deshmukh Vasatigruh Nirvah Bhatta Yojna

  • The child of registered labor/Alpabhudharak (marginal landholder) will be given the following awards:
  • Candidates who belong to the institutes in MMRDA/ PMRDA/ Aurangabad City/ Nagpur City will be awarded Rs. 30,000/- for 10 months.
  • The scholarship amount of Rs. 20,000/- is awarded to candidates in other areas for 10 months.
  • Candidates whose family income is up to 8 Lakhs per annum will be given the following awards:
  • Candidates who belong to the institutes in MMRDA/ PMRDA/ Aurangabad City/ Nagpur City will be awarded Rs. 10,000/- for 10 months.
  • The scholarship amount of Rs. 8,000/- is awarded to candidates in other areas for 10 months.

DTE Scholarship 2019 – Application Process

The application for DTE Scholarship will be available online on September 4, 2019. Candidates can fill and submit the application on or before 28th February 2020. Candidates can apply for this scholarship through Aaple Sarkar DBT portal. It is a unique platform run by the Govt. of Maharashtra to help its citizens for availing benefits of different schemes. From the academic session 2018-19, it is mandatory to link an Aadhaar number. So, candidates should have an Aadhar number to apply for any scholarship through the portal. Candidates must read the instructions carefully and keep ready all the documents prior to fill-up the form. Refer to the below points to apply for DTE Scholarship 2019.

  • Visit the official website www.dtemaharashtra.gov.in.
  • Click on “Click here for Online Application System” available under Scholarships section.
  • It will go to the homepage of the Aaple Sarkar DBT portal. Click on the Directorate of Technical Education (DTE) link to see the scholarships. Click on the scholarship to register yourself.
  • Click on the “New Applicant Registration” link to register for the DTE scholarship. Then, enter the information of Aadhar number and proceed.
  • Candidates have to authenticate their Aadhar number in 2 ways. Either by choosing OTP or Biometric authentication.
  • If the candidate’s mobile number is registered with Aadhar then they can select OTP authentication. Otherwise, they have to select Biometric authentication.
  • Fill all the necessary information and create a user Id and password to access the portal.
  • Login to mahaDBT website by clicking on “Applicant Login”.
  • Fill all the necessary details of the profile correctly.
  • Recheck the filled application to avoid rejection by the authorities during verification.
  • Upload the required documents along with the application.
  • Finally, submit the filled application along with the documents.

DTE Scholarship 2019 – Checklist of Documents

The below-mentioned documents must be uploaded along with the application to apply for DTE Scholarship.

  • Candidates need to upload the mark sheets of class 10th and onwards.
  • Candidates need to provide domicile certificate of Maharashtra State.
  • Candidates should provide caste certificate
  • Annual family income certificate and affidavit of belonging to the minority community
  • Candidates need to provide School leaving / transfer certificate
  • Candidates have to provide an undertaking “In the current year not more than 2 beneficiaries from family”
  • Candidates need to upload a CAP related document
  • Candidates also need to upload a proof of Biometric attendance through the interface UIDAI
  • Residential proof (PAN card, voter Id, aadhar card, and so forth.)
  • Register labor certificate or Alpabhudarak (marginal landholder) certificate. For those who are applying for Dr. Panjabrao Deshmukh Vasatigruh Nirvah Bhatta Yojna scholarship.
  • Candidates who want to opt for hostel need to provide hosteller documents.
  • In the case of the private hostels or paying guest, agreement with the owner should be uploaded

DTE Scholarship 2019 – Contact Details

In case of any queries related to DTE Scholarship or any assistance contact on the below-mentioned details.

Phone Number022-22641150
AddressDirectorate of Technical Education (DTE), Mahapalika Marg, Mumbai 3, Opp.Metro Cinema, Post Box No. 1967, Mumbai – 400001, Maharashtra State

FAQ’s on DTE Scholarship 2019

Question 1.
What is the use of Aaple Sarkar DBT in  DTE Scholarship?

Answer:
Govt. of Maharashtra has launched Aaple Sarkar DBT portal. This portal uses to transfer directly the benefits and subsidies of the schemes into the bank account of the beneficiary. These schemes include pension, disaster, e-scholarships, and so forth.

Question 2.
Can the applicant edit the application form after submission?

Answer:
Yes, if the institute sends back the application to the applicant for modification then the applicant can make the necessary changes.

Question 3.
How the Aadhar based DBT is helpful for the beneficiaries?

Answer:
The Aadhar based DBT confirms that nobody can claim a share of the benefits by imitating the candidate.

Question 4.
What are the DTE Scholarship application starting and closing date?

Answer:
The DTE Scholarship application starting date is 4th September 2019. Whereas, the application closing date is 28th February 2020.

Question 5.
How is the DTE Scholarship helpful to Maharashtra state students?

Answer:
DTE Scholarship offers financial help to economically weak and minority communities students for pursuing professional and technical courses.

Hope this article will help you to get more information about DTE Scholarship 2019. If you have queries related to DTE Scholarship, then leave it in the comment box to get in touch with us.

The post DTE Scholarship 2019 Maharashtra | Dates, Eligibility, Awards, Application Process, Documents  appeared first on Learn CBSE.

K.C. Mahindra Scholarship 2019-20 | Dates, Eligibility, Awards, and Application Process 

$
0
0

K.C. Mahindra Scholarship 2019: KC Mahindra Scholarship is an initiative of the KC Mahindra Education Trust, also known as KCMET. It offers scholarships and grants to the most deserving and needy students. Some of these instituted in the mid-1950s and others founded recently. It has an effort to bring social and economic development through literacy and higher education in the country. 45% of this scholarship awardees secured admissions in the top-ranked universities worldwide. Universities such as Harvard, Oxford, Stanford, Pennsylvania, Cambridge, and so forth.

KCMET founded by the late K.C. Mahindra in 1953. It has an objective to change the lives of people in India through education by providing financial support and recognition to them. KCMET has implemented various initiatives to make a difference in over 40,000 lives of the disadvantaged students. The article below provides more information about the KC Mahindra Scholarship 2019.

K.C. Mahindra Scholarship 2019

K.C. Mahindra Scholarship grants interest-free loans or scholarships exclusively to needy students. Since its inception in 1956, KCMET mainly working to provide an opportunity for bright students. Students can pursue higher education in the selected universities abroad without financial burden. As a part of Corporate Social Responsibility (CSR), the Mahindra Group has also involved in different fields. These fields are utility vehicles, financial services, and information technology. KCMET has provided more than $65 million for grants, scholarships, and loans. These funds are derived from the KCMET investment portfolio and CSR grants. KCMET consists of the following scholarships to encourage and motivate students.

K.C. Mahindra Scholarship for Post Graduation study Abroad

Since 1956, KCMET has granted interest-free loan scholarships to deserving and needy students. These students can pursue post-graduate in various fields in Abroad. In the academic year 2018-19, KC Mahindra Scholarship disbursed grants around Rs. 2.92 Crores. Students from top institutions who placed in reputed universities were using this opportunity.

Rewards

The study in Abroad has a maximum of 2 years duration. The scholars can extend the studies more than 2 years. But, they have to get specific approval for an extension from the trust. The shortlisted scholars have to repay the scholarship amount to KCMET. It should be repaid within 24 months of completing their course. Otherwise, 48 months after availing the scholarship grant. It is the discretion of the trust to decide the expiry period. 62 scholarships are offered to deserving students. KCMET offers the following scholarships every year.

  • The top 3 KC Mahindra fellows awarded a maximum of Rs. 8 Lakh per scholar
  • The remaining successful scholars awarded a maximum of Rs. 4 Lakh per scholar

Eligibility Criteria

Candidates must fulfill the following eligibility criteria to study in Abroad.

  • Candidates must possess a first-class degree or diploma of equivalent standard. Candidates must pursue their education from a recognized University.
  • Candidates must be Indian nationals.
  • Candidates who have secured admission or applied for admission in reputed foreign universities. Admission for courses starting from August 2019 but not after February 2020.
  • Candidates going abroad for conferences, seminars, and UG studies are not eligible.

Mahindra Talent Search Scholarship

Mahindra Talent Search Scholarship was initiated in 1983. This scholarship aims to enthuse and award excellence in academics. It has been continuing in 37 educational institutions across the country.

Rewards

  • Candidates receive this Scholarship more than once will be awarded the “Honour Scholarship”.
  • Candidates can avail a cash prize of Rs. 5000/- and a citation from KCMET in Honour Scholarship.

Eligibility

  • Candidates who have secured the highest aggregate marks based on the year-end examination.

KC Mahindra United World College Scholarship

This scholarship offers to students for studying at the United World Colleges.

Rewards

This scholarship providing the below benefits at the Mahindra United World College

  • Experience of international education
  • Shared learning
  • Community service

Eligibility

  • The age of the deserving candidates should be between 16 to 18 years.

KC Mahindra All India Talent Scholarship (MAITS)

The Mahindra All India Talent Scholarship was established by KCMET in 1995. This scholarship offered to students belonging to economically disadvantaged families. This scholarship helps students to pursue a job-oriented diploma course in recognized Govt. Polytechnic in India. This scholarship encourages and supports the lower-income family students. Students belong to rural and urban areas can pursue job-oriented diplomas. Application releases in national and regional newspapers in June/July every year. The Shortlisted candidates will be informed regarding their interview date and venue details. So that they can attend the interview at 12 centers across India.

Rewards

  • This scholarship awarded to 9640 students till date across India.
  • KCMET awards Rs. 10,000/- per annum to 550 students every year for a period of 3 years.
  • Candidates will be paid the value of travel to the interview center up to a certain amount.

Eligibility

  • Candidates who are studying in the first-year diploma in Govt. Polytechnics.
  • Candidates who have passed SSC/HSC or similar exams in class 10th/12th.
  • Candidates should have secured admission in Govt. or any other recognized Polytechnic for diploma courses.
  • Candidates should have secured more than 60% marks in the board examinations.
  • Girl and lower-income families candidates are given preference.
  • Candidates from differently-abled and armed forces personnel are also preferred.

KC Mahindra Scholarship 2019 – Overview

ParticularsDetails
Organization NameKC Mahindra Education Trust
Scholarship NameKC Mahindra Scholarship
Start date of KC Mahindra PG Study in Abroad Scholarship ApplicationJanuary 2019
Last date of KC Mahindra PG Study in Abroad Scholarship Application SubmissionApril 2019
Final Interview of KC Mahindra PG Study in Abroad Scholarship Shortlisted CandidatesJuly 2019
Start date of KC MAITS ApplicationJune/July 2019
Last date of KC MAITS Application SubmissionBetween August to October 2019
Websitewww.kcmet.org

KC Mahindra Scholarship 2019 – Selection Process

The trustees of KCMET has appointed a specific selection committee. This committee has the responsibility of selecting the deserving and needy students. The committee analyses and evaluates the application based on some qualities of students. These qualities are merit, ambitions, aptitude, character, and others. Students can work in any organization or institution after completion of the course. The trustees of the KCMET are not responsible to provide any employment to any student. Preference will be given to students who are interested to pursue their studies in specific subjects and areas. These specific subjects and areas related to the country’s development and its importance. Some of the examples are Biogas technology for rural areas, Oceanography, Fisheries and Survey, and so forth.

KC Mahindra Scholarship 2019 – Application Process

KC Mahindra Scholarship application will be available in both online and offline mode. The application process for PG Study Abroad Scholarship has closed in April 2019. The PG Study Abroad Scholarship application starts in December/January every year. The application process for KC MAITS closes between August to October 2019. The KC MAITS application starts in June/July every year. Candidates go through the rules and regulations of this scholarship to avoid errors. Refer to the below points of online as well as the offline application process.

Online Application Process

  • Visit the official website of the KC Mahindra Education Trust.
  • Click on apply to fill the application form available in pdf format.
  • Candidates should keep ready all the relevant documents while filling an application.
  • Fill the accurate and necessary details in capital letters in the form.
  • Recheck the filled application to avoid rejection by KCMET during verification.
  • Attach attested copies of the relevant documents to complete the online application process.
  • Then, submit the filled application along with the attached documents.

Offline Application Process

  • Visit the official website www.kcmet.org.
  • Download the application form available in pdf format in Scholarships and Grants page.
  • Take a hard copy of the application form and fill the essential details correctly.
  • Recheck the filled application to avoid rejection by KCMET during verification.
  • Attach attested copies of the relevant documents to complete the offline application process.
  • Then, submit the filled application along with the documents to the following address:
    • KC Mahindra Education Trust, 3rd Floor,
      Near Regal Cinema Colaba, Cecil Court, M.B. Marg,
      Apollo Bandar Colaba, Mumbai – 400001.

KC Mahindra Scholarship 2019 – Checklist of Documents

Candidates have to attach all the essential documents for verification. Candidates must attach the following documents to apply for the KC Mahindra Scholarship.

  • One recommendation letter
  • One copy of the admission letter
  • Passport size photograph with signature
  • A statement was written by the candidate constituting their ambitions, and other aspirations
  • One copy of GRE/GMAT score of the candidate, if applicable
  • One copy of IELTS/TOEFL score of the candidate, if applicable
  • Attested copies of candidates course and degree certificates and mark sheets
  • Candidates rank certificate, if applicable
  • Attested copies of candidates 10th, 12th, and school leaving certificate
  • A self-attested copy of the candidate’s age proof
  • Copy of Family income certificate of the candidate
  • Copy of Aadhaar card
  • Self-attested copy of the first page of bank passbook or canceled cheque

KC Mahindra Scholarship 2019 – Contact Details

In case of any queries related to KC Mahindra Scholarship or any assistance contact on the below-mentioned details.

For queries on KC Mahindra Scholarship for Postgraduate study Abroad contact below

Phone Number022-22895526
AddressKC Mahindra Education Trust, 3rd Floor,
Near Regal Cinema Colaba, Cecil Court, M.B. Marg, Apollo Bandar Colaba, Mumbai – 400001
Email Idrodrigues.kieran@mahindra.com

For queries on KC Mahindra All India Talent Scholarship (MAITS) contact below

Phone Number022-22897848
AddressKC Mahindra Education Trust, 3rd Floor,
Near Regal Cinema Colaba, Cecil Court, M.B. Marg, Apollo Bandar Colaba, Mumbai – 400001
Email IdRAMCHANDRAN.p2@mahindra.com

FAQ’s on KC Mahindra Scholarship 2019

Question 1.
What is KCMET?

Answer:
KC Mahindra Scholarship is an initiative of the KC Mahindra Education Trust, also known as KCMET. It offers scholarships and grants to the most deserving and needy students.

Question 2.
What are the scholarships offered by the KC Mahindra Education Trust?

Answer:
KCMET offers the following scholarships to encourage the lower-income family students.

  • KC Mahindra Scholarship for Post Graduation study Abroad
  • Mahindra Talent Search Scholarship
  • KC Mahindra United World College Scholarship
  • KC Mahindra All India Talent Scholarship (MAITS)

Question 3.
What are the application starting and closing dates for the KC Mahindra PG Study in Abroad Scholarship?

Answer:
The application process for this scholarship starts in January 2019. Whereas the closing date of application submission is April 2019.

Question 4.
What are the application starting and closing dates for the KC MAITS?

Answer:
The application process starts on June/July 2019 for the KC MAITS. Whereas the closing date of application submission is between August to October 2019.

Question 5.
What is the duration of the KC Mahindra PG Study Abroad Scholarship?

Answer:
The duration of this scholarship is 2 years. If the scholars want to extend it then they have to get specific approval from KCMET.

Hope this article will help you to know more about the KC Mahindra Scholarship 2019. If you have queries related to KC Mahindra Scholarship, then leave it in the comment box to get in touch with us.

The post K.C. Mahindra Scholarship 2019-20 | Dates, Eligibility, Awards, and Application Process  appeared first on Learn CBSE.

ONGC Scholarship 2019 | Important Dates, Eligibility, Rewards, Application

$
0
0

ONGC Scholarship 2019: ONGC Scholarship 2019 is a CSR initiative run by the Oil and Natural Gas Company (ONGC) Limited. ONGC Scholarship offers scholarships to SC/ST meritorious students for pursuing professional courses. These are Engineering, Medical and Master degrees in Business Administrative, Geology, and Geophysics. Candidates who are Indian nationals and studying in the specified courses can apply for this scholarship scheme. Candidates can fill the application form and apply for this scholarship on or before the 3rd week of January 2020. 

ONGC scholarship rewards 1000 meritorious candidates every year. 50% of this scholarship is reserved for girl candidates. However, five zones in the states of India are eligible to apply for this scholarship. The article below provides the information that is eligibility, application, and so forth about ONGC Scholarship 2019.

ONGC Scholarship 2019 for SC/ST meritorious students is one of its initiatives to encourage them to pursue higher education. This scholarship scheme helps SC/ST meritorious students to complete professional courses without financial hurdle. ONGC is a multinational oil and gas public enterprise company in India. It is the largest producer of natural gas and oil across the nation. As a part of Corporate Social Responsibility (CSR), the company has introduced various schemes. These schemes focus on areas like education, healthcare, entrepreneurship, women empowerment, water management and many more. 

ONGC Scholarship Overview

ParticularsDetails
Organization NameOil and Natural Gas Corporation (ONGC) Limited
Scholarship NameONGC Scholarship for SC/ST meritorious students
Total number of Available Scholarships1000
Amount of ScholarshipRs. 4,000/- per month to each scholar (Rs. 48,000/- per annum)
Stream-wise Scholarships DistributionEngineering – 494
MBBS – 90
MBA – 146
Masters in Geology/ Geophysics – 270
Zone-wise Scholarships DistributionJ&K, Delhi, Punjab, Himachal Pradesh, Haryana, Chandigarh, Uttarakhand, Uttar Pradesh – 200.
Maharashtra, Gujarat, Rajasthan, Madhya Pradesh, Goa, Daman & Diu, Dadra & Nagar Haveli – 200.
Assam, Sikkim, Mizoram, Arunachal Pradesh, Nagaland, Manipur, Meghalaya, Tripura – 200
Bihar, Jharkhand, Odisha, Chhattisgarh, West Bengal – 200.
Tamil Nadu, Kerala, Karnataka, Andhra Pradesh, Telangana, Puducherry, Lakshadweep, Andaman & Nicobar Islands – 200.

ONGC Scholarship 2019 – Important Dates

EventsImportant Dates
Start of ONGC Scholarship Application3rd Week of November 2019
Deadline to Submit ONGC Scholarship Application3rd Week of January 2020
Declaration of Final List of Scholarship BeneficiariesLast Week of March 2020

Scholarships for Students

ONGC Scholarship 2019 – Eligibility Criteria

Applicants must fulfill the following eligibility criteria to apply for ONGC Scholarship:

  • Students studying in India and must belong to Indian citizenship.
  • The age limit of the students must not be more than 30 years as on 1st November of the current academic session.
  • SC/ ST category students can only apply for this scholarship.
  • Candidate must be pursuing the following full-time courses in an AICTE/UGC/State or Central Govt approved institutes.
  • 1st year of Engineering/ MBBS courses
  • 1st year of MBA or Master in Geology/ Geophysics courses
  • Candidates pursuing the following courses are also eligible
    • 4 years of Engineering or MBBS courses
    • 2 years Masters course in Business Administration
    • 2 years Masters course in Geology/ Geophysics
  • The family income of candidates including all sources should not be more than 4.5 Lakh per annum.
  • Candidates must have passed with minimum 60% marks or equivalent CGPA in class 12 for Engineering/ MBBS degrees.
  • Candidates must have passed graduation with minimum 60% marks or 6.0 CGPA for MBA or Masters in Geology/ Geophysics.

ONGC Scholarship 2019 – Selection Process

The selection process for ONGC Scholarship is given below

  • It is a merit-based scholarship which considers academic merit prescribed for each eligible course for final selection.
  • The company prepares the list of final selected scholars and invite them for an interview.
  • If candidates having an equal percentage in the exam then the one with lower family income will be selected for this scholarship.
  • First preference will be given to candidates from Below Poverty Line (BPL) families.
  • If sufficient candidates from BPL families satisfying the eligibility criteria are not available then only other candidates will be considered.
  • If the candidate pursuing an Engineering or MBBS degree then the selection is based on the performance in the 12th class examination.
  • Whereas the selection of candidates will be based on the performance at the graduation level who are pursuing an MBA or Masters degree in Geology or Geophysics.

ONGC Scholarship 2019 – Application Process

The application for ONGC Scholarship will be available online in the third week of November 2019. Candidates can fill and submit the application in the 3rd week of January 2020. Refer to the below points to apply for ONGC Scholarship 2019.

  • Candidates should visit the official website www.ongcindia.com.
  • Click on the “Advertisement of ONGC Scholarship to SC/ST” appear under the “News and Update” section of the homepage.
  • Download the application format available in pdf form.
  • Read ONGC SC/ST Scholarship details carefully. Fill all the essential details in the application form in capital letters.
  • Recheck the application thoroughly to avoid rejection by the authorities during verification.
  • Attach xerox copies of all required documents along with the application. The documents must be duly certified as well as forwarded by the Head/ Principal of the School/ College/ University.
  • Enclose the completed application along with the documents in an envelope. Then, send it to the designated ONGC office of your respective zone.
  • The zone from where you can apply is based on the location of your college/university of the qualifying exam.
  • The zone is not decided on the basis of your domicile.
  • Send the completed application to the below-mentioned address of designated zone offices.

ONGC Scholarship – Zone-wise Address for Application Submission

ZoneAddress
North ZoneChief Manager (HR), ONGC,
A-Wing, Reservation Cell, Green Hills, Telephone Bhavan, Dehradun – 248003
West ZoneGM (HR),
ONGC, NBP Green Heights, Plot No. C-69, Bandra Kurla Complex, Bandra (E), Mumbai – 400051
North-East ZoneIncharge HR/ER, ONGC, SVS, 2nd Floor, Central Workshop, Assam Asset, B.G. Road, Assam, Sivasagar – 785640
East ZoneIncharge HR/ER,
MBA Basin, ONGC, 50 – J.L. Nehru Road, Kolkata – 700071
South ZoneIncharge HR/ER,
ONGC, 7th Floor, East Wing, CMDA Tower – I, Gandhi Irwin Road, Egmore, Chennai – 600008

ONGC Scholarship 2019 – Checklist of Documents

Candidates must include the below-mentioned documents along with the application to apply for ONGC Scholarship.

  • Passport size photograph of the candidate
  • Caste certificate in English/Hindi language
  • Age proof (birth certificate or 10th class mark sheets)
  • Marksheets of the qualifying examination
  • Annual family income certificate in English/Hindi language
  • Bank details of the candidate in the prescribed format
  • Xerox copy of PAN Card

ONGC Scholarship 2019 – Terms and Conditions

Candidates must refer to the below terms and conditions to apply for ONGC Scholarship:

  • Candidates who are applying for the ONGC scholarship should not be availing any other scholarship or financial assistance.
  • The scholars need to show satisfactory performance in each year’s annual exam in order to continue the scholarship every year.
  • The conduct of the scholar is also considered for scholarship continuation.
  • Candidates have to secure minimum 50% marks or a scale of 5 grade points out of 10.
  • If the candidate fails to maintain the academic requirements or conduct then the scholarship will not be paid to them for the respective year.
  • If the scholarship is discontinued, candidates can apply for a renewal by filling the scholarship renewal application form.
  • The renewal application form should be filled in the specified format.
  • If candidates are able to maintain the said percentage or grades again then only they can apply for scholarship renewal.

ONGC Scholarship 2019 – Result

The result for ONGC Scholarship will be released on the official website in the last week of March 2020. Candidates can go to the official website www.ongcindia.com to check the final list of the scholarship beneficiaries. The shortlisted candidate’s documents will be verified after declaring the result. The Scholarships will be awarded only to the shortlisted candidates by ONGC.

ONGC Scholarship 2019 – Contact Details

In case of any queries related to ONGC Scholarship or any assistance contact on the below-mentioned details.

Phone Number011-26750998
AddressPlot No. 5A-5B, Nelson Mandela Road, Vasant Kunj,
New Delhi – 110070
Email IDvijan_ar@ongc.co.in
Websitewww.ongcindia.com

FAQ’s on ONGC Scholarship 2019

Question 1.
How is ONGC Scholarship helpful to SC/ST students?

Answer:
ONGC Scholarship offers scholarships to SC/ST meritorious students for pursuing professional courses. These professional courses include Engineering, Medical and Master degrees in Business Administrative, Geology, and Geophysics.

Question 2.
How many scholarships are awarded to meritorious candidates every year?

Answer:
ONGC scholarship rewards 1000 meritorious candidates every year.

Question 3.
How many scholarships are reserved for girl candidates?

Answer:
50% of ONGC Scholarship is reserved only for girl candidates.

Question 4.
What are the ONGC Scholarship application starting and closing date?

Answer:
The ONGC Scholarship application starting date is the third week of November 2019. The application closing date is the third week of January 2020.

Question 5.
What is the age limit to apply for ONGC Scholarship 2019?

Answer:
The age limit of the students must not be more than 30 years as on 1st November of the current academic session.

Hope this article will help you to get more information about ONGC Scholarship 2019. If you have queries related to ONGC Scholarship, then leave it in the comment box to get in touch with us.

The post ONGC Scholarship 2019 | Important Dates, Eligibility, Rewards, Application appeared first on Learn CBSE.

HDFC Scholarship | Eligibility, Rewards, Key Dates, Application Process

$
0
0

HDFC Scholarship: HDFC Scholarship has initiated by HDFC Bank Parivartan. This scheme offers HDFC Scholarship to students who are economically and socially disadvantaged. This scholarship provides financial aid to the desired students who drop their education due to unavoidable circumstances. With the aim of empowering individuals towards economic growth, the bank has initiated various projects in the field of education and livelihood training. ECSS program is intended to provide provisional support to children facing a personal or economic crisis.

Economic hardship or a crisis in the family sometimes has a bad impact on the education of the children. In fact, resulting in children dropping out of school/college. ECSS provide help to these students to overcome the difficulties of personal/family crisis without adverse impact on their education. This scholarship is offered only for a single year and can be renewed for a period of maximum 2 years. Students can apply for HDFC Scholarship available on the official website from 1st July to 15th July 2019 to overcome the financial crisis. Students go through the article below to know more about HDFC Educational Crisis Scholarship Support.

HDFC Scholarship Dates

EventsDates
Start of HDFC Scholarship ApplicationJuly 1, 2019
Last Date of HDFC Scholarship ApplicationJuly 15, 2019
HDFC Scholarship Result ReleaseSeptember to October 2019
Selection list for HDFC Scholarship ReleaseOctober to November 2019
Dispatching of Scholarship Cheques/DD to selected studentsNovember 2019

HDFC Scholarship Crisis Criteria

Refer to the below Crisis Criteria

  • Sudden death or separation of parents.
  • Sudden occurrence of a critical or life-threatening illness.
  • Student/family requires financial support due to the sudden occurrence of an incident.
  • Sudden job loss of the breadwinner of the family due to accident or any incident.
  • Accident or death of siblings.

Scholarships for Students

HDFC Scholarship Eligibility Criteria

Students must fulfill the following eligibility criteria:

  • Students studying in class VI to XII from any Government aided or Private school.
  • Students pursuing UG/ PG/ Diploma from recognized institutes under UGC/AICTE/State/Central Govt.
  • Students whose family is facing a Crisis Situation due to the unfortunate incident occurred in the last 2 years.
  • Students who are unable to continue bearing the cost of education due to personal/family crises. Later on, arising the risk of dropping out of school/ college/ university.
  • Only one student per family can apply for this scholarship.
  • The family annual income of the student should not be more than 3 Lakh.

HDFC Scholarship Benefits

The Scholarship will be distributed in the form of a Demand Draft (DD) made in favor of School/ College/ University. The Scholarship will not be disbursed in cash or directly to the students. Each scholar will receive the following benefits:

ScholarsAmount
Class 6 to12Rs.10000/-
Undergraduate/PostgraduateRs.25000/-
Diploma/ITI/PolytechnicRs.25000/-

HDFC Scholarship Selection Procedure

Received applications from students will be assessed on the basis of the criteria defined by HDFC Bank’s management. Students will be selected based on the overall scores given to them during the selection process. Once the application will be submitted online, the verification by HDFC Bank is not required. Since all efforts are made to ensure a fair screening, the management reserves the right to decide the final recipients.

HDFC Scholarship Application Process

HDFC Scholarship application form will be available online in July 2019. Students can fill and submit the form on or before 15th July 2019. The Information Brochure will also be released along with the application form by the authorities of HDFC Bank. The Brochure contains all the important details and guidance for filling the form without any error. So, students must check carefully the details mentioned in the Information Brochure before filling the form. Refer to the below points to apply for HDFC Scholarship 2019.

  • Students should visit the official website of HDFC Bank: www.hdfcbank.com/scholarship.
  • Log in with the registration ID and password which are provided during the registration.
  • Fill the application with correct and valid information. The information will be cross-checked by the authorities at the time of document verification. If found any misperception then the application will not be considered for further processing.
  • Upload all the required documents either in pdf or jpeg format along with the application form.
  • The final submission of the application form will be in offline mode. Students have to send the form along with the self-attested documents to the address given in the official website by the HDFC authorities.
  • Next, take a print out of the application for future purpose.

Documents To Be Uploaded for HDFC Scholarship

Students can fill the scholarship application online on the HDFC Bank official website. After that, send the application in offline mode to the address notified in the official website. While sending the application must remember to attach the required documents. Students must keep ready all the important self-attested documents in prior. Refer to the below documents which need to be attached along with the application.

  • Salary slip along with the duly signed and stamped income proof issued by the concerned authority.
  • In the case of self-employed, attach the self-declaration of the family income.
  • Verification form duly signed and stamped by the head of the Institution.
  • Bank details of the educational institute, in favor of which the DD/Cheque will be issued.
  • Attested copies of last year’s mark sheets.
  • Two copies of address proof (Aadhar Card/ Voter ID/ Driving License).
  • Doctor’s note in case of illness.
  • 10th or 12th Mark Sheets and salary slip, if any.

HDFC Scholarship Important Terms and Conditions

Students have to remember some rules and regulations while applying for this scholarship. Refer to the below-given list of terms and conditions:

  • The documents uploaded must be in PDF or JPEG format and the size should not be more than 15 MB.
  • The scholarship amount is disbursed through Demand Draft (DD)/cheque will be drawn in favor of the educational institute of the shortlisted students.
  • Then, the DD/Cheque will be sent to the nearest branch of the HDFC Bank as mentioned in the application.
  • Student or institute will be intimated about it through the mail so that they can collect it from the bank.
  • The shortlisted students must refrain from getting involved in any malicious/undesirable activities.
  • Students must not violate any rules and regulations related to the School/ University.
  • Students must maintain the academic standards of the School/ University.
  • In case of any change in students School/ College/ University then the scholarship is non-transferable.
  • Students must not provide any false information at any stage of the scholarship process. Otherwise, it may result in the rejection of the application.
  • Also, the candidates will be disqualified from any future scheme initiated by HDFC Bank.

HDFC Scholarship Result

HDFC Scholarship 2019 result will be announced in the last week of September 2019. The result will also be published on the official website hdfcbank.com/ scholarship. Students can download and check the ECSS merit list on the official website. The shortlisted students are notified through the contact numbers given at the time of registration. The selected students will receive the scholarship amount through cheque or DD. Shortlisted students will get a call from the nearest HDFC bank to collect the scholarship cheque on November 2019.

HDFC Scholarship Renewal

The shortlisted students can apply for the HDFC Scholarship Renewal Programme. The scholarship can be availed for a maximum of 2 consecutive years as per HDFC Scholarship notification. Students will be able to avail the amount as per the duration. Students make sure to keep a steady score in the course they are pursuing. Also, students need to reapply and requalify for the scholarship each year.

HDFC Scholarship Contact Details

Students can contact the nearest branch of HDFC Bank for any query or assistance regarding the scholarship. However, students can also call or SMS on the number or email which is given below.

Phone No.022 3075 1030/022 3075 1006
AddressHDFC Bank G-1, Harbhajan, CST Road, Kalina Santacruz East, Mumbai – 400098
Email IDecss@hdfcbank.com
Websitewww.hdfcbank.com

FAQ’s on HDFC Scholarship 2019

Question 1.
How is HDFC Scholarship helpful to students?

Answer:
HDFC Scholarship has initiated by HDFC Bank Parivartan. This Scholarship offers support to economically and socially disadvantaged students. It provides help to school/college drop out students to overcome the difficulties of personal/family crisis without adverse impact on their education.

Question 2.
What is the duration of HDFC Scholarship?

Answer:
HDFC Scholarship is offered only for a single year and can be renewed for a maximum of 2 consecutive years.

Question 3.
What is the application deadline to apply for HDFC Scholarship 2019?

Answer:
The application deadline to apply for HDFC Scholarship 2019 is 15th July 2019.

Hope this article will help you to get information about HDFC Educational Crisis Scholarship Support (ECSS). If you have queries related to this article, leave it in the comment box to get in touch with us.

The post HDFC Scholarship | Eligibility, Rewards, Key Dates, Application Process appeared first on Learn CBSE.

Fair & Lovely Foundation Scholarship 2019 | Dates, Eligibility, Application

$
0
0

Fair and Lovely Foundation Scholarship: Fair & Lovely Scholarship has initiated by Fair & Lovely Foundation, a leading Hindustan Unilever global skincare brand. Hindustan Unilever Limited (HUL) is the largest manufacturing Company in India. HUL works to help people look and feel good, and get more out of life with quality brands and services. Fair & Lovely Foundation Scholarship provides financial support and confidence to only female students for pursuing higher education. Since it launched in 2003, offers scholarship benefits to thousands of female students across India. Application form for this scholarship will be available online in the first week of September 2019. Interested candidates can fill the form and apply for this scholarship until the last week of October 2019.

Fair and Lovely Foundation encourages females to fulfill their dreams and prove their capability. This Scholarship is available for countries like India, Bangladesh, Egypt, Pakistan, and Saudi Arabia until now with the aim to join more countries in the future. This Scholarship helps females to become independent and self-reliant to brighten their career. It supports talented females to overcome their bad circumstances and fulfill their mission for higher education, vocational, job training and career guidance. It is also giving confidence to females to overcome hesitations and fears to achieve goals in a career. With the help of this scholarship, brilliant women who are economically disadvantaged can continue their study without financial constraint. Students go through the article below to know more about the Fair & Lovely Foundation Scholarship.

Fair and Lovely Foundation Scholarship Dates

EventsDates
Start of ApplicationSeptember 1, 2019
Last Date of ApplicationOctober 30, 2019
Shortlisting and Finalizing StudentsNovember 10, 2019
Slot Booking for Telephonic Round
Interview
November 2019
Slot for Telephonic InterviewNovember to December 2019
Fair & Lovely Foundation Scholarship Result ReleaseDecember 2019
Scholar Onboarding, Document Verification2nd Week of January 2020
Disbursement of Scholarship to Qualified Students1st Week of February 2020

Scholarships for Students

Brief Details about Fair and Lovely Scholarship

ParticularsDetails
Scholarship amountINR 25,000 to INR 50,000 to each of the 55 selected students
Eligibility criteriaClass 12 passed students to postgraduates. candidates taking coaching classes
Annual family income of INR 6 Lakhs
Applicants must be 15 to 30 years old.
Application processOnline application

Scholarships for Students

Fair and Lovely Foundation Scholarship Eligibility Criteria

Females must fulfill the following eligibility criteria:

  • Candidates who have passed with distinction or secured high percentage of marks are eligible.
  • All-female candidates who have scored up to 60% marks in 10th and 12th class and desire to pursue higher education.
  • Candidates studying vocational courses should have passed 8th class with a minimum of 50% of the marks.
  • The family annual income of the candidate must be less than 4 Lakh.
  • The age limit of general and OBC candidates must be between 15 to 30 years. Whereas the age of SC and ST category candidates should not be more than 32 years as per the Foundation norms.
  • Candidates should be interested to pursue graduation or post-graduation from Govt. recognized institutes in India.
  • Candidates must be pursuing following courses at different levels.
    • At Undergraduate Level: BA, B.Com., B.Sc., BCA, BBA, BE/ B.Tech., LLB, B.Pharma, B.Arch., MBBS, BDS, BHMS, BAMS, BHM, B.P.Ed, B.Ed., B.SL LL.B, BBA LLB, B.Com LLB
    • At Postgraduate Level: MA, M.Com., M.Sc., MCA, MBA, ME/ MTech, LLM, M.Pharma, M.Arch, MDS, MHMS, MAMS, MD, MHHM, M.P.Ed, M.Ed, others
    • Coaching Classes: CAT-MBA, Civil Services, Banking Services, CA-CS-ICWA, IIT-JEE, and so forth.

Fair and Lovely Foundation Scholarship Benefits

Fair & Lovely Scholarship amount will be awarded only to the selected students. So that they can pursue graduation/ post-graduation or coaching classes for certain courses. Refer to the following benefits:

  • This scholarship empowering women society to fulfill their dreams of pursuing higher studies.
  • Fair & Lovely Foundation awarded scholarship benefits to 55 selected students last year.
  • Fair & Lovely Foundation rewarded the one-time scholarship amount of Rs. 25,000/- to Rs. 40,000/- per student.
  • Selected students will receive the benefit of a scholarship amount of Rs. 1 Lakh per annum.
  • An additional amount of Rs. 3,000/- per year is provided to the girls who are from rural areas to meet their education-related expenses.

Fair and Lovely Foundation Scholarship Selection Process

The selection process for Fair & Lovely Scholarship is given below:

  • Female candidates have to apply for this Scholarship online by providing all the required details.
  • Candidates are asked to answer two subjective questions related to themselves. Then, candidates will be shortlisted on the basis of their submitted application.
  • Candidates get a percentile ranking based on merit, family income, streams and special criteria such as physically handicapped, single parents, orphans, and so forth.
  • Shortlisted candidates will be called for the telephonic interview before the result declaration.
  • An experienced panel of interviewers will conduct a small 10-15 minutes telephonic interview. The interview will be taken in English, Hindi or South Indian language only. The HUL Team Consultation will only decide the interview questions.
  • The top student’s list will be prepared on the basis of their performance in the interview round.
  • Candidates can get information through an SMS or Email about F&L Scholarship selection in the interview round.
  • The final selection of scholars will be decided by HUL scholarship team.

Fair and Lovely Foundation Scholarship Application Process

The Scholarship application form will be available online on 1st September 2019. Students can fill and submit the form on or before 30th October 2019. Students can fill the Fair & Lovely Scholarship application form in both online and offline mode.

Refer to the below points to apply for Fair & Lovely Foundation Scholarship 2019 in both online and offline mode.

  • Visit the official website www.fairandlovelyfoundation.in.
  • Click on the Registration link to create your account. Register yourself by entering Name, Email, Password, Date of Birth and Gender.
  • An email will be sent to your registered email id for verification purposes.
  • Click on the email verification link which will be redirected to the partner’s website to fill the application form.
  • Select the apply online option to fill the application with correct and valid information.
  • The application form will be cross-checked by the authorities at the time of document verification.
  • If found any false information then the application will not be accepted for further processing.
  • After filling the application, download it for offline submission.
  • Send the filled application along with the relevant documents by courier to the address below:
    • Fair & Lovely Foundation,
      Hindustan Unilever House,
      B.D. Sawant Marg, Chakala,
      Mumbai-400099
  • For online submission, upload scanned copy of all the relevant documents along with the application form.
  • Click on submit to apply for Fair & Lovely Foundation Scholarship.
  • Finally, take a photocopy of the application for future purpose.

Documents Need to Be Submitted for Fair and Lovely Foundation Scholarship

Candidates must keep ready the scanned copy of all the self-attested documents at the time of filing the application. Refer to the below documents which need to be submitted along with the application.

  • ID proof (Aadhar Card/ Voter ID/ Driving License/ PAN Card/ 10th class mark sheet with photo).
  • One recent passport size photograph.
  • Attested copies of class 10th and 12th certificates and mark sheets.
  • Age proof and family income proof (Valid Govt. approved documents like income certificate, IT Return or BPL card).
  • Latest graduation mark sheet
  • Latest college fee receipt
  • Application Letter

Fair and Lovely Foundation Scholarship Result

Fair & Lovely Foundation Scholarship 2019 result will be announced in December 2019. The result will also be published on the official website www.fairandlovelyfoundation.in. Candidates can download and check the Fair & Lovely Scholarship merit list on the official website. The shortlisted candidates can get the selection information through the contact numbers given at the time of registration. The selected candidates will have to provide the documents for verification upon the declaration of result. The scholarship will be disbursed to the Shortlisted candidates in the first week of February 2020.

Fair and Lovely Foundation Scholarship Contact Details

In case of any query or assistance regarding Fair & Lovely Foundation Scholarship, send the queries to the below address. However, candidates can also call or send an email which is mentioned below.

Phone No.+91-8448857958
AddressFair & Lovely Foundation,
Hindustan Unilever House,
B.D. Sawant Marg, Chakala,
Mumbai-400099
Email IDlever.care@unilever.com
Websitewww.fairandlovelyfoundation.in

FAQ’s on Fair and Lovely Foundation Scholarship 2019

Question 1.
How is Fair & Lovely Foundation Scholarship helpful to students?

Answer:
Fair & Lovely Foundation Scholarship provides financial support and confidence to only female students for pursuing higher education.

Question 2.
How many students selected for receiving the Fair & Lovely Scholarship last year?

Answer:
Fair & Lovely Foundation awarded scholarship benefits to 55 selected students last year.

Question 3.
What is the application deadline to apply for Fair & Lovely Foundation Scholarship 2019?

Answer:
The application deadline to apply for Fair & Lovely Foundation Scholarship 2019 is 30th October 2019.

Question 4.
How much scholarship amount will be offered to the selected students?

Answer:
Selected students will receive the benefit of a scholarship amount of Rs. 1 Lakh per annum. An additional amount of Rs. 3,000/- per year is provided to the girls who are from rural areas to meet their education-related expenses.

Question 5.
What is the age limit to apply for Fair & Lovely Foundation Scholarship 2019?

Answer:
The age limit of general and OBC candidates must be between 15 to 30 years. Whereas the age of SC and ST category candidates should not be more than 32 years as per the Foundation norms.

Hope this article will help you to get information about the Fair & Lovely Foundation Scholarship 2019. In case of any queries related to this article, leave it in the comment box to get in touch with us.

The post Fair & Lovely Foundation Scholarship 2019 | Dates, Eligibility, Application appeared first on Learn CBSE.


District Merit Scholarship (DMS) 2019 | Important Dates, Eligibility, Application

$
0
0

District Merit Scholarship (DMS) 2019: The Directorate of Collegiate Education (DCE), Government of Kerala has released the notification for the District Merit Scholarship (DMS) 2019. The District Merit Scholarship has initiated by the Directorate of Collegiate Education, Govt. of Kerala. This scheme is available only for the students of Kerala State. The government of Kerala grants District Merit Scholarship (DMS) 2019 to students who have passed in all subjects in the SSLC exam with A+ grade. This SSLC exam must be conducted by the Board of Public Examination, Kerala.

The application form for District Merit Scholarship will be available online on August 1, 2019. Candidates can fill the application and apply for this scholarship till 30th September 2019. The motive behind the DMS is to notify candidates regarding engineering courses after class 10th or 12th every year. Read the article below to get more information about District Merit Scholarship 2019.

District Merit Scholarship (DMS) 2019

DMS is available for students of Kerala who are currently studying in Higher Secondary, VHSC, ITI or polytechnic Courses. This Scholarship is one of the initiatives of Govt. of Kerala to encourage Kerala students for pursuing engineering courses. DMS provides financial support to the underprivileged students who cannot afford their education. It also helps the meritorious students to complete engineering courses without financial hassle.

Govt. of Kerala has introduced DMS to award the outstanding meritorious students in the education sector as well as in the economically and socially backward classes. This project envisages the development and implementation of a web-based scholarship management system. It is an efficient and foolproof system which facilitates DCE for selection of students and disbursements of scholarship.

Govt. of Kerala releases both Fresh and Renewal application form to apply for DMS. It is mandatory that candidates applying for DMS Fresh Application should not applied for it earlier. On the other hand, a Renewal Application is for those candidates who have applied for it earlier.

District Merit Scholarship (DMS) Overview

ParticularsDetails
Organization NameKerala Directorate of Collegiate Education (DCE Scholarship)
Scholarship NameDistrict Merit Scholarship (DMS) Fresh & Renewal 2019-20
Amount of ScholarshipRs. 1,250/- per annum to each scholar
Applicable StateKerala
Official Websitewww.dcescholarship.kerala.gov.in

Scholarships for Students

District Merit Scholarship 2019 – Important Dates

EventsImportant Dates
Start of DMS Fresh Scholarship ApplicationAugust 1, 2019
Deadline to Submit DMS Fresh Scholarship ApplicationSeptember 30, 2019
Release of DMS ResultOctober 2019
Disbursement of Scholarship to the selected studentsOctober 2019
Start of DMS Renewal Scholarship ApplicationAugust 1, 2019
Deadline to Submit DMS Renewal Scholarship ApplicationSeptember 30, 2019

District Merit Scholarship 2019 – Eligibility Criteria

Applicants must fulfill the following eligibility criteria to apply for DMS 2019:

  • Students must have passed A+ grade in all subjects in the SSLC examination.
  • Students must have passed the SSLC examination conducted by the Board of Public Examinations, Kerala State.
  • Students studying in Higher Secondary / VHSC / ITI / Polytechnic courses can apply for DMS.

District Merit Scholarship 2019 – Selection Process

The Higher Education Department of Govt. of Kerala is responsible for the selection of the meritorious students. The final selection for DMS depends on the academic record and income of parents.

District Merit Scholarship 2019 – Application Process

The application for DMS will be available online on August 1, 2019. Candidates can fill and submit the application on or before September 30, 2019. Refer to the below points to apply for District Merit Scholarship 2019.

  • Candidates should visit the official website of DCE, Govt.of Kerala.
  • Select the “District Merit Scholarship (DMS)” from the types of scholarship list. Refer to the picture below:

DMS

  • Then, click on “Apply Online” to redirect to the Candidate Login page.
  • Already registered candidates can enter the login details as shown in the below picture

DMS Scholarship

  • Otherwise, click on the “New Registration” to generate an account.
  • Fill the correct and valid details in the application form in capital letters.
  • Recheck the application thoroughly to avoid rejection by DCE, Govt of Kerala during verification.
  • Upload scanned copies of all required documents along with the application. The documents must be attested by the Head of the School/ College/ University.
  • Submit the completed application along with the required documents.
  • Then, you will receive an automated email regarding login Id and password.
  • Keep the credentials safe for your future usage.
  • Finally, take a print out of the DMS application form for future purposes.

District Merit Scholarship 2019 – Checklist of Documents

The below-mentioned documents must be uploaded along with the application to apply for DMS 2019.

  • Recent passport size photograph
  • Attested copies of school/college certificates and mark Sheets
  • Candidates annual family income certificate and affidavit for any additional income
  • Attested copies of caste certificate
  • Age proof (birth certificate or class 10th board certificate)
  • Residential proof (electricity bill, ration card, voter Id, aadhar card, and so forth)
  • Attested copies of disability certificates, if any
  • Candidates Bank Account details in the prescribed format

District Merit Scholarship 2019 – Renewal

The DMS Renewal application form will be released online on August 1, 2019. Candidates can fill the application and apply for renewal on or before September 30, 2019. Candidates who have already registered for DMS must renew it every year. Candidates should have satisfactory performance in each year’s annual exam to renew this scholarship. Candidates should maintain the academic requirements and conduct to apply for DMS Renewal. Refer the below points to Renew the District Merit Scholarship application form.

  • Candidates should visit the official website of DCE, Govt. of Kerala to renew the application.
  • Login by entering registration Id, date of birth, password and selecting Renewal. Edit the old application form which exists in the DCE portal.
  • Make sure that the modified details are accurate and filled in the specified format. For any corrections modify the form accordingly.
  • Then, submit the DMS Renewal application form and check the status online.

District Merit Scholarship 2019 – Result

The result for DMS will be released on the official website in October 2019. DCE prepares the final selection list of candidates to award the scholarship. Candidates can go to the official website of DCE, Govt. of Kerala to check the selection list of the scholarship beneficiaries. Candidates can check the selection list by entering the Scholarship Type, State, District, and College/ Institution Name. The list of selected candidates name and institution name will appear on the screen. Candidates can search their name in the selection list to confirm their chances of receiving the scholarship. The selected candidate’s documents will be verified by DCE after declaring the result. The Scholarships will be disbursed only to the selected candidates by DCE, Govt.of Kerala on October 2019.

District Merit Scholarship 2019 – Contact Details

In case of any queries related to District Merit Scholarship or any assistance contact on the below-mentioned details.

Phone Number0471-2306580 / 9446096580
AddressDirectorate of Collegiate Education (DCE), 6th Floor, Vikas Bhawan, Thiruvananthapuram -695033
Email IDdcescholarship@gmail.com

FAQ’s on District Merit Scholarship 2019

Question 1.
How is District Merit Scholarship helpful to Kerala students?

Answer:
The government of Kerala grants District Merit Scholarship (DMS) 2019 to students who have passed in all subjects in the SSLC exam with A+ grade. This SSLC exam must be conducted by the Board of Public Examination, Kerala.

Question 2.
Is District Merit Scholarship available to all state students?

Answer:
No, this scholarship is available only for the students of Kerala State. The District Merit Scholarship has initiated by the Directorate of Collegiate Education, Govt. of Kerala.

Question 3.
How much scholarship amount will be disbursed to the meritorious students?

Answer:
The scholarship amount of Rs. 1,250/- per annum will be disbursed to each scholar.

Question 4.
What are the DMS application starting and closing date?

Answer:
The DMS application starting date is August 1, 2019. Whereas the closing date is September 30, 2019.

Question 5.
The District Merit Scholarship is available for which courses?

Answer:
District Merit Scholarship is available for students of Kerala who are currently studying in Higher Secondary, VHSC, ITI or polytechnic Courses.

Hope this article will help you to get more information about District Merit Scholarship (DMS) 2019. If you have queries related to DMS, then leave it in the comment box to get in touch with us.

The post District Merit Scholarship (DMS) 2019 | Important Dates, Eligibility, Application appeared first on Learn CBSE.

Wharton Business School MBA Scholarship | Eligibility, Benefits and Application

$
0
0

Wharton MBA Scholarship: Wharton MBA Scholarship organized by Wharton Business School, University of Pennsylvania. Wharton provides a broad range of fellowship opportunities for many outstanding students. The credit goes to the generosity of individuals, corporate and foundation donors, and the Wharton School. Wharton fellowship provides to all admitted students based on the application for admission. Wharton business school offer awards in the Admission and Financial Aid Letter. Fellowships are two-year awards which are split equally over four semesters for a typical MBA student. Wharton fellowship committee does not provide any reconsideration of fellowship support.

Wharton school of business offers scholarships to students to study or research there. The number of Wharton Business School scholarships depends on the availability of funds. Wharton broad range of fellowships target students of all backgrounds and cover tuition fees plus living expenses. As we know the education cost of USA is very high. However, Wharton school of business for an MBA degree offers up to 100% scholarship which makes studying MBA in USA affordable. Students pursuing an MBA is an investment in themselves and in their future. Wharton helps students exploring financing options so that they make informed decisions.

Wharton Business School MBA Scholarship

The Wharton School of the University of Pennsylvania was founded in 1881 as the first collegiate business school. It is recognized globally for intellectual leadership. It is an ongoing innovation across every major discipline of business education. With the most published business school faculties and broad global community, Wharton creates economic and social value across the world. The school has around 5000 undergraduate, MBA, executive MBA, and Ph.D. students. It has more than 13000 participants in executive education programs every year. It has also a powerful alumni network of 98000 graduates. The Wharton School of business is accredited by the International Association for Continuing Education and Training (IACET) and is authorized to issue the IACET CEU.

Wharton MBA Scholarships

Wharton School has a range of scholarships for talented students with financial constraints. The Wharton School provides the following merit-based MBA Scholarships for full-time MBA study.

Joseph Wharton Fellowships

These fellowships are named in honor of Joseph Wharton, the founder of the Wharton Business School. These are awarded to students with the following criteria:

  • Outstanding records of academic.
  • Personal, and professional achievements.

Howard E.Mitchell Fellowships

This fellowship was established in 1992, in honor of former Wharton professor Howard E. Mitchell, WG’51. These are awarded to traditionally underrepresented domestic populations. Specifically, Balck/African American, Latino/Hispanic, Native American/American Indian or Alaskan Native, and Native Hawaiian/Asian Pacific Islander. It provides full-tuition, leadership programming, and the facility to join a large network of other Mitchell fellows and alumni to a group of students in each year. These awards provide financial support to graduate students of color based on the following criteria:

  • Proven leadership
  • Academic excellence
  • Work experience or demonstrated commitment to empowering traditionally underrepresented groups

Emerging Economy Fellowships

These are offered to support students from emerging economies throughout the world.

Forte Fellowships

These are awarded to outstanding female students each year who demonstrate the following criteria:

  • Exemplary leadership academically
  • Professional or creative leadership
  • Proven commitment to the empowerment of women in education or the workplace.

Social Impact Fellowships

These are offered to students with demonstrated leadership in the public or not-for-profit sectors.

Corporate and Foundation Fellowships

The following list of selected corporations and foundations are fellowship supporters of Wharton MBA fellowships:

  • American Express Philanthropic Program
  • Alcoa Inc.
  • Bank of America Corporation
  • Bain & Company, Inc.
  • Bechtel Foundation
  • Credit Suisse
  • Citigroup Foundation
  • Deutsche Bank
  • Daimlerchrysler Corporation Fund
  • Exxon Mobil Corporation
  • Ford Motor Company Fund
  • General Mills, Inc.
  • General Motors Foundation
  • The Goldman Sachs Group, Inc.
  • Merrill Lynch & Co., Inc.
  • New America Alliance Institute
  • Pension Real Estate Association
  • Robert R. Nathan Memorial Foundation
  • Robert A. Toigo Foundation

Yellow Ribbon Program Award for Military Students

These are awarded to funding all students who are 100% eligible under the Post-9/11 GI Bill. This award is a provision of the Post-9/11 Veterans Educational Assistance Act of 2008. The Yellow Ribbon Program eligible students must submit a VA Certificate of Eligibility (COE) to the Registrar’s Office.

Toigo Fellowships

This fellowship provides minority MBA students committed to a career in finance with the following criteria:

  • An unmatched network of contacts
  • A prestigious point of difference in the eyes of employers
  • One-on-one career guidance
  • Leadership training
  • A merit award, etc.

2nd Year Financial Aid Awards

These awards provide additional financial support to Wharton MBA students in their 2nd year of the MBA program. The Wharton Fellowship Committee will determine fellowship support based on the following:

  • Submitted applications
  • Review of existing loan balances
  • Prior fellowships
  • Other support received, etc.

Wharton MBA Scholarship Application Dates

RoundsApplication DeadlineInterview InvitationsDecisions
Round 1September 17, 2019October 29, 2019December 18, 2019
Round 2January 7, 2020February 13, 2020March 26, 2020
Round 3April 1, 2020April 16, 2020May 8, 2020

Wharton MBA Scholarship Benefits

Refer to the below benefits of applying for Wharton Business School MBA Scholarship.

  • Each fellowship provides a financial value of up to $20,000 to cover full tuition fees.
  • The emerging economy fellowship covers funding for both years of the Wharton MBA program.
  • Other scholarships are available in the 2nd year of study for students in the leadership roles and for teaching assistants.
  • Provides education-related expenses.
  • Provides living allowance for top students.
  • Eligible students may get a maximum Yellow Ribbon Award of up to $17,500.

Tuition and Cost Breakdown

First-Year Budget, 2019-20

Tuition and Fees ( Includes $2000 Pre-term Fee)$81,378
Room and Board$22,670
Books and Supplies$1,494
Personal, Health & Other Expenses$9,354
Total$114,896

Wharton MBA Scholarship Eligibility Criteria

First-year students from developing countries admitted to Wharton’s MBA program can only apply for the scholarship. Refer to the following eligibility criteria

  • Academic achievement.
  • Community involvement.
  • Exceptional professional development.
  • Unique personal qualities and background.

Wharton MBA Scholarship Application Deadlines

Students who are admitted to Wharton School will be automatically considered for the fellowships. There is no separate application form available to apply for these scholarships. Wharton considers admission applications as the scholarship application. Admitted students will be notified for the fellowship support with the admission decision. Students have to apply through the Wharton Business School online application system. The Wharton MBA program has 3 application deadlines every year with 3 key dates for each round. These key dates are Application Deadline,  Interviews, and Decision Dates. These application deadlines are also available for International students. The current application deadlines are 17th September 2019, 7th January 2020 and 1st April 2020.

Important Instructions

  • Students must submit a completed application by 5:00 p.m. Eastern Time (ET) on the day of the deadline will be considered for the round.
  • If students submit their application after the deadline for Rounds 1 or 2, then it will be rolled into the next round.
  • If students submit their application after the deadline for Round 3, then it will not be accepted.
  • Students interviews will be held by invitations only.
  • If students selected for an interview, then they will be contacted to arrange an interview as part of the application process.
  • At any stage, students can use the online application system to check the status of their application.

FAQ’s on Wharton Business School MBA Scholarship

Question 1.
Why Wharton Business School?

Answer:
Wharton provides a broad range of fellowship opportunities for many outstanding students. The credit goes to the generosity of individuals, corporate and foundation donors, and the Wharton School. However, Wharton school of business for an MBA degree offers up to 100% scholarship which makes studying MBA in USA affordable.

Question 2.
How to apply for the Wharton MBA Scholarship?

Answer:
There is no separate application form available to apply for these scholarships. Wharton considers admission applications as the scholarship application. It offers awards in the Admission and Financial Aid Letter. Students have to apply through the Wharton Business School online application system.

Question 3.
What are the application deadlines to apply for Wharton MBA Scholarship?

Answer:
The application deadlines are 17th September 2019, 7th January 2020 and 1st April 2020 to apply for Wharton MBA.

Hope this article will help you to get information about the Wharton Business School MBA Scholarship. If you have queries related to this article, leave it in the comment box and we will get back to you soon.

The post Wharton Business School MBA Scholarship | Eligibility, Benefits and Application appeared first on Learn CBSE.

CBSE Class 9 Hindi B व्याकरण वर्ण-विच्छेद

$
0
0

CBSE Class 9 Hindi B व्याकरण वर्ण-विच्छेद

वर्ण :

भाषा की वह छोटी से छोटी ध्वनि जिसके और छोटे खंड नहीं किए जा सकते हैं, उसे ‘वर्ण’ कहते हैं। अ, क्, प्, ट्, म्, ह आदि वर्ण हैं। वर्णों के और टुकड़े नहीं किए जा सकते हैं। ये भाषा की सबसे छोटी इकाई हैं, जो हमारे मुख से निकली हुई ध्वनियों के लिखित रूप होते हैं, इन्हें ही वर्ण कहा जाता है।

वह छोटी से छोटी ध्वनि जिसके और छोटे टुकड़े न किए जा सकें, उसे वर्ण कहते हैं। वर्ण-विच्छेद-किसी शब्द की रचना में जिन वर्णों का प्रयोग होता है, उन वर्गों को अलग-अलग करना वर्ण-विच्छेद कहलाता है; जैसे –

अनूप विद्यालय जाएगा-वाक्य के शब्दों का वर्ण-विच्छेद करें तो निम्नलिखित वर्ण मिलते हैं –

अनूप = अ + न् + ऊ + प् + अ,
विद्यालय= + व् + इ + द् + य् + आ + ल् + अ + य् + अ,
जाएगा = ज् + आ + ए + ग् + आ

वर्णमाला :

वर्णों के क्रमबद्ध समूह को ‘वर्णमाला’ कहते हैं।
हिंदी वर्णमाला में निम्नलिखित वर्ण हैं –
स्वर – अ, आ, इ, ई, उ, ऊ, ऋ, ए, ऐ, ओ, औ।
अयोगवाह – अं, अः
विसर्ग-अः
CBSE Class 9 Hindi B व्याकरण वर्ण-विच्छेद - 2
CBSE Class 9 Hindi B व्याकरण वर्ण-विच्छेद - 1

CBSE Class 9 Hindi B व्याकरण वर्ण-विच्छेद - 2(i)

अन्य वर्ण ‘ड’ और ‘ढ’- देखने में दोनों वर्ण ‘ड’ और ‘ढ’, ‘ड’ और ‘ढ’ के समान ही लगते हैं, परंतु इनके उच्चारण में पर्याप्त अंतर होता है। हाँ, एक बात अवश्व ही ध्यान रखने की है कि ‘ड’ और ‘ढ’ शब्द के प्रारंभ में नहीं आते हैं। इनसे कोई भी शब्द आरंभ नहीं होता है। ये शब्दों के बीच में या अंत में ही आते हैं। इसके विपरीत ‘ड’ और ‘ढ’ शब्द के आरंभ, मध्य या अंत में अर्थात् कहीं भी आ सकते हैं; जैसे –

  • बड़ा, घड़ा, घड़ियाल
  • डमरू, गुड्डी, हड्डी
  • दाढ़ी, दढ़ियल, बुढ़िया
  • ढक्कन, गड्ढा, बुड्ढा

संयुक्त व्यंजन – ‘क्ष’, ‘त्र’, ‘ज्ञ’ और ‘श्र’ को संयुक्त व्यंजन कहा जाता है, क्योंकि ये वर्ण एक से अधिक वर्गों के मेल से बने हैं; जैसे –

क्ष = क् + ष् + अ – क्षत्रिय, क्षमा, कक्षा, परीक्षा, तक्षक, तक्षशिला, लक्ष्मी आदि।
त्र = त् + र् +अ – त्रिशूल, त्रिफला, पत्र, त्रिनेत्र, पत्रिका, पत्रोत्तर आदि।
ज्ञ = ज् + ञ् + अ – यज्ञ, विज्ञ, विज्ञान, ज्ञान, संज्ञान, प्रतिज्ञा, अज्ञात आदि।
श्र = श् + र् + अ – श्रमजीवी, श्रमिक, श्री, आश्रय, आश्रम, विश्राम, श्रोता, श्रवण।

वर्गों के दो भेद हैं –

1. स्वर-जो वर्ण स्वतंत्र रूप से बोले जाते हैं तथा जिनके उच्चारण में हवा (वायु) बिना रुकावट के मुँह से बाहर आती है, उन्हें
स्वर कहते हैं। स्वर स्वतंत्र ध्वनियाँ हैं, जिनकी संख्या 11 है।

स्वर के भेद-उच्चारण में लगने वाले समय के आधार पर इन्हें तीन वर्गों में बाँटा जा सकता है –
(अ) ह्रस्व स्वर-जिस स्वरों के उच्चारण में कम समय लगता है, उन्हें ‘ह्रस्व स्वर’ कहते हैं। इनकी संख्या 4 है। ये स्वर हैं – अ, इ, उ तथा ऋ।
(ब) दीर्घ स्वर-जिन स्वरों के उच्चारण में ह्रस्व स्वरों के उच्चारण में लगने वाले समय का दुगुना समय लगता है, उन्हें ‘दीर्घ
स्वर’ कहते हैं। इनकी संख्या 7 है। ये स्वर हैं-आ, ई, ऊ, ए, ऐ, ओ तथा औ।
(स) प्लत स्वर-जिन स्वरों के उच्चारण में दीर्घ स्वरों के उच्चारण में लगने वाले समय का दुगुना समय लगता है, उन्हें ‘प्लुत
स्वर’ कहते हैं। इनका प्रयोग प्रायः किसी को पुकारने के लिए किया जाता है; जैसे-ओम्।
CBSE Class 9 Hindi B व्याकरण वर्ण-विच्छेद - 3
CBSE Class 9 Hindi B व्याकरण वर्ण-विच्छेद - 4

2. व्यंजन-जिन वर्णों का उच्चारण स्वर की मदद से किया जाता है तथा जिनके उच्चारण में वायु मुँह के विभिन्न अवयवों से रगड़ खाकर निकलती है, उन्हें व्यंजन कहते हैं।

उच्चारण स्थान के आधार पर व्यंजन के तीन भेद होते हैं –
(अ) स्पर्श व्यंजन-जिन वर्णों के उच्चारण में जिह्वा (जीभ) मुख के विभिन्न भागों को स्पर्श करती हुई निकलती है, उन्हें स्पर्श व्यंजन कहते हैं। इनकी संख्या 25 है। प्रत्येक वर्ग के पहले वर्ण के नाम पर इन्हें पाँच वर्गों में बाँटा गया है –
CBSE Class 9 Hindi B व्याकरण वर्ण-विच्छेद - 5

(ब) अंतस्थ व्यंजन-जिन वर्णों का उच्चारण स्वर और व्यंजन वर्गों के मध्य का-सा लगता है, उन्हें अंतस्थ व्यंजन कहते हैं।
य, र्, ल् और व् अंतस्थ व्यंजन हैं। इनकी संख्या चार है।

(स) ऊष्म व्यंजन-जिन वर्गों के उच्चारण में ऊष्म (गरम) वायु मुँह से बाहर निकलती है, उन्हें ऊष्म व्यंजन कहते हैं। श,
छ, स् और ह ऊष्म व्यंजन हैं। इकी संख्या चार है।

CBSE Class 9 Hindi B व्याकरण वर्ण-विच्छेद - 5(i)
कुछ स्थितियों को छोड़कर पंचमाक्षर के स्थान पर इसका प्रयोग किया जाता है; जैसे-चंदन, बंदर, कंगन आदि।
CBSE Class 9 Hindi B व्याकरण वर्ण-विच्छेद - 5(ii)
जब स्वरों का उच्चारण नाक तथा मुख दोनों से एक साथ होता है; जैसे-आँख, गाँव, पाँव, ठाँव आदि।
ध्यान दें- जब शिरोरेखा के ऊपर मात्रा होती है तो इसका प्रयोग अनुस्वार जैसा ही किया जाता है; जैसे – मैं, गोंद, हैं आदि।

व्यंजन द्वित्व और संयुक्ताक्षर :

संयक्त व्यंजन- जब दो या दो से अधिक व्यंजनों का मेल होता है, तो उसे संयुक्त व्यंजन कहते हैं।
क्ष =क् + ष
त्र = त् + र
ज्ञ = ज् + ञ
श्र = श् + र ।

व्यंजन द्वित्व – जब एक व्यंजन ध्वनि अपने समान ही अन्य व्यंजन ध्वनि से जुड़ती है, तो उसे व्यंजन द्वित्व या द्वित्व व्यंजन कहते हैं; जैसे –
च् + च = च्च = बच्चा
ट् + ट = ट्ट = लटू
द् + द = द्द = कद्दू
त् + त = त्त = पत्ता

संयुक्ताक्षर बनाने के नियम –

(i) खड़ी पाई वाले वर्णों की पाई हटाकर –
CBSE Class 9 Hindi B व्याकरण वर्ण-विच्छेद - 6

(ii)
CBSE Class 9 Hindi B व्याकरण वर्ण-विच्छेद - 6(i)
क्वार = (क + वा = क्वा)
पक्की = (क + की = क्की)
रफ़्तार= प + ता = फ्ता
हफ्ता= प + ता = फ्ता

(iii) CBSE Class 9 Hindi B व्याकरण वर्ण-विच्छेद - 6(ii)
जैसे –
वाङ्मय = ङ् + म
लड्डू = ड् + डू
लटू = ट् + टू
गड्ढा = ड् + ढा
कंठ्य = ठ् + य
असह्य = ह् + य

शब्द का वर्ण-विच्छेद :

किसी शब्द या ध्वनि के समूह के वर्गों को अलग-अलग लिखना वर्ण-विच्छेद कहलाता है। आइए, वर्ण-विच्छेद के कुछ उदाहरण देखते हैं –
सड़क = स् + अ + डू + अ + क् + अ
भक्त = भ् + अ + क् + त् + अ
महात्मा = म् + अ + ह् + आ + त् + म् + आ
कविता = क् + अ + व् + इ + त् + आ
प्रयोग = प + र् + अ + य् + ओ + ग् + अ
अद्भुत = अ + द् + भ् + उ + त् + अ
कलम = क् + अ + ल् + अ + म् + अ
पाठशाला = प् + आ + ठ् + अ + श् + आ + ल् + आ
आराधना = आ + र् + आ + ध् + अ + न् + आ
प्राकृतिक = प् + र् + आ + क् + ऋ + त + इ + क् + अ
सर्वमान्य = स् + अ + र् + व् + अ + म् + आ + न् + य् + अ
अर्जुन = अ + र् + ज् + उ + न् + अ
परिश्रम = प् + अ + र + इ + श् + र् + अ + म् + अ
क्षत्रिय = क् + ष् + अ + त् + र् + इ + य् + अ
परिक्रमा = प् + अ + र् + इ + क् + र् + अ + म् + आ
क्षमा = क् + ष् + अ + म् + आ
विज्ञान = व् + इ + ज् + ञ् + आ + न् + अ
गुरुद्वारा = ग् + उ + र् + उ + द् + व् + आ + र् + आ

आइए इन्हें भी जानें –

वर्ण-विच्छेद करते समय निम्नलिखित बातों को जानना आवश्यक है –

(i) ‘रि’ और ‘ऋ’ के रूप –
परिचय = प् + अ + र् + इ + च् + अ + य् + अ
रिषभ = र् + इ + ष् + अ + भ् + अ ।
पृथक = प् + ऋ + थ् + अ + क् + अ
कृपा = क् + ऋ + प् + आ

(ii) ‘र’ और ‘ ‘ तथा ‘ ‘ की मात्राएँ –
रुस्तम = र् + उ + स् + त् + अ + म् + अ
रुपया = र् + उ + प् + अ + य् + आ
रूपा = र् + ऊ + प् + आ
रूठना = र् + ऊ + ठ् + अ + न् + आ

(iii) ‘ह’ के विभिन्न संयुक्त –
रूपह्रस्व = ह् + र + अ + स् + व् + अ
हृदय = ह् + ऋ + द् + अ + य् + अ
प्रह्लाद = प् + र् + अ + ह् + ल् + आ + द् + अ
चिह्न = च् + इ + ह् + न् + अ

(iv) ‘द’ के संयुक्त –
रूपद्वार = द् + व् + आ + र् + अ
द्रव्य = द् + र् + अ + व् + य् + अ
उद्देश्य = उ + द् + द् + ए + श् + य् + अ
विरुद्ध = व् + इ + र् + उ + द् + ध् + अ

(v) CBSE Class 9 Hindi B व्याकरण वर्ण-विच्छेद - 6(iV)

(vi)
CBSE Class 9 Hindi B व्याकरण वर्ण-विच्छेद - 6(iii)
अंगूर = अं + ग् + ऊ + र् + अ
गंगा = ग् + अ + ङ् + ग् + आ
कंबल = क् + अं(अ + म्) + ब् + अ + ल् + अ
पम्प = प् + अ + म् + प् + अ
चंद्र = च् + अं(अ + न्) + द् + र् + अ
ठंडक = ठ् + अं + ड् + अ + क् + अ
चंदन = च् + अं(अ + न्) + द् + अ + न् + अ
चंचु = च् + अं(अ + ञ्) + च् + उ
ठंडा = ठ् + अं(अ + ण्) + ड् + आ

वर्ण-विच्छेद के उदाहरण –

अंग = अं + ग् + अ
अंधा = अं+ ध् + आ
अंग्रेज़ = अं+ ग् + र् + ए + ज् + अ
अंगार = अँ + ग् + आ + र
अचला _ = अ + च् + अ + ल् + आ
अधुना = अ + ध् + उ + न् + आ
अक्षर = अ + क् + ष् + अ + र् + अ
अवश्य = अ + व् + अ + श् + य् + अ
अग्नि = अ + ग् + न् + इ
अमृत = अ + म् + ऋ + त् + अ
अप्रतिभ = अ + प् + र् + अ + त् + इ + भ् + अ
अभ्यागत = अ + भ् + य् + आ + ग् + अ + त् + अ
आश्रम = आ + श् + र् + अ + म् + अ
इज़्ज़त = इ + ज् + ज् + अ + त् + अ
इस्तेमाल = इ + स् + त् + ए + म् + आ + ल् + अ
उक्ति = उ + क् + त् + इ
उच्चारण = उ + च् + च् + आ + र् + अ + ण् + अ
ऋचा = ऋ+ च् + आ
एकाग्र = ए + क् + आ + ग् + र् + अ
एकाक्षर = ए + क् + आ + क् + ष् + अ + र् + अ
औषधि = औ + ष् + अ + ध् + इ
कंगारू = क् + अं+ ग् + आ + र् + ऊ
कन्हाई = क् + अ + न् + ह् + आ + ई
कुशाग्र = क् + उ + श् + आ + ग् + र् + अ
अव्वल = अ + व् + व् + अ + ल् + अ
आकार = आ + क् + आ + र् + अ
आख्यान = आ + ख् + य् + आ + न् + अ
आक्रांत = आ + क् + र् + आ + न् + त् + अ
आजीवन = आ + ज् + ई + व् + अ + न् + अ
आज्ञा = आ + ज् + ञ् + आ
आधार = आ + ध् + आ + र् + अ
आपूर्ति = आ + प् + ऊ + र् + त् + इ
आरूढ़ = आ + र् + ऊ + द + अ
गृहस्थ = ग् + ऋ + ह् + अ + स् + थ् + अ
ग्राहक = ग् + र् + आ + ह् + अ + क् + अ
ग्राह्य = ग् + र् + आ + ह् + य् + अ
घृणित = घ् + ऋ + ण् + इ + त् + अ
घंटी = घ् + अं+ ट् + ई
चकोर = च् + अ + क् + ओ + र् + अ
चक्कर = च् + अ + क् + क् + अ + र् + अ
चक्र = च् + अ + क् + र् + अ
चतुर्थ = च् + अ + त् + उ + र् + थ् + अ
चाँदनी = च् + आँ + द् + अ + न् + ई ।
चक्षु = च् + अ + क् + ष् + उ
चित्रित = च् + इ + त् + र् + इ + त् + अ
चिह्नित = च् + इ + ह् + न् + इ + त् + अ
ज़ख्मी = ज् + अ + ख् + म् + ई
जीवन = ज् + ई + व् + अ + न् + अ
जुर्माना = ज् + उ + र् + म् + आ + न् + आ
जागृति = ज् + आ + ग् + ऋ + त् + इ
जिज्ञासा = ज् + इ + ज् + ञ् + आ + स् + आ
जिह्वा = ज् + इ + ह् + व् + आ
जुझारू = ज् + उ + झ् + आ + र् + ऊ
ज्ञापित _ = ज् + ञ् + आ + प् + इ + त् + अ
ज्योत्स्ना = ज् + य् + ओ + त् + स् + न् + आ
झंडा = झ् + अं+ ड् + आ
टिप्पणी = ट् + इ + प् + प् + अ + ण + ई
ठाकुर = ठ् + आ + क् + उ + र् + आ
ढूँढना = द् + ऊ + * + द + अ + न् + अ
तांत्रिक = त् + आ + अं+ त् + र् + इ + क् + अ
त्रुटि = त् + र् + उ + ट् + इ
त्वरित = त् + व् + अ + र् + इ + त् + अ
तालाब = त् + आ + ल् + आ + ब् + अ
कौतुक = क् + औ + त् + उ + क् + अ
क्रय = क् + र् + अ + य् + अ
कृत्रिम = क् + ऋ + त् + र् + इ + म् + अ
क्रोध = क् + र् + ओ + ध् + अ
क्ले श = क् + ल् + ए + श् + अ
खट्टा = ख् + अ + ट् + ट् + आ
ख्याति = ख + य् + आ + त् + इ
गाँठ = ग् + आँ + ठ् + अ
गद्य = ग् + अ + द् + य् + अ
त्रिभुज = त् + र + इ + भ् + उ + ज् + अ
तृष्णा = त् + ऋ + ष् + ण् + आ
दफ़्तर = द् + अ + फ़् + त् + अ + र् + अ
देवत्व = द् + ए + व् + अ + त् + व् + अ
दंभी = द् + अं+ भ् + ई
दृष्टि = द् + ऋ + ष् + ट् + इ
द्रवित = द् + र् + अ + व् + इ + त् + अ
दैनंदिनी = द् + ऐ + न् + अं + द् + इ + न् + ई
द्युति = द् + य् + उ + त् + इ
द्वापर = द् + व् + आ + प् + अ + र् + अ
द्वितीया = द् + व + इ + त् + ई + य् + आ
दविज = द् + व् + इ + ज् + अ
द्वैत = द् + व् + ऐ + त् + अ
ध्वजा = ध् + व् + अ + ज् + आ
ध्वस्त = ध् + व् + अ + स् + त् + अ
नंदिनी = न् + अं + द् + इ + न् + ई
नक्काशी = न् + अ + क् + क् + आ + श् + ई
निरुपम = न् + इ + र् + उ + प् + अ + म् + अ
नास्तिक = न् + आ + स् + त् + इ + क् + अ
निस्तब्ध = न् + इ + स् + त् + अ + ब् + ध् + अ
नृत्य = न् + ऋ + त् + य् + अ
पुष्प = प् + उ + ष् + प् + अ
प्रधान = प् + र् + अ + ध् + आ + न् + अ
प्राच्य = प् + र् + आ + च् + य् + अ
पृथ्वी = प् + ऋ + थ् + व् + ई
फलदार = फ् + अ + ल् + अ + द् + आ + र् + अ
फिक्र = फ् + इ + क् + र् + अ
बाँस = ब् + आँ + स् + अ
बृहद = ब् + ऋ + ह् + अ + द् + अ
ब्रह्मा = ब् + र् + अ + ह् + म् + आ
लक्षण = ल् + अ + क् + ष् + अ + ण् + अ
वत्सल = व् + अ + त् + स् + अ + ल् + अ
विरुद्ध = व् + इ + र् + उ + द् + ध् + अ
व्योम = व् + य् + ओ + म् + अ
व्रत = व् + र् + अ + त् + अ
शक्ति = श् + अ + क् + त् + इ
सृष्टि = स् + ऋ + ष् + ट् + इ
हार्दिक = ह् + आ + र् + द् + इ + क् + अ
ब्रह्मर्षि = ब् + र् + अ + ह् + म् + अ + र् + ष् + इ
भेद्य = भ् + ए + द् + य् + अ
मंगल = म् + अं+ ग् + अ + ल् + अ
मित्र = म् + इ + त् + र् + अ
मौक्तिक = म् + औ + क् + त् + इ + क् + अ
याचक = य् + आ + च् + अ + क् + अ
रूहानी = र् + ऊ + ह् + आ + न् + ई

पाठ्यपुस्तक के गद्य पाठों से लिए गए कुछ प्रमुख शब्द और उनका वर्ण-विच्छेद –

धूल :

कविता = क् + अ + व् + इ + त् + आ
श्रृंगार = श् + र् + ऋ + अं+ ग् + आ + र् + अ
आविष्कार = आ + व् + इ + ष् + क् + आ + र् + अ
संसर्ग = स् + अं+ स् + अ + र् + ग् + अ
बुद्धि = ब् + उ + द् + ध् + इ
स्पर्श = स् + प् + अ + र् + श् + अ
विडंबना = व् + इ + ड् + अं (अ + म्) + ब् + अ + न् + आ
व्यंजनाएँ = व् + य् + अं( + अ + न्) + ज् + अ + न् + आ + ए + अँ
चकाचौंध = च् + अ + क् + आ + च् + औ + अ + ध् + अ
पार्थिवता – प् + आ + र् + थ् + इ + व् + अ + त् + आ
वास्तविकता = व् + आ + स् + त् + अ + व् + इ + क् + त् + आ
बड़प्पन = ब् + अ + ड़ + अ + प् + प् + अ + न् + अ
निरवंद्व = न् + इ + र् + द् + व् + अं + द् + व् + अ
सूर्यास्त = स् + ऊ + र् + य् + आ + स् + त् + अ

दुख का अधिकार :

श्रेणियाँ = श् + र् + ए + ण् + इ + य् + आँ
निश्चित = न् + इ + श् + च् + इ + त् + अ
फुटपाथ = फ् + उ + ट् + अ + प् + आ + थ् + अ
दक्षिणा = द् + अ + क् + ष् + इ + ण् + आ
दुअन्नी = द् + उ + अ + न् + न् + ई
संभ्रांत = स् + अं(अ + म्) + भ् + र + आ + अं + त् + अ
सहूलियत = स् + अ + ह् + ऊ + ल् + इ + य् + अ + त् + अ
परिस्थिति = प् + अ + र् + इ + स् + थ् + इ + त् + इ
व्यवधान = व् + य् + अ + व् + अ + ध् + आ + न् + अ
विश्राम = व् + इ + श् + र् + आ + म् + अ
सर्प = स् + अ + र् + प् + अ
वियोगिनी = व् + इ + य् + ओ + ग् + इ + न् + ई
द्रवित = द् + र् + अ + व् + इ + त

एवरेस्ट-मेरी शिखर यात्रा :

एवरेस्ट = ए + व् + अ + र् + ए + स् + ट् + अ
अग्रिम = अ + ग् + र् + इ + म् + अ
सर्वप्रथम = स् + अ + र् + व् + अ + प् + र् + अ + थ्
अधिकांश = अ + ध् + इ + क् + आ + अं+ श् + अ
विचित्र = व् + इ + च् + इ + त् + र् + अ
ल्होत्से = ल् + ह् + ओ + त् + स् + ए
नेतृत्व = न् + ए + त् + ऋ + त् + व् + अ
ग्लेशियर = ग् + ल् + ए + श् + इ + य् + अ + र् + अ
एक्सप्रेस = ए + क् + स् + अ + प् + र् + ए + स् + अ
काठमांडू = क् + आ + ल् + अ + म् + आ + अं+ ड् + ऊ
दुर्गम = द् + उ + र् + ग् + अ + म् + अ
प्रसिद्ध = प् + र् + अ + स् + इ + द् + ध् + अ + अ + म् + अ
दृश्य = द् + ऋ + श् + य् + अ ।
दक्षिण = द् + अ + क् + ष् + इ + ण् + अ
आकर्षित = आ + क् + अ + र् + ष् + इ + त् + अ
मृत्यु = म् + ऋ + त् + य् + उ
झंडियाँ = झ् + अं + ड् + इ + य् + आ + अँ
विस्तृत = व् + इ + स् + त् + ऋ + त् + अ
सुरक्षा = स् + उ + र् + अ + क् + ष् + आ
कृतज्ञता = क् + ऋ + त् + अ + ज् + ञ् + अ + त् + आ
क्षमता = क् + ष् + अं + म् + अ + त् + आ
दृढ़ता .. = द् + ऋ + ढ़ + अ + त् + आ
सुरक्षित = स् + उ + र् + अ + क् + ष् + इ + त् + अ

तुम कब जाओगे, अतिथि :

चतुर्थ = च् + अ + त् + उ + र् + थ् + अ
नम्रता = न् + अ + म् + र् + अ + त् + आ
आग्रह = आ + ग् + र् + अ + ह् + अ
निर्मूल = न् + इ + र् + म् + ऊ + ल् + अ
संक्रमण = स् + अं+ क् + र् + अ + म् + अ + ण् + अ
धुआँ = ध् + उ + आ + अँ
अज्ञात = अ + ज् + ञ् + आ + त् + अ
मार्मिक = म् + आ + र् + म् + इ + क् + अ
रूपांतरित = र् + ऊ + प् + आ + अं+ त् + अ + र् + इ + त् + अ
सुरक्षित = स् + उ + र् + अ + क् + ष् + इ + त् + अ

वैज्ञानिक चेतना के वाहक चंद्रशेखर वेंकट रामन :

असंख्य = अ + स् + अं+ ख् + य् + अ
समक्ष = स् + अ + म् + अ + क् + ष् + अ
जिज्ञासा = ज् + इ + ज् + ञ् + आ + स् + आ
तिरुचिरापल्ली = त्+ इ + र् + उ + च् + इ + र् + आ + प् + अ + ल् + ल् + ई
शिक्षक = श् + इ + क् + ष् + अ + क + अ
अतिशयोक्ति = अ + त् + इ + श् + अ + य् + ओ + क् + त् + इ
परीक्षा = प् + अ + र् + ई + क् + ष् + आ
समर्पित = स् + अ + म् + अ + र् + प् + इ + त् + अ
समृद्ध = स् + अ + म् + ऋ + द् + ध् + अ
वाद्ययंत्र = व् + आ + द् + य् + अ + य् + अं+ त् + र् + अ
शिक्षाशास्त्री = श् + इ + क् + ष् + आ + श् + आ + स् + त् + र् + ई
सृजित = स् + ऋ + ज् + इ + त् + अ ।
प्रत्यक्ष = प् + र + अ + त् + य् + अ + क् + ष् + अ
कृत्रिम _ = क् + ऋ + त् + र् + इ + म् + अ
आह्लादित = आ + ह् + ल् + आ + द् + इ + त् + अ

कीचड़ का काव्य :

आकर्षण = आ + क् + अ + र् + ष् + अ + ण् + अ
यथार्थ = य् + अ + थ् + आ + र् + थ् + अ
पचिह्न = प् + अ + द् + अ + च् + इ + ह् + न् + अ
घृणास्पद = घ् + ऋ + ण् + आ + स् + प् + अ + द् + अ
श्वेत = श् + व् + ए + त् + अ
सौंदर्य = स् + औ + अं(अ + न्) + द् + अ + र् + य् + अ
तृप्ति = त् + ऋ + प् + त् + इ
मातुश्री = म् + आ + त् + उ + श् + र् + ई

धर्म की आड़ :

दुरुपयोग = द् + उ + र् + उ + प् + अ + य् + ओ + ग् + अ
अट्टालिका = अ + ट् + ट् + आ + ल् + इ + क् + आ
स्वार्थ = स् + व् + आ + र् + थ् + अ
विरुद्ध = व् + इ + र् + उ + द् + ध् + अ
प्रपंच = प् + र् + अ + प् + अं+ च् + अ
शुद्धाचरण = श् + उ + द् + ध् + आ + च् + अ + र् + अ + ण् + अ
अपवित्र = अ + प् + अ + व् + इ + त् + र् + अ
मनुष्यत्व = म् + अ + न् + उ + ष् + य् + अ + त् + व् + अ

शुक्रतारे के समान :

नक्षत्र = न् + अ + क् + ष् + अ + त् + र् + अ
संक्षिप्त = स् + अं+ क् + ष् + इ + प् + त् + अ
द्वारकादास = द् + व् + आ + र् + अ + क् + आ + द् + आ + स् + अ
विद्यार्थी = व् + इ + द् + य् + आ + र् + थ् + ई
मंत्रमुग्ध = म् + अं+ त् + र् + अ + म् + उ + ग् + ध् + अ
भर्तृहरि = भ् + अ + र् + त् + ऋ + ह् + अ + र् + इ
स्वतंत्रता = स् + व् + अ + त् + अं + त् + र् + अ + त् + आ
संपादक = स + अं+ प् + आ + द् + अ + क् + अ
आश्रम = आ + श् + र् + अ + म् + अ
भ्रमण = भ् + र् + अ + म् + अ + ण् + अ
शीर्षक = श् + ई + र् + ष् + अ + क् + अ
अद्यतन = अ + द् + य् + अ + त् + अ + न् + अ

अभ्यास प्रश्न

प्रश्नः 1.
निम्नलिखित शब्दों के वर्ण-विच्छेद कीजिए –

  1. असंख्य
  2. रहस्य
  3. प्रकृति
  4. भावुक
  5. जिज्ञासा
  6. प्रतिभा
  7. रुझान
  8. फुर्सत
  9. नितांत
  10. समृद्ध
  11. आकृष्ट
  12. सृजित
  13. प्रसिद्ध
  14. अक्षुण्ण
  15. आकर्षक
  16. अंकित
  17. अल्पोक्ति
  18. आह्लाद
  19. उत्पाद
  20. स्वार्थ
  21. प्रपंच
  22. नक्षत्र
  23. अग्रगण्य
  24. अद्यतन

उत्तरः

  1. अ + स् + अं+ ख् + य् + अ
  2. र् + अ + ह् + अ + स् + य् + अ
  3. प् + र् + अ + क् + ऋ + त् + इ
  4. भ् + आ + व् + उ + क् + अ
  5. ज् + इ + ज् + ञ् + आ + स् + आ
  6. प् + र् + अ + त् + इ + भ् + आ
  7. र् + उ + इ + आ + न् + अ
  8. फ् + उ + र् + स् + अ + त् + अ
  9. न् + इ + त् + आ + अं + त् + अ
  10. स् + अ + म् + ऋ + द् + ध् + अ
  11. आ + क् + ऋ + ष् + ट् + अ
  12. स् + ऋ + ज् + इ + त् + अ
  13. प् + र् + अ + स् + इ + द् + ध् + अ
  14. अ + क् + ष् + उ + ण् + ण् + अ
  15. आ + क् + अ + र् + ष् + अ + ण् + अ
  16. अं+ क् + इ + त् + अ
  17. अ + ल् + प् + ओ + क् + त् + इ
  18. आ + ह् + ल् + आ + द् + अ
  19. उ + त् + प् + आ + द् + अ
  20. स् + व् + आ + र् + थ् + अ
  21. प् + र् + अ + प् + अं + च् + अ
  22. न् + अ + क् + ष् + अ + त् + र् + अ
  23. अ + ग् + र् + अ + ग् + अ + ण् + य् + अ
  24. अ + द् + य् + अ + त् + अ + न् + अ

प्रश्नः 2.
निम्नलिखित वर्ण-विच्छेदों के लिए शब्द लिखिए

  1. अ + भ् + इ + ज् + आ + त् + अ
  2. प् + र् + अ + स् + आ + ध् + अ + न् + अ
  3. स् + अं + स् + अ + र् + ग् + अ
  4. अ + स् + आ + र् + अ + त् + आ
  5. व् + य् + अ + ज् + अ + न् + आ
  6. य् + अ + थ् + आ + र् + थ् + अ + व् + आ + द् + ई
  7. अ + न् + उ + भ् + ऊ + त् + इ
  8. व् + य् + अ + व् + अ + ध् + आ + न् + अ
  9. न् + इ + र् + व् + आ + ह् + अ
  10. आ + क् + अ + र् + ष् + इ + त् + अ
  11. स् + औ + ह् + आ + र् + द् + अ
  12. स् + अं+ क् + र् + अ + म् + अ + ण् + अ
  13. श् + ओ + ध् + अ + क् + आ + र् + य् + अ
  14. व् + ऐ + ज् + ञ् + आ + न् + इ + क् + अ

उत्तरः

  1. अभिजात
  2. प्रसाधन
  3. संसर्ग
  4. असारता
  5. व्यंजना
  6. यथार्थवाची
  7. अनुभूति
  8. व्यवधान
  9. निर्वाह
  10. आकर्षित
  11. सौहार्द
  12. संक्रमण
  13. शोधकार्य
  14. वैज्ञानिक

विभिन्न परीक्षाओं में पूछे गए प्रश्न

निम्नलिखित शब्दों का वर्ण-विच्छेद कीजिए –

  1. देशभक्ति, काव्य
  2. हिंदुस्तान, आर्यावर्त
  3. न्याय, वृद्धा
  4. प्रायोगिक, मातुश्री
  5. प्रस्तुति, क्षेत्रफल
  6. संदिग्ध, श्रेणियाँ
  7. प्रकृति, दिल्ली
  8. विज्ञापित, क्रमित
  9. संपादन, वैज्ञानिक
  10. विच्छेद, समाहार
  11. पार्थिव, विक्रेता
  12. मनुष्यता, राजनीति
  13. विस्तृत श्रमसाध्य
  14. वृष्टि परिभाषा
  15. मक्खन, सूर्योदय
  16. गोधूलि, कृष्ण
  17. विद्यालय, शिक्षक
  18. रूपांतरित, घृणा
  19. संस्कृति, कलाभिज्ञ
  20. कला, नेतृत्व
  21. आविष्कार, व्यंजित
  22. स्वर्ग, सभ्यता
  23. शिक्षाशास्त्री, सहानुभूति
  24. स्त्री, नक्षत्र
  25. नास्तिक श्रद्धा
  26. हत्याकांड वैज्ञानिक

उत्तरः

  1. द् + ए + श् + अ + भ् + अ + क् + त् + इ
  2. ह् + इ + न् + द् + उ + स् + त् + आ + न् + अ क् + आ + व् + य् + अ
    आ + र् + य् + आ + व् + अ + र् + त् + अ
  3. न् + य् + आ + य् + अ
  4. प् + र् + आ + य् + ओ + ग् + इ + क् + अ . व् + ऋ + द् + ध् + अ
    म् + आ + त् + उ + श् + र् + ई
  5. प् + र् + अ + स् + त् + उ + त् + इ
  6. स् + अं + द् + इ + ग् + ध् + अ क् + ष् + ए + त् + र् + अ + फ् + अ + ल् + अ
    श् + र् + ए + ण् + इ + य् + आ + अँ
  7. प् + र् + अ + क् + ऋ + त् + इ
  8. व् + इ + ज् + ञ् + आ + प् + इ + त् + अ द् + इ + ल् + ल् + ई
    क् + र् + अ + म् + इ + त् + अ
  9. स + अं(अ + म्)प् + आ + द् + अ + न् + अ
  10. व् + इ + च् + छ् + ए + द् + अ व् + ऐ + ज् + ञ् + आ + न् + इ + क् + अ
    स् + अ + म् + आ + ह् + आ + र् + अ
  11. प् + आ + र् + थ् + इ + व् + अ
  12. म् + अ + न् + उ + ष् + य् + अ + त् + व् + अ व् + इ + क् + र् + ए + त् + आ
    र् + आ + ज् + अ + न् + ई + त् + इ
  13. व् + इ + स् + त् + ऋ + त् + अ
  14. व् + ऋ + ष् + ट् + इ श् + र् + अ + म् + अ + स् + आ + ध् + य् + अ
    प् + अ + र + इ + य् + आ + ष् + आ
  15. म् + अ + क् + ख् + अ + न् + अ
  16. ग् + ओ + ध् + ऊ + ल् + इ स् + ऊ + र् + य् + ओ + द् + अ + य् + अ
    क् + ऋ + ष् + ण् + अ
  17. व् + इ + द् + य + आ + ल् + अ + य् + अ
  18. र् + ऊ + प् + आ + अं+ त् + अ + र् + इ + त् + अ श् + इ + क् + ष + अ + क् + अ
    घ् + ऋ + ण् + आ
  19. स् + अं+ स् + क् + ऋ + त् + इ
  20. क् + अ + ल् + आ क् + अ + ल् + आ + भ् + इ + ज् + ञ् + अ
    न् + ए + त् + ऋ + त् + व् + अ
  21. आ + व् + इ + ष् + क् + आ + र् + अ
  22. स् + व् + अ + र् + ग् + अ व् + य् + अं+ ज् + इ + त् + अ
    स् + अ + भ् + य् + अ + त् + आ
  23. श् + इ + क् + ष् + आ + श् + आ + स् + त् + र् + ई ..
    स् + अ + ह् + आ + न् + उ + भ् + ऊ + त् + इ
  24. स् + त् + र् + ई
    न् + अ + क् + ष् + अ + त् + र् + अ
  25. न् + आ + स् + त् + इ + क् + अ
    श् + र् + अ + द् + ध् + आ
  26. ह् + अ + त् + य् + आ + क् + आ + अं+ इ + अ
    ब् + ऐ + ज् + ञ् + आ + न् + इ + क् + अ

NCERT Solutions for Class 9 Hindi

The post CBSE Class 9 Hindi B व्याकरण वर्ण-विच्छेद appeared first on Learn CBSE.

CBSE Class 9 Hindi B व्याकरण अनुस्वार एवं अनुनासिक

$
0
0

CBSE Class 9 Hindi B व्याकरण अनुस्वार एवं अनुनासिक

CBSE Class 9 Hindi B व्याकरण अनुस्वार एवं अनुनासिक 13

अनुस्वार के उच्चारण में ‘अं’ की ध्वनि मुख से निकलती है। हिंदी में लिखते समय इसका प्रयोग शिरोरेखा के ऊपर बिंदु लगाकर किया जाता है। इसका प्रयोग ‘अ’ जैसे किसी स्वर की सहायता से ही संभव हो सकता है; जैसे – संभव।
इसका वर्ण-विच्छेद करने पर ‘स् + अं(अ + म्) + भ् + अ + व् + अ’ वर्ण मिलते हैं। इस शब्द में अनुस्वार ‘अं’ का उच्चारण (अ + म्) जैसा हुआ है, पर अलग-अलग शब्दों में इसका रूप बदल जाता है; जैसे
संचरण = स् + अं(अ + न्) + च् + अ + र् + अ + ण् + अ
संभव = स् + अं(अ + म्) + भ् + अ + व् + अ ।
संघर्ष = स् + अं(अ + ङ्) + घ् + अ + र् + ष् + अ
संचयन = स् + अं(अ + न्) + च् + अ + य् + अ + न् + अ

अनुस्वार प्रयोग के कुछ नियम
अनुस्वार के प्रयोग के निम्नलिखित नियम हैं-

(i) पंचमाक्षर का नियम – जब किसी वर्ण से पहले अपने ही वर्ग का पाँचवाँ वर्ग (पंचमाक्षर) आए तो उसके स्थान पर अनुस्वार का प्रयोग होता है; जैसे –
गङ्गा = गंगा,
ठण्डा = ठंडा,
सम्बन्ध = संबंध,
अन्त = अंत आदि।

(ii) य, र, ल, व (अंतस्थ व्यंजनों) और श, ष, स, ह (ऊष्म व्यंजनों) से पूर्व यदि पंचमाक्षर आए, तो अनुस्वार का ही प्रयोग किया जाता है; जैसे –
सन्सार = संसार,
सरक्षक = संरक्षक,
सन्शय = संशय आदि।

ध्यान दें – हिंदी को सरल बनाने के उद्देश्य से भिन्न-भिन्न नासिक्य ध्वनियों (ङ, ञ, ण, न और म) की जगह बिंदु का प्रयोग किया जाए। संस्कृत में इनका वही रूप बना रहेगा।

संस्कृत में – अङ्क, चञ्चल, ठण्डक, चन्दन, कम्बल।

हिंदी में – अंक, चंचल, ठंडक, चंदन, कंबल।

अनुस्वार का प्रयोग कब न करें-
निम्नलिखित स्थितियों में अनुस्वार का प्रयोग नहीं करना चाहिए-

(i)
CBSE Class 9 Hindi B व्याकरण अनुस्वार एवं अनुनासिक 14
CBSE Class 9 Hindi B व्याकरण अनुस्वार एवं अनुनासिक 1

(ii) यदि अनुस्वार के पश्चात् कोई पंचमाक्षर (ङ, ञ, ण, न, म) आता है, तो अनुस्वार का प्रयोग मूलरूप में किया जाता है। अनुस्वार का बिंदु रूप अस्वीकृत होता है; जैसे –
CBSE Class 9 Hindi B व्याकरण अनुस्वार एवं अनुनासिक 2

CBSE Class 9 Hindi B व्याकरण अनुस्वार एवं अनुनासिक 15
सम्+हार = संहार
सम्+सार = संसार
सम्+चय = संचय
सम्+देह = संदेह
सम्+ चार = संचार
सम्+भावना = संभावना
सम्+कल्प = संकल्प
सम्+जीवनी = संजीवनी

अनुनासिक

CBSE Class 9 Hindi B व्याकरण अनुस्वार एवं अनुनासिक 16
ध्यान दें- अनुनासिक की जगह अनुस्वार और अनुस्वार की जगह अनुनासिक के प्रयोग से शब्दों के अर्थ में अंतर आ जाता है,जैसे –
हँस (हँसने की क्रिया)
हंस (एक पक्षी)।
CBSE Class 9 Hindi B व्याकरण अनुस्वार एवं अनुनासिक 17
हैं = ह + एँ
मैं = म् + एँ
में = म् + एँ
कहीं = क् + अ + ह् + ईं
गोंद = ग् + ओं + द् + अ
भौंकना = भ् + औं + क् + अ + न् + आ
पोंगल = प् + औं + ग् + अ + ल् + अ
जोंक = ज् + औं + क् + अ
शिरोरेखा के ऊपर मात्रा न होने पर इसे चंद्रबिंदु के रूप में ही लिखा जाता है; जैसे-आँगन, आँख, कुँआरा, चूंट आदि।

यह भी जानें-
अर्धचंद्राकार और अनुनासिक में अंतर-

हिंदी भाषा में अंग्रेज़ी के बहुत-से शब्द प्रयोग होते हैं। इनको बोलते समय इनकी ध्वनि ‘आ’ और ‘ओ’ के बीच की निकलती है। इसे दर्शाने के लिए अर्धचंद्राकार लगाया जाता है; जैसे-डॉक्टर, ऑफिस, कॉलेज आदि। इन शब्दों की ध्वनियाँ क्रमशः ‘डा और डो’, ‘आ और ओ’, ‘का और को’ के मध्य की हैं। इनके उच्चारण के समय मुँह आधा खुला रहता है। आगत भी कहा जाता है। ध्यान रहे कि अर्धचंद्राकार का प्रयोग अंग्रेजी शब्दों के लिए होता है जबकि अनुनासिक हिंदी की ध्वनि है।

CBSE Class 9 Hindi B व्याकरण अनुस्वार एवं अनुनासिक 18

उदाहरण
CBSE Class 9 Hindi B व्याकरण अनुस्वार एवं अनुनासिक 3

पाठ्यपुस्तक के पाठों पर आधारित शब्दों में अनुस्वार/अनुनासिक का प्रयोग

CBSE Class 9 Hindi B व्याकरण अनुस्वार एवं अनुनासिक 4
CBSE Class 9 Hindi B व्याकरण अनुस्वार एवं अनुनासिक 5

दुख का अधिकार
CBSE Class 9 Hindi B व्याकरण अनुस्वार एवं अनुनासिक 6

एवरेस्ट-मेरी शिखर यात्रा
CBSE Class 9 Hindi B व्याकरण अनुस्वार एवं अनुनासिक 7

तुम कब जाओगे, अतिथि
CBSE Class 9 Hindi B व्याकरण अनुस्वार एवं अनुनासिक 8

वैज्ञानिक चेतना के वाहक चंद्रशेखर वेंकट रामन्
CBSE Class 9 Hindi B व्याकरण अनुस्वार एवं अनुनासिक 9

कीचड़ का काव्य
CBSE Class 9 Hindi B व्याकरण अनुस्वार एवं अनुनासिक 10

धर्म की आड़
CBSE Class 9 Hindi B व्याकरण अनुस्वार एवं अनुनासिक 11

शुक्रतारे के समान
CBSE Class 9 Hindi B व्याकरण अनुस्वार एवं अनुनासिक 12

अभ्यास प्रश्न

प्रश्नः 1.
नीचे दिए गए शब्दों में उचित स्थान पर अनुस्वार का प्रयोग करते हुए शब्दों का मानक रूप लिखिए –
नालँदा, अँतर, संक्षिप्त, अंबर, चंद्रमा, संघर्ष, निताँत, भाँति, यन्त्र, सँस्कार, अँक, सम्बन्ध, गङ्गा, दीनबन्धु, अन्दर, मन्त्रालय,
खण्डित, छन्द, हिन्दुस्तान, अँगली, तँगी, तन्त्र, तम्बाकू, पँखुड़ी, कम्पन, दंगल, पँकज, दैत्य, बन्डल, धन्धा, पन्चायत, बँजारा।
उत्तर:
नालंदा, अंतर, संक्षिप्त, अंबर, चंद्रमा, संघर्ष, नितांत, भ्रांति, यंत्र, संस्कार, अंक, संबंध, गंगा, दीनबंधु, अंदर, मंत्रालय, खंडित,
छंद, हिंदुस्तान, जंगली, तंगी, तंत्र, तंबाकू, पंखुड़ी, कंपन, दंगल, पंकज, दंत्य, बंडल, धंधा, पंचायत, बंजारा।

प्रश्नः 2.
नीचे दिए गए शब्दों में उचित स्थान पर अनुनासिक का प्रयोग करके शब्दों को पुनः लिखिए-
बंटवारा, संकरा, आंख, हंसमुख, अंगड़ाई, आंचल, सांस, कहां, ऊंट, आंवला, ऊंघना, आंधी, कांटा, गांव, चांदनी, आंसू, ऊंचाई, छंटनी, जांच, टांग, डांट, पहुंचना।
उत्तर:
बँटवारा, सँकरा, आँख, हँसमुख, अंगड़ाई, आँचल, साँस, कहाँ, ऊँट, आँवला, ऊँघना, आँधी, काँटा, गाँव, चाँदनी, आँसू, ऊँचाई, छंटनी, जाँच, टाँग, डाँट, पहुँचना।

प्रश्नः 3.
निम्नलिखित शब्दों में से उस शब्द को चुनिए जिनमें अनुस्वार का प्रयोग होता है-

  1. सगति दाव
  2. पडित महगाई
  3. पजाब, पाव
  4. सास सभावना
  5. आच सुदर
  6. पाचवा निमत्रण
  7. सूघना ससार
  8. सभव धुआ
  9. सावला आनद

उत्तर:

  1. संगति
  2. पंडित
  3. पंजाब
  4. संभावना
  5. सुंदर
  6. निमंत्रण
  7. संसार
  8. संभव
  9. आनंद
  10. संत

प्रश्नः 4.
उचित स्थान पर लगे अनुस्वार वाले शब्द छाँटिए

  1. व्यंजन, कंचन, मंदिर
  2. सयोगं हसं संगम
  3. परतुं प्रंबध व्यंजन
  4. प्रतिबध तुंग मडंली
  5. गांव नोंक नंदन
  6. चंपक कबंल गदंगी
  7. सन्यासी अंधकार आंतक
  8. गोंद अंत्यत आनंद

उत्तर:

  1. कंचन
  2. संगम
  3. व्यंजन
  4. तुंग
  5. नदन
  6. चंपक
  7. अंधकार
  8. आनंद

प्रश्नः 5.
उचित स्थान पर अनुनासिक का प्रयोगकर पुनः लिखिए

  1. यहा
  2. कुआ
  3. भाषाए
  4. काच
  5. साप
  6. हसी
  7. पाच
  8. जाएगे

उत्तर:

  1. यहाँ
  2. कुआँ
  3. भाषाएँ
  4. काँच
  5. साँप
  6. हँसी
  7. पाँच
  8. जाएँगे

प्रश्नः 6.
उचित स्थान पर लगे अनुनासिक वाले शब्द छाँटिए

  1. काँखा आँगन नँदन
  2. मक प्रसँग अँधेरा
  3. चूंघट व्यँजन हँगामा
  4. अभिनंदन छंटनी उमँग
  5. पँडित सुंदर सूंघना
  6. दाँत सायँ जंजीर
  7. चाँदनी आँख खुंखार

उत्तर:

  1. आँगन
  2. अँधेरा
  3. घूघट
  4. छंटनी
  5. सूंघनी
  6. दाँत
  7. खूखार

विभिन्न परीक्षाओं में पूछे गए प्रश्न

प्रश्नः 1.
उस शब्द को चुनिए जिसमें अनुस्वार का प्रयोग होता है-

  1. डाट दाव ढग
  2. यात्रिक पाच गाव
  3. गाठ चदन महगाई
  4. माग साराश लाघना
  5. सास सभव पाव
  6. सूघना वसत रीतिया

उत्तर:

  1. ढंग
  2. यांत्रिक
  3. चंदन
  4. सारांश
  5. संभव
  6. वसंत

प्रश्नः 2.
निम्नलिखित शब्दों में से उचित स्थान पर लगे अनुस्वार वाले शब्द छाँटिए-

  1. संभव, कंचन कंगन
  2. हंस, आनंद, अलकनंदा
  3. दांत, अंधेरा, अंधकार
  4. पंजाब, सिंध, मंडल
  5. अकं, कहीं, दंड
  6. प्रारंभ, कांटा, संगमरमर
  7. संबंध अत्यंत प्रपंच
  8. हंसी हंसी संसार
  9. अनँय गगा अंतर्धान
  10. अलंकार, संस्कार चंचल

उत्तर:

  1. कंचन
  2. अलकनंदा
  3. अंधकार
  4. मंडल
  5. दंड
  6. संगमरमर
  7. संबंध
  8. संसार
  9. अंतर्ध्यान
  10. अलंकार

प्रश्नः 3.
नीचे दिए गए शब्दों में उचित अनुस्वार लगाकर पुनः लिखिए-

  1. सतरी ……………
  2. रग ……………..
  3. मगल ……………..
  4. चपक ……………..
  5. गाधारी ……………..
  6. प्रबध ……………..
  7. कबल ……………..
  8. कपन ……………..
  9. सगति ……………..
  10. वश ……………..

उत्तर:

  1. संतरी
  2. रंग
  3. मंगल
  4. चंपक
  5. गांधारी
  6. प्रबंध
  7. कंबल
  8. कंपन
  9. संगति
  10.  वंश

प्रश्नः 4.
निम्नलिखित शब्दों में उचित अनुनासिक का प्रयोगकर पुनः लिखिए-

  1. साझ ……………..
  2. स्थितिया ……………..
  3. बासुरी ……………..
  4. याऊ ……………..
  5. धुंधला ……………..
  6. पाच ……………..
  7. छूट ……………..
  8. आवला ……………..
  9. फादना ……………..
  10. गाव ……………..

उत्तर:

  1. साँझ
  2. स्थितियाँ
  3. बाँसुरी
  4. याऊँ
  5. धुंधला
  6. पाँच
  7. छुंट
  8. आँवला
  9. फाँदना
  10. गाँव

प्रश्नः 5.
निम्नलिखित शब्दों में उचित स्थान पर लगे अनुनासिक शब्द छाँटिए-

    1. अंधकार, चिड़िया, ब्रह्मांड
  1. आँच, महँगाई, खुशियाँ
  2. बँद, हँसी, मुँह
  3. चंद्रशेखर, ऊँचाई, व्यंजन
  4. पाँचवा, सँगम, बाँसुरी
  5. चंचल, भयँकर, कारवाँ
  6. साँसारिक, कठिनाइयाँ, सूंघना
  7. वसँजय, जहाँ, आँगन
  8. स्वयँ, दाँत, ढूँढ़ना
  9.  चंपक, संयुक्त, अँधेरा
  10. सँभव, हँसना, सिँह
  11. कुंआ, कँपन, आँख
  12. बूंद, आँगन, दिनांक
  13. आँख, चिड़ियाँ, पँखा
  14. अंतिम, घूघट, काँटा

उत्तर:

  1. चिड़ियाँ
  2. खुशियाँ
  3. मुँह
  4. ऊँचाई
  5. बाँसुरी
  6. कारवाँ
  7. सूंघना
  8. आँगन
  9. दाँत
  10. अँधेरा
  11. हँसना
  12. आँख
  13. आँगन
  14. चिड़ियाँ
  15. काँटा

NCERT Solutions for Class 9 Hindi

The post CBSE Class 9 Hindi B व्याकरण अनुस्वार एवं अनुनासिक appeared first on Learn CBSE.

CBSE Class 9 Hindi B व्याकरण नुक्ता

$
0
0

CBSE Class 9 Hindi B व्याकरण नुक्ता

नुक्ता – हिंदी भाषा के शब्द भंडार को समृद्ध बनाने में आगत शब्दों का बड़ा योगदान है। इन शब्दों में अंग्रेजी भाषा के शब्दों की तरह ही उर्दू, फारसी और अरबी शब्दों का भी अपना विशेष योगदान है। इनमें बहुत-से शब्द ऐसे हैं जिनकी उच्चारण शुद्धता और उनका सही अर्थ बनाए रखने के लिए लिखते समय उनके नीचे छोटी बिंदी लगा दी जाती है। इसी बिंदी को नुक्ता कहा जाता है।

नुक्ते का प्रयोग क्यों करें –

1.  नुक्ते का प्रयोग न करने पर कई बार उस शब्द का अर्थ पूरी तरह बदल जाता है। इससे अर्थ का अनर्थ होने की संभावना बढ़ जाती है;
जैसे
राज – (शासन)
तेज – (चमक)
जरा – (बुढ़ापा)
खाना – (भोजन)
खान – (खदान, भंडार)
राज़ – (रहस्य)
तेज़ – (रफ़्तार, वेग)
ज़रा – (थोड़ा)
ख़ाना – (घर, मकान)
ख़ान – (सरदार, स्वामी)

2.  कुछ शब्दों के शुद्ध उच्चारण के लिए नुक्ते का प्रयोग आवश्यक है; जैसे-उर्दू की ध्वनियाँ, क, ख, ग ध्वनियाँ नुक्ता युक्त हैं। इनके उच्चारण भी हिंदी के क, ख, ग से भिन्न हैं।
CBSE Class 9 Hindi B व्याकरण नुक्ता - 1

अभ्यास प्रश्न

प्रश्नः 1.
निम्नलिखित शब्दों में उचित स्थानों पर नुक्ते का प्रयोग करके शब्दों को पुनः लिखिए –

  1. कफन
  2. खौफ
  3. जुकाम
  4. फसल
  5. गुजरना
  6. गुफ्तगू
  7. फैशन
  8. फीस
  9. गजट
  10. फौलाद
  11. फजल
  12. ज्यादा
  13. जरूर
  14. जिंदगी
  15. कयामत
  16. गजब
  17. जख्म
  18. फिल्माना
  19. मजबूर
  20. ग्रेफाइट
  21. फौजी
  22. फारसी
  23. फौरन
  24. खत
  25. फतह
  26. खाक
  27. कर्जा
  28. फतवा
  29. खमीर
  30. जालिम
  31. काबिल
  32. फर्जी
  33. जमानत
  34. फकीर
  35. फजूल
  36. फेल

उत्तरः

  1. कफ़न,
  2. ख़ौफ़
  3. जुकाम
  4. फ़सल
  5. गुज़रना
  6. गुफ़्तगू
  7. फैशन
  8. फ़ीस
  9. गज़ट
  10. फ़ौलाद
  11. फ़जल
  12. ज्यादा
  13. ज़रूर
  14. ज़िदगी
  15. क़यामत
  16. गज़ब
  17. जख़्म
  18. फ़िल्माना
  19. मज़बूर
  20. ग्रेफ़ाइट
  21. फ़ौजी
  22. फ़ारसी
  23. फ़ौरन
  24. ख़त
  25. फ़तह
  26. ख़ाक
  27. कर्जा
  28. फ़तवा
  29. ख़मीर
  30. ज़ालिम
  31. क़ाबिल
  32. फ़र्जी
  33. ज़मानत
  34. फ़कीर
  35. फ़जूल
  36. फ़ेल

प्रश्नः 2.
निम्नलिखित शब्दों में उचित स्थान पर लगे नुक्ते वाले शब्द लिखिए –

  1. फ़लदार, फूलदान, फ़सल
  2. ग़ाय, ग़रीब, गोपाल
  3. ख़रगोश, ख़्याल, जीवन
  4. ज़रा, रोटी, सफ़ल
  5. नज्म, कमज़ोर, क़माऊ
  6. इज़्ज़त, क़ामयाब, क़यामत

उत्तरः

  1. फ़सल
  2. ग़रीब
  3. ख़याल
  4. ज़रा
  5. नज़्म कमज़ोर
  6. इज्ज़त क़यामत

विभिन्न परीक्षाओं में पूछे गए प्रश्न

प्रश्नः 1.
निम्नलिखित शब्दों में उचित स्थान पर लगे नुक्ते वाले शब्द लिखिए –

  1. नफ़रत, बिजली
  2. ताज़गी, हिफ़ाज़त, जयकार
  3. हाज़िर, पकड़
  4. ज़हर जुलम क़िताब
  5. दरवाज़ा जीवन लिफ़ाफ़े
  6. हफ़्ता, जामुन, कागज़
  7. ज़मानत जीवन जिंदगी
  8. रफ़्तार अंग्रेज़ जंगल
  9. तकलीफ़ कर्ज़ क़लम
  10. फ़रमान, मरीज़, फ़ल

उत्तरः

  1. नफ़रत
  2. ताज़गी, हिफ़ाज़त
  3. हाज़िर
  4. ज़हर
  5. दरवाज़ा, लिफ़ाफ़े
  6. हफ़्ता, कागज़
  7. ज़मानत जिंदगी
  8. रफ़्तार अंग्रेज़
  9. तकलीफ़, कर्ज
  10. फ़रमान मरीज़

प्रश्नः 2.
निम्नलिखित शब्दों में उचित स्थान पर नुक्ते लगाकर लिखिए –

  1. जमीन
  2. सब्जी
  3. हुजूर
  4. जरूरत
  5. जुबान
  6. नजीर
  7. मुफलिस
  8. जरदार
  9. मस्जिद
  10. दिलपजीर

उत्तरः

  1. ज़मीन
  2. सब्जी
  3. हुजूर
  4. ज़रूरत
  5. जुबान
  6. नज़ीर
  7. मुफ़लिस
  8. ज़रदार
  9. मस्ज़िद
  10. दिलपज़ीर

NCERT Solutions for Class 9 Hindi

The post CBSE Class 9 Hindi B व्याकरण नुक्ता appeared first on Learn CBSE.

Viewing all 7904 articles
Browse latest View live




Latest Images